i combined everything we possibly have

Pataasin ang iyong marka sa homework at exams ngayon gamit ang Quizwiz!

The nucleotide sequence of one DNA strand of a DNA double helix is 5ʹ-GGATTTTTGTCCACAATCA-3ʹ.What is the sequence of the complementary strand?

5ʹ-TGATTGTGGACAAAAATCC-3

Which of the following statements is true? (a) Ribosomes are large RNA structures composed solely of rRNA. (b) Ribosomes are synthesized entirely in the cytoplasm. (c) rRNA contains the catalytic activity that joins amino acids together. (d) A ribosome binds one tRNA at a time.

(c) rRNA contains the catalytic activity that joins amino acids together

Below is the sequence from the 3′ end of an mRNA. 5′-CCGUUACCAGGCCUCAUUAUUGGUAACGGAAAAAAAAAAAAAA-3′ If you were told that this sequence contains the stop codon for the protein encoded by this mRNA, what is the anticodon on the tRNA in the P site of the ribosome when release factor binds to the A site? (use codon table) (a) 5′-CCA-3′ (b) 5′-CCG-3′ (c) 5′-UGG-3′ (d) 5′-UUA-3′

(a) 5′-CCA-3′

Which of the following statements is false? (a) A mutation that arises in a mother's somatic cell often causes a disease in her daughter. (b) All mutations in an asexually reproducing single-celled organism are passed on to progeny. (c) In an evolutionary sense, somatic cells exist only to help propagate germline cells. (d) A mutation is passed on to offspring only if it is present in the germ line.

(a) A mutation that arises in a mother's somatic cell often causes a disease in her daughter

Compared to the normal situation, in which actin monomers carry ATP, what do you predict would happen if actin monomers that bind a nonhydrolyzable form of ATP were incorporated into actin filaments? (a) Actin filaments would grow longer. (b) Actin filaments would grow shorter because depolymerization would be enhanced. (c) Actin filaments would grow shorter because new monomers could not be added to the filaments. (d) No change, as addition of monomers binding nonhydrolyzable ATP would not affect actin filament length.

(a) Actin filaments would grow longer

The sigma subunit of bacterial RNA polymerase ___________________. (a) contains the catalytic activity of the polymerase. (b) remains part of the polymerase throughout transcription. (c) recognizes promoter sites in the DNA. (d) recognizes transcription termination sites in the DNA.

(c) recognizes promoter sites in the DNA.

Which of the following structures shorten during muscle contraction? (a) myosin filaments (b) flagella (c) sarcomeres (d) actin filaments

(c) sarcomeres

The microtubules in a cell form a structural framework that can have all the following functions except which one? (a) holding internal organelles such as the Golgi apparatus in particular positions in the cell (b) creating long, thin cytoplasmic extensions that protrude from one side of the cell (c) strengthening the plasma membrane (d) moving materials from one place to another inside a cell

(c) strengthening the plasma membrane

Which of the statements below about intermediate filaments is false? (a) They can stay intact in cells treated with concentrated salt solutions. (b) They can be found in the cytoplasm and the nucleus. (c) They can be anchored to the plasma membrane at a cell-cell junction. (d) Each filament is about 10 μm in diameter.

(d) Each filament is about 10 μm in diameter

The pufferfish, Fugu rubripes, has a genome that is one-tenth the size of mammalian genomes. Which of the following statements is not a possible reason for this size difference? (a) Intron sequences in Fugu are shorter than those in mammals. (b) Fugu lacks the repetitive DNA found in mammals. (c) The Fugu genome seems to have lost sequences faster than it has gained sequences over evolutionary time. (d) Fugu has lost many genes that are part of gene families.

(d) Fugu has lost many genes that are part of gene families

Which of the following statements is true? (a) Another name for the fertilized egg cell is the zygote. (b) Diploid organisms reproduce only sexually. (c) All sexually reproducing organisms must have two copies of every chromosome. (d) Gametes have only one chromosome.

(a) Another name for the fertilized egg cell is the zygote

Which of the following statements about gene families is false? (a) Because gene duplication can occur when crossover events occur, genes are always duplicated onto homologous chromosomes. (b) Not all duplicated genes will become functional members of gene families. (c) Whole-genome duplication can contribute to the formation of gene families. (d) Duplicated genes can diverge in both their regulatory regions and their coding regions.

(a) Because gene duplication can occur when crossover events occur, genes are always duplicated onto homologous chromosomes

Which of the following statements is false? (a) The human genome is more similar to the orangutan genome than it is to the mouse genome. (b) A comparison of genomes shows that 90% of the human genome shares regions of conserved synteny with the mouse genome. (c) Primates, dogs, mice, and chickens all have about the same number of genes. (d) Genes that code for ribosomal RNA share significant similarity in all eukaryotes but are much more difficult to recognize in archaea.

(d) Genes that code for ribosomal RNA share significant similarity in all eukaryotes but are much more difficult to recognize in archaea

Which of the following statements is true? (a) Anaphase A must be completed before anaphase B can take place. (b) In cells in which anaphase B predominates, the spindle will elongate much less than in cells in which anaphase A dominates. (c) In anaphase A, both kinetochore and interpolar microtubules shorten. (d) In anaphase B, microtubules associated with the cell cortex shorten.

(d) In anaphase B, microtubules associated with the cell cortex shorten

Which of the following is true of a retrovirus but not of the Alu retrotransposon? (a) It requires cellular enzymes to make copies. (b) It can be inserted into the genome. (c) It can be excised and moved to a new location in the genome. (d) It encodes its own reverse transcriptase.

(d) It encodes its own reverse transcriptase

Which of the following statements most correctly describes meiosis? (a) Meiosis involves two rounds of DNA replication followed by a single cell division. (b) Meiosis involves a single round of DNA replication followed by four successive cell divisions. (c) Meiosis involves four rounds of DNA replication followed by two successive cell divisions. (d) Meiosis involves a single round of DNA replication followed by two successive cell divisions.

(d) Meiosis involves a single round of DNA replication followed by two successive cell divisions

Which of the following statements about vesicle budding from the Golgi is false? (a) Clathrin molecules are important for binding to and selecting cargoes for transport. (b) Adaptins interact with clathrin. (c) Once vesicle budding occurs, clathrin molecules are released from the vesicle. (d) Clathrin molecules act at the cytosolic surface of the Golgi membrane.

(a) Clathrin molecules are important for binding to and selecting cargoes for transport

Which of the following statements about kinetochores is true? (a) Kinetochores assemble onto chromosomes during late prophase. (b) Kinetochores contain DNA-binding proteins that recognize sequences at the telomere of the chromosome. (c) Kinetochore proteins bind to the tubulin molecules at the minus end of microtubules. (d) Kinetochores assemble on chromosomes that lack centromeres.

(a) Kinetochores assemble onto chromosomes during late prophase

Mobile genetic elements are sometimes called "jumping genes," because they move from place to place throughout the genome. The exact mechanism by which they achieve this mobility depends on the genes contained within the mobile element. Which of the following mobile genetic elements carries both a transposase gene and a reverse transcriptase gene? (a) L1 (b) B1 (c) Alu (d) Tn3

(a) L1

Which of the following methods is not used by cells to regulate the amount of a protein in the cell? (a) Genes can be transcribed into mRNA with different efficiencies. (b) Many ribosomes can bind to a single mRNA molecule. (c) Proteins can be tagged with ubiquitin, marking them for degradation. (d) Nuclear pore complexes can regulate the speed at which newly synthesized proteins are exported from the nucleus into the cytoplasm.

(d) Nuclear pore complexes can regulate the speed at which newly synthesized proteins are exported from the nucleus into the cytoplasm

Keratins, neurofilaments, and vimentins are all categories of intermediate filaments. Which of the following properties is not true of these types of intermediate filaments? (a) They strengthen cells against mechanical stress. (b) Dimers associate by noncovalent bonding to form a tetramer. (c) They are found in the cytoplasm. (d) Phosphorylation causes disassembly during every mitotic cycle.

(d) Phosphorylation causes disassembly during every mitotic cycle

Cell movement involves the coordination of many events in the cell. Which of the following phenomena is not required for cell motility? (a) Myosin-mediated contraction at the rear of the moving cell. (b) Integrin association with the extracellular environment. (c) Nucleation of new actin filaments. (d) Release of Ca2+ from the sarcoplasmic reticulum.

(d) Release of Ca2+ from the sarcoplasmic reticulum

Which of the following statements about the globin gene family is true? (a) The globin protein, which can carry oxygen molecules throughout an organism's body, was first seen in ancient vertebrate species about 500 million years ago. (b) The gene duplication that led to the expansion of the globin gene family led to the separation and distribution of globin on many chromosomes in mammals, such that no chromosome has more than a single functional member of the globin gene family. (c) As globin gene family members diverged over the course of evolution, all the DNA sequence variations that have accumulated between family members are within the regulatory DNA sequences that affect when and how strongly each globin gene is expressed. (d) Some of the duplicated globin genes that arose during vertebrate evolution acquired inactivating mutations and became pseudogenes in modern vertebrates.

(d) Some of the duplicated globin genes that arose during vertebrate evolution acquired inactivating mutations and became pseudogenes in modern vertebrates

Which of the following statements about actin is false? (a) ATP hydrolysis decreases actin filament stability. (b) Actin at the cell cortex helps govern the shape of the plasma membrane. (c) Actin filaments are nucleated at the side of existing actin filaments in lamellipodia. (d) The dynamic instability of actin filaments is important for cell movement.

(d) The dynamic instability of actin filaments is important for cell movement

Which of the following statements is true? (a) The mitotic spindle is largely made of intermediate filaments. (b) The contractile ring is made largely of microtubules and actin filaments. (c) The contractile ring divides the nucleus in two. (d) The mitotic spindle helps segregate the chromosomes to the two daughter cells.

(d) The mitotic spindle helps segregate the chromosomes to the two daughter cells

Which of the following statements about the function of the centrosome is false? (a) Microtubules emanating from the centrosome have alternating polarity such that some have their plus end attached to the centrosome while others have their minus end attached to the centrosome. (b) Centrosomes contain hundreds of copies of the γ-tubulin ring complex important for microtubule nucleation. (c) Centrosomes typically contain a pair of centrioles, which is made up of a cylindrical array of short microtubules. (d) Centrosomes are the major microtubule-organizing center in animal cells.

(a) Microtubules emanating from the centrosome have alternating polarity such that some have their plus end attached to the centrosome while others have their minus end attached to the centrosome

Which of the following statements regarding dynamic instability is false? (a) Each microtubule filament grows and shrinks independently of its neighbors. (b) The GTP cap helps protect a growing microtubule from depolymerization. (c) GTP hydrolysis by the tubulin dimer promotes microtubule shrinking. (d) The newly freed tubulin dimers from a shrinking microtubule can be immediately captured by growing microtubules and added to their plus end.

(d) The newly freed tubulin dimers from a shrinking microtubule can be immediately captured by growing microtubules and added to their plus end

The hydrolysis of GTP to GDP carried out by tubulin molecules _________. (a) provides the energy needed for tubulin to polymerize. (b) occurs because the pool of free GDP has run out. (c) tips the balance in favor of microtubule assembly. (d) allows the behavior of microtubules called dynamic instability.

(d) allows the behavior of microtubules called dynamic instability

When introduced into mitotic cells, which of the following is expected to impair anaphase B but not anaphase A? (a) an antibody against myosin (b) ATPγS, a nonhydrolyzable ATP analog that binds to and inhibits ATPases (c) an antibody against the motor proteins that move from the plus end of microtubules to the minus end (d) an antibody against the motor proteins that move from the minus end of microtubules toward the plus end

(d) an antibody against the motor proteins that move from the minus end of microtubules toward the plus end

In which phase of the cell cycle do cells check to determine whether the DNA is fully and correctly replicated? (a) at the transition between G1 and S (b) when cells enter G0 (c) during M (d) at the end of G2

(d) at the end of G2

snRNAs ___________________. (a) are translated into snRNPs. (b) are important for producing mature mRNA transcripts in bacteria. (c) are removed by the spliceosome during RNA splicing. (d) can bind to specific sequences at intron-exon boundaries through complementary base-pairing.

(d) can bind to specific sequences at intron-exon boundaries through complementary base-pairing.

During nervous-system development in Drosophila, the membrane-bound protein Delta acts as an inhibitory signal to prevent neighboring cells from developing into neuronal cells. Delta is involved in _______ signaling. (a) endocrine (b) paracrine (c) neuronal (d) contact-dependent

(d) contact-dependent

Levels of Cdk activity change during the cell cycle, in part because _________. (a) the Cdks phosphorylate each other. (b) the Cdks activate the cyclins. (c) Cdk degradation precedes entry into the next phase of the cell cycle. (d) cyclin levels change during the cycle.

(d) cyclin levels change during the cycle

Transcription in bacteria differs from transcription in a eukaryotic cell because _____________. (a) RNA polymerase (along with its sigma subunit) can initiate transcription on its own. (b) RNA polymerase (along with its sigma subunit) requires the general transcription factors to assemble at the promoter before polymerase can begin transcription. (c) the sigma subunit must associate with the appropriate type of RNA polymerase to produce mRNAs. (d) RNA polymerase must be phosphorylated at its C-terminal tail for transcription to proceed.

(a) RNA polymerase (along with its sigma subunit) can initiate transcription on its own

You discover that the underlying cause of a disease is a protein that is now less stable than the non-disease-causing version of the protein. This change is most likely to be due to ________. (a) a mutation within a gene. (b) a mutation within the regulatory DNA of a gene. (c) gene duplication. (d) horizontal gene transfer.

(a) a mutation within a gene

Which amino acid would you expect a tRNA with the anticodon 5'-CUU-3' to carry? (use codon table) (a) lysine (b) glutamic acid (c) leucine (d) phenylalanine

(a) lysine

A large protein that passes through the nuclear pore must have an appropriate _______. (a) sorting sequence, which typically contains the positively charged amino acids lysine and arginine. (b) sorting sequence, which typically contains the hydrophobic amino acids leucine and isoleucine. (c) sequence to interact with the nuclear fibrils. (d) Ran-interacting protein domain.

(a) sorting sequence, which typically contains the positively charged amino acids lysine and arginine

All intermediate filaments are of similar diameter because _________. (a) the central rod domains are similar in size and amino acid sequence. (b) the globular domains are similar in size and amino acid sequence. (c) covalent bonds among tetramers allow them to pack together in a similar fashion. (d) there is only a single type of intermediate filament in every organism.

(a) the central rod domains are similar in size and amino acid sequence

Proteins that are fully translated in the cytosol and lack a sorting signal will end up in ____. (a) the cytosol. (b) the mitochondria. (c) the interior of the nucleus. (d) the nuclear membrane.

(a) the cytosol

The extent of complementarity of a miRNA with its target mRNA determines _______________. (a) whether the mRNA will be immediately degraded or whether the mRNA will first be transported elsewhere in the cell before degradation. (b) whether the mRNA will be transported to the nucleus. (c) whether RISC is degraded. (d) whether the miRNA synthesizes a complementary strand.

(a) whether the mRNA will be immediately degraded or whether the mRNA will first be transported elsewhere in the cell before degradation

A diploid cell containing 32 chromosomes will make a haploid cell containing ___ chromosomes. (a) 8 (b) 16 (c) 30 (d) 64

(b) 16

You have a piece of DNA that includes the following sequence: 5′-ATAGGCATTCGATCCGGATAGCAT-3′ 3′-TATCCGTAAGCTAGGCCTATCGTA-5′ Which of the following RNA molecules could be transcribed from this piece of DNA? (a) 5′-UAUCCGUAAGCUAGGCCUAUGCUA-3′ (b) 5′-AUAGGCAUUCGAUCCGGAUAGCAU-3′ (c) 5′-UACGAUAGGCCUAGCUUACGGAUA-3′ (d) none of the above

(b) 5′-AUAGGCAUUCGAUCCGGAUAGCAU-3′

Which of the following statements is false? (a) Cytochalasins prevent actin polymerization. (b) Actin filaments are usually excluded from the cell cortex. (c) Integrins are transmembrane proteins that can bind to the extracellular matrix. (d) ARPs can promote the formation of branched actin filaments.

(b) Actin filaments are usually excluded from the cell cortex

Which of the following statements about transport into mitochondria and chloroplasts is false? (a) The signal sequence on proteins destined for these organelles is recognized by a receptor protein in the outer membrane of these organelles. (b) After a protein moves through the protein translocator in the outer membrane of these organelles, the protein diffuses in the lumen until it encounters a protein translocator in the inner membrane. (c) Proteins that are transported into these organelles are unfolded as they are being transported. (d) Signal peptidase will remove the signal sequence once the protein has been imported into these organelles.

(b) After a protein moves through the protein translocator in the outer membrane of these organelles, the protein diffuses in the lumen until it encounters a protein translocator in the inner membrane

Consider the mechanism by which actin and tubulin polymerize. Which of the items below does not describe something similar about the polymerization mechanisms of actin and microtubules? (a) Although both filaments can grow from both ends, the growth rate is faster at the plus ends. (b) Depolymerization initiates at the plus ends of filaments. (c) Nucleotide hydrolysis promotes depolymerization of filaments. (d) Free subunits (actin and tubulin) carry nucleoside triphosphates.

(b) Depolymerization initiates at the plus ends of filaments

Which of the following statements about prokaryotic mRNA molecules is false? (a) A single prokaryotic mRNA molecule can be translated into several proteins. (b) Ribosomes must bind to the 5' cap before initiating translation. (c) mRNAs are not polyadenylated. (d) Ribosomes can start translating an mRNA molecule before transcription is complete.

(b) Ribosomes must bind to the 5' cap before initiating translation.

Molecules to be packaged into vesicles for transport are selected by ________. (a) clathrin. (b) adaptins. (c) dynamin. (d) SNAREs.

(b) adaptins

The formation of a bivalent during meiosis ensures that _______. (a) one chromatid from the mother and one chromatid from the father will segregate together during meiosis I. (b) all four sister chromatids remain together until the cell is ready to divide. (c) recombination will occur between identical sister chromatids. (d) the sex chromosomes, which are not identical, will line up separately at the metaphase plate during meiosis I.

(b) all four sister chromatids remain together until the cell is ready to divide

The number of distinct protein species found in humans and other organisms can vastly exceed the number of genes. This is largely due to ______________. (a) protein degradation. (b) alternative splicing. (c) homologous genes. (d) mutation.

(b) alternative splicing

The piece of RNA below includes the region that codes for the binding site for the initiator tRNA needed in translation. 5′-GUUUCCCGUAUACAUGCGUGCCGGGGGC-3′ Which amino acid will be on the tRNA that is the first to bind to the A site of the ribosome? (use codon table) (a) methionine (b) arginine (c) cysteine (d) valine

(b) arginine

Condensins _________. (a) are degraded when cells enter M phase. (b) assemble into complexes on the DNA when phosphorylated by M-Cdk. (c) are involved in holding sister chromatids together. (d) bind to DNA before DNA replication begins.

(b) assemble into complexes on the DNA when phosphorylated by M-Cdk

Where are proteins in the chloroplast synthesized? (a) in the cytosol (b) in the chloroplast (c) on the endoplasmic reticulum (d) in both the cytosol and the chloroplast

(d) in both the cytosol and the chloroplast

All members of the steroid hormone receptor family _________. (a) are cell-surface receptors. (b) do not undergo conformational changes. (c) are found only in the cytoplasm. (d) interact with signal molecules that diffuse through the plasma membrane.

(d) interact with signal molecules that diffuse through the plasma membrane

Which of the following is not a general mechanism that cells use to maintain stable patterns of gene expression as cells divide? (a) a positive feedback loop, mediated by a transcriptional regulator that activates transcription of its own gene in addition to other cell-type specific genes (b) faithful propagation of condensed chromatin structures as cells divide (c) inheritance of DNA methylation patterns when cells divide (d) proper segregation of housekeeping proteins when cells divide

(d) proper segregation of housekeeping proteins when cells divide

Proteins that are fully translated in the cytosol do not end up in _______. (a) the cytosol. (b) the mitochondria. (c) the interior of the nucleus. (d) transport vesicles.

(d) transport vesicles

The distinct characteristics of different cell types in a multicellular organism result mainly from the differential regulation of the _________________. (a) replication of specific genes. (b) transcription of genes transcribed by RNA polymerase II. (c) transcription of housekeeping genes. (d) proteins that directly bind the TATA box of eukaryotic genes.

(b) transcription of genes transcribed by RNA polymerase II

Explain how the spindle assembly checkpoint ensures that all chromosomes are attached to the spindle and why this checkpoint can delay the onset of anaphase and the exit from mitosis.

The cell makes use of a negative signal: the kinetochores of unattached chromosomes send a "stop" signal to the cell-cycle control system. This signal inhibits further progress through mitosis by blocking the activation of APC/C. Without active APC/C, the sister chromatids remain glued together. Thus, none of the duplicated chromosomes can be pulled apart until every chromosome has been positioned correctly on the spindle. The absence of APC/C also prevents the destruction of cyclins, so that Cdks remain active, prolonging mitosis.

Match the type of intermediate filament with its appropriate location. lamins _________ neurofilaments _________ vimentins _________ keratins _________ A. nerve cells, B. epithelia, C. nucleus, D. connective tissue

C, A, D, B

Place the following in order of size, from the smallest to the largest. A. protofilament B. microtubule C. α-tubulin D. tubulin dimer E. mitotic spindle

C, D, A, B, E

Calmodulin regulates the intracellular Ca2+concentration.

False. Calmodulin senses but does not regulate

For a retrotransposon, reverse transcriptase produces DNA copies using which of the following as a template?

RNA

Compare and contrast mismatch repair and homologous recombination in DNA

Homologous recombination (HR) and mismatch repair (MMR) are inextricably linked. HR pairs homologous chromosomes before meiosis I and is ultimately responsible for generating genetic diversity during sexual reproduction. mismatched nucleotides may be processed by MMR, resulting in nonreciprocal exchange of genetic information (gene conversion).

Describe the process of horizontal gene transfer and state how this form of genetic exchange has significantly impacted human health.

Horizontal gene transfer can take place within an individual organism or organisms of different species. genes that develop resistance to antibiotics area product of horizontal gene transfer.

Which type of molecule has the potential to perform the catalytic act of reproducing itself?

RNA

Transcription is catalysed by:

RNA polymerase

Which of the following is considered a housekeeping protein?

RNA polymerase

Recall how apoptosis balances cell division in an adult tissue such as liver.

If a part of the liver is removed in an adult rat, liver cells proliferate to make up the loss. Conversely, if a rat is treated with the drug phenobarbital, which stimulates liver cell division, the liver enlarges.

Identify the phases that are shortened during the cleavage divisions of early embryos, and explain the effects of these divisions on cell size.

In an early frog embryo, the first cell divisions after fertilization serve to subdivide the giant egg cell into smaller cells as quickly as possible. In such embryonic cell cycles, the G1 and G2 phases are drastically shortened, and the cells do not grow before they divide.

What is the "central dogma"?

Within the cell, genetic information flows from DNA to RNA to protein

Identify where gene regulation can occur in the pathway of gene expression for bacteria and eukaryotes.

In bacteria, transcription regulators usually bind to regulatory DNA sequences close to where RNA polymerase binds. This binding can either activate or repress transcription of the gene. In eukaryotes, regulatory DNA sequences are often separated from the promoter by many thousands of nucleotide pairs. Eukaryotic transcription regulators act in two main ways: (1) they can directly affect the assembly process that requires RNA polymerase and the general transcription factors at the promoter, and (2) they can locally modify the chromatin structure of promoter regions.

List the many steps required for protein production in prokaryotic cells.

Initiation, elongation, and termination. the only major difference between eukaryote and prokaryotes translation is that, rather than being processed and transported, the mRNA has a 5' strand availible immediately after transcription.

State the total size of the human genome, the total number of genes, and the number of chromosomes used to carry this genetic information.

Its 3.2x10^9 nucleotide pairs, spread out over 23 sets of chromosomes.

Compare the general structures of GPCRs and enzyme-coupled receptors such as receptor-tyrosine kinases (RTKs).

Like GPCRs, enzyme-coupled receptors are transmembrane proteins that display their ligand-binding domains on the outer surface of the plasma membrane. Instead of associating with a G protein, however, the cytoplasmic domain of the receptor either acts as an enzyme itself or forms a complex with another protein that acts as an enzyme. Responses to them are typically slow and can act at very low concentrations. - The largest class of enzyme couple receptors consists of receptors with a cytoplasmic domain that functions as a tyrosine kinase, which phosphorylates particular tyrosine's on specific intracellular signaling proteins.

What do cells do during the G1 phase?

Repair DNA damage and destroy cyclins

Compare the movement of DNA-only transposons and retrotransposons.

Retrotransposons appear to only be in eukaryotes. one type of R.T., the L1 element, is transcribed into RNA by a host cell's RNA polyamerase. A double-stranded DNA copy is then made via reverse-transcriptase, an unusual DNA polyamerase that uses RNA as the template. the DNA copy is then free to reintegrate into another site in the genome.

In present-day organisms, which trait or gene is commonly acquired by horizontal gene transfer?

antibiotic resistance

Ras is activated by a Ras-activating protein that does what?

causes Ras to exchange GDP for GTP

Which of the following occurs during metaphase?

chromosomes being aligned under tension at the spindle equator

Describe function of the contractile ring during cytokenesis.

consists mainly of actin and myosin filaments arranged in a ring around the metaphase plate; as the ring contracts, it pulls the membrane inward, thereby dividing the cell in two

The inactivation of M-Cdk leads to which of the following?

exit from mitosis

In eukaryotes, what separate component generally encodes individual protein domains?

exons

Review how a point mutation affects our ability to digest lactose.

Point mutations in regulatory DNA sequences affect the ability to digest lactose

Which statement about positive feedback regulation is accurate?

Positive feedback regulation can generate an abrupt, all-or-none response in which the cell moves from ignoring a signal to responding to it very strongly.

List the stages of M phase.

Prophase: duplicated chromosomes condense; mitotic spindle assembles between centromeres. Prometaphase: breakdown of nuclear envelope; chromosomes now attach to spindle microtubules; undergo active movement. Metaphase: chromosomes are aligned at equator; kinetochore microtubules on each chromatid attach to opposite poles of the spindle. Anaphase: sister chromatids separate and pull towards the poles; kinetochore gets shorter. Telophase: new nuclear envelope reassembles; division of cytoplasm begins with assembly of contractile ring.

What are homologous genes?

genes that are similar in their nucleotide sequence because of a common ancestry

The bonds that form between the anticodon of a tRNA molecule and the three nucleotides of a codon in mRNA are _____

hydrogen bonds that form when the tRNA is at the A site

Why is the transition from metaphase to anaphase so important?

if chromosomes are not lined up properly on the metaphase plate before separation, one daughter cell could receive a surplus or incomplete set of chromosomes; both situations are lethal

Nucleosome:

includes the DNA wrapped around this histone core plus a segment of linker DNA

PI 3-kinase acts by phosphorylating what molecule(s)?

inositol phospholipids

What do both retroviruses and retrotransposons do?

integrate into the host cell genome

The entire period between one M phase and the subsequent M phase is called what?

interphase

Microtubules capture chromosomes by binding specifically to which of the following?

kinetochores on the sister chromatids

What two molecules does inositol phospholipid form, what enzyme catalyzes that production?

made by cleavage of a lipid molecule, present in small quantities in cytosolic leaflet of membrane bilayer. Cleavage of membrane to inositol phospholipid by phospholipase C generates 2 secondary messengers: inositol 1,4,5-triphopshate (IP3) and diacylglycerol (DAG) both important for signal relay

Where do transcription regulators usually bind on a DNA double helix?

major groove

Many of the extracellular signal molecules that regulate inflammation are released locally at the site of infection. What form of cell-cell signaling is being used?

paracrine

Which biochemical reaction is catalyzed by a ribozyme?

peptide bond formation in protein synthesis

Which is NOT specifically targeted for destruction by the proteasome?

phosphorylated proteins

To begin transcription, eukaryotic RNA polymerase recognizes nucleotide sequences in what region of the DNA?

promoter region

The long noncoding RNA Xist silences genes on the X chromosome by doing what?

promoting the formation of heterochromatin

Mitotic chromosome:

replicated chromosomes become condensed into mitotic chromosomes, which are transcriptionally inactive and are designed to be readily distributed between the two daughter cells.

What is the best term for an RNA molecule that possesses catalytic activity?

ribozyme

What are small intracellular signaling molecules often called?

second messengers

When cells respond to an extracellular signal, they most often convert the information carried by this molecule from one form to another. What is this process called?

signal transduction

Most of the genetic variation in the human genome takes what form?

single-nucleotide polymorphisms

Which of the following cell types, when fully differentiated, does not divide to form new cells?

skeletal muscle cells

Explain how aminoacyl-tRNA synthetases attach amino acids to tRNAs.

tRNAs act as adaptor molecules in protein synthesis. Enzymes called aminoacyl-tRNA synthetases covalently link amino acids to their appropriate tRNAs. Each tRNA contains a sequence of three nucleotides, the anticodon, which recognizes a codon in an mRNA through complementary base-pairing.

At which site on the DNA of a gene does RNA polymerase release its newly made RNA?

terminator

Life depends on:

the stable storage, maintenance, and inheritance of genetic information

Although all of the steps involved in expressing a gene can in principle be regulated, what is the most important stage of control for most genes?

transcription initiation

SHORT ANSWER QUESTIONS

SHORT ANSWER QUESTIONS

What do cyclic AMP, Calcium and Nitric Oxide have in common? Explain.

Second messengers are molecules that relay signals received at receptors on the cell surface — such as the arrival of protein hormones, growth factors, etc. — to target molecules in the cytosol and/or nucleus. But in addition to their job as relay molecules, second messengers serve to greatly amplify the strength of the signal. Binding of a ligand to a single receptor at the cell surface may end up causing massive changes in the biochemical activities within the cell.

The _______ makes up about half of the total cell volume of a typical eukaryotic cell. Ingested materials within the cell will pass through a series of compartments called _______ on their way to the _______, which contains digestive enzymes and will ultimately degrade the particles and macromolecules taken into the cell and will also degrade worn-out organelles. The ________ has a cis and trans face and receives proteins and lipids from the _______, a system of interconnected sacs and tubes of membranes that typically extends throughout the cell. Cytosol, Golgi apparatus, nucleus, endoplasmic reticulum, lysosome, peroxisomes, endosomes, mitochondria, plasma membrane

The cytosol makes up about half of the total cell volume of a typical eukaryotic cell. Ingested materials within the cell will pass through a series of compartments called endosomes on their way to the lysosome, which contains digestive enzymes and will ultimately degrade the particles and macromolecules taken into the cell and will also degrade worn-out organelles. The Golgi apparatus has a cis and trans face and receives proteins and lipids from the endoplasmic reticulum, a system of interconnected sacs and tubes of membranes that typically extends throughout the cell.

Recall why nitric oxide acts as a paracrine signal only on cells near its site of synthesis.

The distance NO diffuses is limited by its reaction with oxygen and water in the extracellular environment, which convert NO into nitrates and nitrites quickly Target cells; NO binds and activates guanylyl cyclase, stimulating formation of cyclic GMP from GTP. Cyclic GMP is a second messenger in the NO signaling chain. Viagra enhances erection by blocking the enzyme that degrades cyclic GMP, prolonging the NO signal

The cell-cycle control system initiates chromosome segregation only after which of the following has occurred?

The duplicated chromosomes are correctly aligned on the mitotic spindle

Describe chromatin remodeling. How does this occur and why?

The dynamic modification of chromatin architecture to allow access of condensed genomic DNA to the regulatory transcription machinery proteins, and thereby control gene expression.

Present evidence that supports the occurrence of whole genome duplication events.

The frog, xenopus, includes species that differ dramatically in DNA content. some are diploid, some are tetraploid or octoploid. whole genome duplication is also shown in several plants, such as sugarcane.

Contrast the origin recognition complex (ORC) and the prereplicative complex in terms of composition and when each assembles on the DNA.

The origin recognition complex (ORC) remains perched on the replication origins throughout the cell cycle. To prepare the DNA for replication, the ORD recruits a protein called Cdc6, whose concentration rises early in G1. Together, these proteins load the DNA helicases that will ultimately open up the double helix at the origin of replication. Once this prereplicative complex is in place, the replication origin is loaded and ready to go.

steps of gene expression

The process of gene expression includes 1) transcription, the conversion of DNA to RNA, and 2) translation, the conversion of RNA to proteins.

Summarize how the Fugu genome differs from the human genome in terms of size and approximate number of genes.

A Fugu(pufferfish) genome is about 1/8 the size of a mammilian genome because it is missing nearly all the repetitive DNA, and its introns are shorter. The positions of introns are still perfectly conserved.

Present the components of a typical transposon.

A general name for short segments of DNA that can move from one location to another in the genome. Also known as mobile genetic elements. Bacteria contain DNA- only transposons. They encode a transposase that mediates its movement. These enzymes recognize and act on unique DNA sequences that are present on the mobile genetic elements that code for the transposase.

What are tranposons and what impact do they have on the genome?

A general name for short segments of DNA that can move from one location to another in the genome. Also known as mobile genetic elements.transposons can drive the evolution of genomes by facilitating the translocation of genomic sequences, the shuffling of exons, and the repair of double-stranded breaks. Insertions and transposition can also alter gene regulatory regions and phenotypes.

Outline the key differences between the synthesis of the leading and lagging strands.

A leading strand is the strand which is synthesized in the 5'-3'direction while a lagging strand is the strand which is synthesized in the 3'-5' direction. ... The leading strand is synthesized continuously while a lagging strand is synthesized in fragments which are called Okazaki fragments.

List several intracellular signaling proteins activated by RTKs.

A phospholipase C that functions in the same way as the phospholipase C activated by GPCRs to trigger the inositol phospholipid signaling pathway. Another intracellular signaling protein is a small GTP-binding protein called Ras.

Name the basic components needed for an extracellular signal molecule to change the behavior of a target cell, and identify the site at which the primary step in signal transduction takes place.

The signal molecule needs a receptor protein to start the primary step of signal transduction: The receptor recognizes the extracellular signal, generates a new intracellular signal, this signal is passed downstream from one signal molecule to the next. Each transition activates or generates a the next signaling molecule in the pathway. The final response is usually a metabolic enzyme, a change in cytoskeleton confirmation, or a gene switched on or off. Intracellular signaling molecules leads to effector proteins (metabolic enzymes, cytoskeletal proteins, transcription regulators, etc) that generate the response (altered metabolism, altered shape or movement, altered gene expression, etc)

Discuss the various ways that organisms benefit from gene regulation.

A typical eukaryotic cell expresses only a fraction of its genes, and the distinct types of cells in multicellular organisms arise because different sets of genes are expressed as cells differentiate.

Match the target of the G protein with the appropriate signaling outcome. adenylyl cyclase ________ ion channels _________ phospholipase C _________ A. cleavage of inositol phospholipids, B. increase in cAMP levels, C. changes in membrane potential

B, C, A

Irradiated mammalian cells usually stop dividing and arrest at a G1 checkpoint. Place the following events in the order in which they occur. A. production of p21 B. DNA damage C. inhibition of cyclin-Cdk complexes D. accumulation and activation of p53

B, D, A, C

Which of these method(s) of controlling eukaryotic gene expression is NOT employed in prokaryotic cells? A. controlling how often a gene is transcribed B. controlling how an RNA transcript is spliced C. controlling which mRNAs are exported from the nucleus to the cytosol D. controlling which mRNAs are translated into protein by the ribosomes E. controlling how rapidly proteins are destroyed once they are made

B. controlling how an RNA transcript is spliced, C. controlling which mRNAs are exported from the nucleus to the cytosol

Indicate how Ras can fuel uncontrolled proliferation in cancer.

Before Ras was discovered in normal cells, a mutant form of the protein was found in human cancer cells. This mutation inactivates the GTPase activity of Ras, so that the protein cannot shut itself off, promoting uncontrolled cell proliferation and the development of cancer. About 30% of human cancers contain such activating mutations in a Ras gene.

Explain the process of splicing that produces mature eukaryotic mRNA.

(Pictured is alternative splicing) Introns are removed from the RNA transcripts in the nucleus by RNA splicing, a reaction catalyzed by small ribonucleoprotein complexes known as snRNPs. Splicing removes the introns from the RNA and joins together the exons—often in a variety of combinations, allowing multiple proteins to be produced from the same gene.

Which of the following molecules of RNA would you predict to be the most likely to fold into a specific structure as a result of intramolecular base-pairing? (a) 5′-CCCUAAAAAAAAAAAAAAAAUUUUUUUUUUUUUUUUAGGG-3′ (b) 5′-UGUGUGUGUGUGUGUGUGUGUGUGUGUGUGUGUGUGUGUG-3′ (c) 5′-AAAAAAAAAAAAAAAAAAAAAAAAAAAAAAAAAAAAAAAA-3′ (d) 5′-GGAAAAGGAGAUGGGCAAGGGGAAAAGGAGAUGGGCAAGG-3′

(a) 5′-CCCUAAAAAAAAAAAAAAAAUUUUUUUUUUUUUUUUAGGG-3′

Which of the following generalities about genomes is true? (a) All vertebrate genomes contain roughly the same number of genes. (b) All unicellular organisms contain roughly the same number of genes. (c) The larger an organism, the more genes it has. (d) The more types of cell an organism has, the more genes it has.

(a) All vertebrate genomes contain roughly the same number of genes

Genes in eukaryotic cells often have intronic sequences coded for within the DNA. These sequences are ultimately not translated into proteins. Why? (a) Intronic sequences are removed from RNA molecules by the spliceosome, which works in the nucleus. (b) Introns are not transcribed by RNA polymerase. (c) Introns are removed by catalytic RNAs in the cytoplasm. (d) The ribosome will skip over intron sequences when translating RNA into protein.

(a) Intronic sequences are removed from RNA molecules by the spliceosome, which works in the nucleus

What is the role of the nuclear localization sequence in a nuclear protein? (a) It is bound by cytoplasmic proteins that direct the nuclear protein to the nuclear pore. (b) It is a hydrophobic sequence that enables the protein to enter the nuclear membranes. (c) It aids in protein unfolding so that the protein can thread through nuclear pores. (d) It prevents the protein from diffusing out of the nucleus through nuclear pores.

(a) It is bound by cytoplasmic proteins that direct the nuclear protein to the nuclear pore

Outline how a set of mutant RTKs can be used to determine which tyrosines serve as docking sites for the intracellular signaling proteins that propagate the signal.

To determine which tyrosine binds to a specific intracellular signaling protein, a series of mutant receptors is constructed. In the mutants shown, tyrosines Tyr2 or Tyr3 have been replaced, one at a time, by phenylalanine, thereby preventing phosphorylation at that site. As a result, the mutant receptors no longer bind to one of the intracellular signaling proteins. The effect on the cell's response to the signal can then be determined. It is important that the mutant receptor is tested in a cell that does not have its own normal receptors for the signal molecule.

Recall how signals transmitted by RTKs can be terminated.

To help terminate the response, the tyrosine phosphorylation's are reversed by adding tyrosine phosphatases which removes the phosphates that were added to the tyrosines of both the RTKs and other intracellular signaling proteins in response to the extracellular signal. In some cases, activated RTKs are inactivated in a more brutal way: they are dragged into the interior of the cell by endocytosis and then destroyed by digestion in lysosomes.

Outline how GPCRs in the photoreceptors of the retina transmit an extremely rapid signal in response to stimulation by light.

Rhodopsin, a G protein coupled light receptor, when stimulated by light, activated the G protein transducin. The activated α subunit of transducin activates an intracellular signaling cascade, causing cation channels to close in the plasma membrane. A voltage change is produced across the membrane, altering neurotransmitter release and leads to nerve impulses sent to the brain. Rod is stimulated by light, signal is sent from rhodopsin through the cytosol, to ion channels that allow the positive ions to flow through PM of outer segment. Cation channels close in response to cytosolic signal, this produces a change in membrane potential of rod cell.

The signal to commence DNA replication comes from which of the following?

S-Cdk

Summarize what has been revealed by the comparison of human and Neanderthal DNA.

Scientists have discovered which genomic regions have undergone change.

Which of the following statements about what we have learned by comparing the modern-day human genome to other genomes is true? (a) Modern humans whose ancestors come from Europe or Asia share up to 4 percent of their genome with Neanderthals. (b) Accelerated changes, which were found when comparing the human genome to other mammalian genomes, were not found when comparing the modern-day human genome to the Neanderthal genome. (c) The human genome is far more gene-dense than the yeast genome. (d) In syntenic regions of the human and mouse genomes, both gene order and the placements of more than 95% of the mobile genetic elements are conserved.

(a) Modern humans whose ancestors come from Europe or Asia share up to 4 percent of their genome with Neanderthals

Which of the following statements about peroxisomes is false? (a) Most peroxisomal proteins are synthesized in the ER. (b) Peroxisomes synthesize phospholipids for the myelin sheath. (c) Peroxisomes produce hydrogen peroxide. (d) Vesicles that bud from the ER can mature into peroxisomes.

(a) Most peroxisomal proteins are synthesized in the ER

Which of the following statements about microtubules is true? (a) Motor proteins move in a directional fashion along microtubules by using the inherent structural polarity of a protofilament. (b) The centromere nucleates the microtubules of the mitotic spindle. (c) Because microtubules are subject to dynamic instability, they are used only for transient structures in a cell. (d) ATP hydrolysis by a tubulin heterodimer is important for controlling the growth of a microtubule.

(a) Motor proteins move in a directional fashion along microtubules by using the inherent structural polarity of a protofilament

Which of the following statements about the cell cycle is false? (a) Once a cell decides to enter the cell cycle, the time from start to finish is the same in all eukaryotic cells. (b) An unfavorable environment can cause cells to arrest in G1. (c) A cell has more DNA during G2 than it did in G1. (d) The cleavage divisions that occur in an early embryo have short G1 and G2 phases.

(a) Once a cell decides to enter the cell cycle, the time from start to finish is the same in all eukaryotic cells

Which of the following statements about phagocytic cells in animals is false? (a) Phagocytic cells are important in the gut to take up large particles of food. (b) Phagocytic cells scavenge dead and damaged cells and cell debris. (c) Phagocytic cells can engulf invading microorganisms and deliver them to their lysosomes for destruction. (d) Phagocytic cells extend pseudopods that surround the material to be ingested.

(a) Phagocytic cells are important in the gut to take up large particles of food

Which of the following statements about molecular switches is false? (a) Phosphatases remove the phosphate from GTP on GTP-binding proteins, turning them off. (b) Protein kinases transfer the terminal phosphate from ATP onto a protein. (c) Serine/threonine kinases are the most common types of protein kinase. (d) A GTP-binding protein exchanges its bound GDP for GTP to become activated.

(a) Phosphatases remove the phosphate from GTP on GTP-binding proteins, turning them off

Which of the following statements about pseudogenes is false? (a) Pseudogenes code for microRNAs. (b) Pseudogenes share significant nucleotide similarity with functional genes. (c) Pseudogenes are no longer expressed in the cell. (d) There are estimated to be approximately 20,000 pseudogenes in the human genome.

(a) Pseudogenes code for microRNAs

Which of the following statements about the benefits of sexual reproduction is false? (a) Sexual reproduction permits enhanced survival because the gametes that carry alleles enhancing survival in harsh environments are used preferentially during fertilization. (b) Unicellular organisms that can undergo sexual reproduction have an increased ability to adapt to harsh environments. (c) Sexual reproduction reshuffles genes, which is thought to help species survive in novel or varying environments. (d) Sexual reproduction can speed the elimination of deleterious alleles.

(a) Sexual reproduction permits enhanced survival because the gametes that carry alleles enhancing survival in harsh environments are used preferentially during fertilization

Recall the general structure of a G protein and describe how the protein responds when activated by a GPCR.

3 G Protein Subunits - α, β, γ - βγ complex, tethered by γ to plasma membrane - α also tethered to plasma membrane (via short lipid tails) · α - unstimulated, inactive when GDP is bound (attached to βγ) · when receptor binds to GPCR, receptor is activated and changes confirmation · GCPR binds to α, GDP dissociation & exchanged for GTP · Activated α subunit breaks away from βγ (also activated) · Both activated can interact directly with target proteins that relay signals to other destinations in the cell

Rank the following types of cell signaling from 1 to 4, with 1 representing the type of signaling in which the signal molecule travels the least distance and 4 the type of signaling in which the signal molecule travels the largest distance. ______ paracrine signaling ______ contact-dependent signaling ______ neuronal signaling ______ endocrine signaling

3, 1, 2, 4

Meiosis is a highly specialized cell division in which several events occur in a precisely defined order. Please order the meiotic events listed below. 1. loss of cohesins near centromeres 2. chromatid pairing 3. chromosome condensation 4. chromosome replication 5. degradation of cohesins bound to chromosome arms 6. formation of chiasmata (chiasmata = plural of chiasma) 7. homolog pairing 8. alignment of chromosomes at the metaphase plate

4, 2, 7, 6, 3, 8, 5, 1

Describe the post-transcriptional modiications of 5' cap and 3' poly A tail to eukaryotic mRNA.

5' cap modification-protects mRNA from degradation. In addition, initiation factors involved in protein synthesis recognize the cap to help initiate translation by ribosomes. 3' poly A tail modification- protects the mRNA from degradation, aids in the export of the mature mRNA to the cytoplasm, and is involved in binding proteins involved in initiating translation.

List and briefly define three post transcriptional controls.

5' cap modification-protects mRNA from degradation. In addition, initiation factors involved in protein synthesis recognize the cap to help initiate translation by ribosomes. 3' poly A tail modification- protects the mRNA from degradation, aids in the export of the mature mRNA to the cytoplasm, and is involved in binding proteins involved in initiating translation. long noncoding RNAs-The complex silences certain gene expression by cleaving the mRNA molecules coding the target genes. This cleavage results in mRNA fragments that are further degraded by cellular exonucleases.

Place the numbers 1-8 next to the letter headings to indicate the normal order of events. A. alignment of the chromosomes at the spindle equator B. attachment of spindle microtubules to chromosomes C. breakdown of nuclear envelope D. pinching of cell in two E. separation of two centrosomes and initiation of mitotic spindle assembly F. re-formation of the nuclear envelope G. condensation of the chromosomes H. separation of sister chromatids

5, 4, 3, 8, 2, 7, 1, 6

Use examples to differentiate between somatic and germ line mutations.

A germline mutation occurs before conception. It is present in the zygote. So every cell in the fetus will have it. The offspring of that fetus can inherit it. A somatic mutation occurs after conception. It is in one cell in the body, not all cells. Though that cell can grow and form more cells with it. Usually, it'll be in something like blood, muscle, gut, lung. So it isn't passed on to offspring. Examples of germline mutations include inherited diseases. All cells in the body have a mutation. That includes sex cells, so the mutation's transmitted to the next generation. Examples of somatic mutations include changes in a body cell that causes it to turn cancerous (spontaneous cancer).

Which of the situations below will enhance microtubule shrinkage? (a) addition of a drug that inhibits GTP exchange on free tubulin dimers (b) addition of a drug that inhibits hydrolysis of the GTP carried by tubulin dimers (c) addition of a drug that increases the affinity of tubulin molecules carrying GDP for other tubulin molecules (d) addition of a drug that blocks the ability of a tubulin dimer to bind to γ- tubulin

(a) addition of a drug that inhibits GTP exchange on free tubulin dimers

MicroRNAs ____________________. (a) are produced from a precursor miRNA transcript. (b) are found only in humans. (c) control gene expression by base-pairing with DNA sequences. (d) can degrade RNAs by using their intrinsic catalytic activity

(a) are produced from a precursor miRNA transcript

Which of the following precede the re-formation of the nuclear envelope during M phase in animal cells? (a) assembly of the contractile ring (b) decondensation of chromosomes (c) reassembly of the nuclear lamina (d) transcription of nuclear genes

(a) assembly of the contractile ring

Programmed cell death occurs ________. (a) by means of an intracellular suicide program. (b) rarely and selectively only during animal development. (c) only in unhealthy or abnormal cells. (d) only during embryonic development.

(a) by means of an intracellular suicide program

The principal microtubule-organizing center in animal cells is the _______. (a) centrosome. (b) centromere. (c) kinetochore. (d) cell cortex.

(a) centrosome

Which of the following mechanisms is not directly involved in inactivating an activated RTK? (a) dephosphorylation by serine/threonine phosphatases (b) dephosphorylation by protein tyrosine phosphatases (c) removal of the RTK from the plasma membrane by endocytosis (d) digestion of the RTK in lysosomes

(a) dephosphorylation by serine/threonine phosphatases

miRNAs, tRNAs, and rRNAs all _____________. (a) do not code for proteins. (b) act in the nucleus. (c) are packaged with other proteins to form RISC. (d) form base pairs with mRNA molecules.

(a) do not code for proteins

Cells in the G0 state _______. (a) do not divide. (b) cannot re-enter the cell cycle. (c) have entered this arrest state from either G1 or G2. (d) have duplicated their DNA.

(a) do not divide

Which of the following processes is not thought to contribute to diversity in the genome of human individuals? (a) exon shuffling (b) single-nucleotide polymorphisms (c) CA repeats (d) duplication and deletion of large blocks of sequence

(a) exon shuffling

Mitogens are _____. (a) extracellular signals that stimulate cell division. (b) transcription factors important for cyclin production. (c) kinases that cause cells to grow in size. (d) produced by mitotic cells to keep nearby neighboring cells from dividing.

(a) extracellular signals that stimulate cell division

Which of the following does not occur during M phase in animal cells? (a) growth of the cell (b) condensation of chromosomes (c) breakdown of nuclear envelope (d) attachment of chromosomes to microtubules

(a) growth of the cell

Cytokinesis in animal cells _______. (a) requires ATP. (b) leaves a small circular "scar" of actin filaments on the inner surface of the plasma membrane. (c) is often followed by phosphorylation of integrins in the plasma membrane. (d) is assisted by motor proteins that pull on microtubules attached to the cell cortex.

(a) requires ATP

Which of the following items is not important for flagellar movement? (a) sarcoplasmic reticulum (b) ATP (c) dynein (d) microtubules

(a) sarcoplasmic reticulum

A friend declares that chromosomes are held at the metaphase plate by microtubules that push on each chromosome from opposite sides. Which of the following observations does not support your belief that the microtubules are pulling on the chromosomes? (a) the jiggling movement of chromosomes at the metaphase plate (b) the way in which chromosomes behave when the attachment between sister chromatids is severed (c) the way in which chromosomes behave when the attachment to one kinetochore is severed (d) the shape of chromosomes as they move toward the spindle poles at anaphase

(a) the jiggling movement of chromosomes at the metaphase plate

Which of the following regions of the genome is the least likely to be conserved over evolutionary time? (a) the upstream regulatory region of a gene that encodes the region conferring tissue specificity (b) the upstream regulatory region of a gene that binds to RNA polymerase (c) the portion of the genome that codes for proteins (d) the portion of the genome that codes for RNAs that are not translated into protein

(a) the upstream regulatory region of a gene that encodes the region conferring tissue specificity

The modular nature of the Eve gene's regulatory region means that ______. (a) there are seven regulatory elements and each element is sufficient for driving expression in a single stripe. (b) all the regulatory elements for each stripe use the same transcriptional activators. (c) the E. coli LacZ gene is normally only expressed in a single stripe—unlike Eve, which is expressed in seven stripes. (d) transcription regulators only bind to the stripe 2 regulatory DNA segment in stripe 2.

(a) there are seven regulatory elements and each element is sufficient for driving expression in a single stripe

When using a repeating trinucleotide sequence (such as 5′-AAC-3′) in a cellfree translation system, you will obtain: (a) three different types of peptides, each made up of a single amino acid (b) peptides made up of three different amino acids in random order (c) peptides made up of three different amino acids, each alternating with each other in a repetitive fashion (d) polyasparagine, as the codon for asparagine is AAC

(a) three different types of peptides, each made up of a single amino acid

The nucleotide sequences between individuals differ by 0.1%, yet the human genome is made up of about 3 × 109 nucleotide pairs. Which of the following statements is false? (a) In most human cells, the homologous autosomes differ from each other by 0.1%. (b) All changes between human individuals are single-nucleotide polymorphisms. (c) Any two individuals (other than identical twins) will generally have more than 3 million genetic differences in their genomes. (d) Much of the variation between human individuals was present 100,000 years ago, when the human population was small.

(b) All changes between human individuals are single-nucleotide polymorphisms

You create cells with a version of Cdc6 that cannot be phosphorylated and thus cannot be degraded. Which of the following statements describes the likely consequence of this change in Cdc6? (a) Cells will enter S phase prematurely. (b) Cells will be unable to complete DNA synthesis. (c) The origin recognition complex (ORC) will be unable to bind to DNA. (d) Cdc6 will be produced inappropriately during M phase.

(b) Cells will be unable to complete DNA synthesis

Name the membrane-enclosed compartments in a eukaryotic cell where each of the functions listed below takes place. A. photosynthesis B. transcription C. oxidative phosphorylation D. modification of secreted proteins E. steroid hormone synthesis F. degradation of worn-out organelles G. new membrane synthesis H. breakdown of lipids and toxic molecules

A photosynthesis = chloroplast B. transcription = nucleus C. oxidative phosphorylation = mitochondrion D. modification of secreted proteins = Golgi apparatus and rough endoplasmic reticulum (ER) E. steroid hormone synthesis = smooth ER F. degradation of worn-out organelles = lysosome G. new membrane synthesis = ER H. breakdown of lipids and toxic molecules = peroxisome

What is a polyribosome? How does it function?

A polyribosome (or polysome or ergasome) is a group of ribosomes bound to an mRNA molecule like "beads" on a "thread". It consists of a complex of an mRNA molecule and two or more ribosomes that act to translate mRNA instructions into polypeptides.

How many possible nucleotide sequences are there for a stretch of single-stranded DNA that is N nucleotides long?

A single-stranded DNA molecule that is N nucleotides long can have any one of 4N possible sequences.

Contrast the cell signaling systems used by plants and animals.

A spindly weed has hundreds of genes encoding receptor serine/threonine kinases but they are structurally distinct from the ones found in animal cells. In contrast to animal cells, plant cells seem not to use RTKs, steroid-hormone-type nuclear receptors, or cyclic AMP, and they seem to use few GPCRs.

Describe the structure and function of tRNA.

A tRNA molecule consists of a single RNA strand that is only about 80 nucleotides long. If it were to be flattened into one plane to reveal its base pairing, a tRNA molecule would look like a cloverleaf. tRNA is a vital component of the translation process that produces polypeptides.

Distinguish the appearance of cell necrosis from that of apoptosis.

Cell necrosis: eruption triggers a potentially damaging inflammatory response. By contrast, a cell that undergoes apoptosis dies neatly, without damaging its neighbors. A cell in the throes of apoptosis may develop irregular bulges or blebs on its surface, then it shrinks and condenses.

Differentiate the types of cell responses that occur rapidly with those that take minutes or hours to execute.

Cell response times vary based on if the proper proteins are available within the cell to carry out the response. Acetylcholine on a skeletal muscle cell causes it to contract in milliseconds because the proteins affected by the signal are already present in the cell. Growth and division responses take more time, hours, because the signals require change in gene expression and production of proteins.

Cells can signal to each other in various ways. A signal that must be relayed to the entire body is most efficiently sent by ______ cells, which produce hormones that are carried throughout the body through the bloodstream. On the other hand, ______ methods of cell signaling do not require the release of a secreted molecule and are used for very localized signaling events. During ______ signaling, the signal remains in the neighborhood of the secreting cell and thus acts as a local mediator on nearby cells. Finally, ______ signaling involves the conversion of electrical impulses into a chemical signal. Cells receive signals through a _______, which can be an integral membrane protein or can reside inside the cell. Amplification, G protein, phosphorylation, contact-dependent, K+ channel, receptor, endocrine, neuronal, target, epithelial, paracrine

Cells can signal to each other in various ways. A signal that must be relayed to the entire body is most efficiently sent by endocrine cells, which produce hormones that are carried throughout the body through the bloodstream. On the other hand, contact-dependent methods of cell signaling do not require the release of a secreted molecule and are used for very localized signaling events. During paracrine signaling, the signal remains in the neighborhood of the secreting cell and thus acts as a local mediator on nearby cells. Finally, neuronal signaling involves the conversion of electrical impulses into a chemical signal. Cells receive signals through a receptor, which can be an integral membrane protein or can reside inside the cell.

Explain the mechanisms that create specialized cell types and promote cell memory.

Cells in multicellular organisms have mechanisms that enable their progeny to "remember" what type of cell they should be. A prominent mechanism for propagating cell memory relies on transcription regulators that perpetuate transcription of their own gene—a form of positive feedback.

Outline how a comparison of the nucleotide sequences from two closely related organisms can be used to reconstruct the amino acid sequence of a protein from their extinct, common ancestor.

Changes in genomes have produced only minor differences, and a large percentage of nucleotides match.

Compare cohesins and condensins in terms of structure, function, and how and when they assemble onto chromosomal DNA.

Cohesions: protein complex that holds sister chromatids together. They assemble along the length of each chromatid as the DNA is replicated. This cohesion between sister chromatids is crucial for proper chromosome segregation, and it is broken completely only in late mitosis to allow the sisters to be pulled apart by the mitotic spindle. Condensins: these protein complexes help carry our chromosome condensation, which reduces mitotic chromosomes to compact bodies that can be more easily segregated within the crowded confines of the dividing cell. The assembly of condensin complexes onto the DNA is triggered by the phosphorylation of condensins by M-Cdk. Condensins and cohesions are structurally related, and both are thought to form ring structures around chromosomal DNA. However, whereas cohesions encircle the two sister chromatids, tying them together, condensins assemble along each individual sister chromatid, helping each of these double helices to coil up into a more compact form.

Chromatin:

Complex of DNA and proteins that makes up the chromosomes in a eukaryotic cell

Which of the following statements about retroviruses is false? (a) Retroviruses are packaged with a few molecules of reverse transcriptase in each virus particle. (b) Retroviruses use the host-genome integrase enzyme to create the provirus. (c) The production of viral RNAs can occur long after the initial infection of the host cell by the retrovirus. (d) Viral RNAs are translated by host-cell ribosomes to produce the proteins required for the production of viral particles.

(b) Retroviruses use the host-genome integrase enzyme to create the provirus

You have a bacterial strain with a mutation that removes the transcription termination signal from the Abd operon. Which of the following statements describes the most likely effect of this mutation on Abd transcription? (a) The Abd RNA will not be produced in the mutant strain. (b) The Abd RNA from the mutant strain will be longer than normal. (c) Sigma factor will not dissociate from RNA polymerase when the Abd operon is being transcribed in the mutant strain. (d) RNA polymerase will move in a backward fashion at the Abd operon in the mutant strain.

(b) The Abd RNA from the mutant strain will be longer than normal

Which of the following statements about the Ey transcriptional regulator is false? (a) Expression of Ey in cells that normally form legs in the fly will lead to the formation of an eye in the middle of the legs. (b) The Ey transcription factor must bind to the promoter of every eyespecific gene in the fly. (c) Positive feedback loops ensure that Ey expression remains switched on in the developing eye. (d) A homolog of Ey is found in vertebrates; this homolog is also used during eye development.

(b) The Ey transcription factor must bind to the promoter of every eyespecific gene in the fly

What do you predict would happen if you replace the Lac operator DNA from the Lac operon with the DNA from the operator region from the tryptophan operon? (a) The presence of lactose will not cause allosteric changes to the Lac repressor. (b) The Lac operon will not be transcribed when tryptophan levels are high. (c) The lack of glucose will no longer allow CAP binding to the DNA. (d) RNA polymerase will only bind to the Lac promoter when lactose is present.

(b) The Lac operon will not be transcribed when tryptophan levels are high

Which of the following statements about skeletal muscle contraction is false? (a) When a muscle cell receives a signal from the nervous system, voltagegated channels open in the T-tubule membrane. (b) The changes in voltage across the plasma membrane that occur when a muscle cell receives a signal from the nervous system cause an influx of Ca2+ into the sarcoplasmic reticulum, triggering a muscle contraction. (c) A change in the conformation of troponin leads to changes in tropomyosin such that it no longer blocks the binding of myosin heads to the actin filament. (d) During muscle contraction, the Z discs move closer together as the myosin heads walk toward the plus ends of the actin filaments.

(b) The changes in voltage across the plasma membrane that occur when a muscle cell receives a signal from the nervous system cause an influx of Ca2+ into the sarcoplasmic reticulum, triggering a muscle contraction

Investigators performed nuclear transplant experiments to determine whether DNA is altered irreversibly during development. Which of the following statements about these experiments is true? (a) Because the donor nucleus is taken from an adult animal, the chromosomes from the nucleus must undergo recombination with the DNA in the egg for successful development to occur. (b) The embryo that develops from the nuclear transplant experiment is genetically identical to the donor of the nucleus. (c) The meiotic spindle of the egg must interact with the chromosomes of the injected nuclei for successful nuclear transplantation to occur. (d) Although nuclear transplantation has been successful in producing embryos in some mammals with the use of foster mothers, evidence of DNA alterations during differentiation has not been obtained for plants.

(b) The embryo that develops from the nuclear transplant experiment is genetically identical to the donor of the nucleus

Which of the following statements about the genetic code is correct? (a) All codons specify more than one amino acid. (b) The genetic code is redundant. (c) All amino acids are specified by more than one codon. (d) All codons specify an amino acid.

(b) The genetic code is redundant

Which of the following statements about mRNA half-life is false? (a) The half-life of mRNAs produced from different genes will vary more than the half-life of mRNAs produced from the same gene. (b) The half-life of most eukaryotic-cell mRNAs is >24 hours. (c) The half-life of most bacterial mRNAs is shorter than the half-life of a typical eukaryotic mRNA. (d) The 5' and 3' untranslated regions of an mRNA often contain specific sequences that determine the lifetime of the mRNA molecule.

(b) The half-life of most eukaryotic-cell mRNAs is >24 hours

Which of the following statements is false? (a) DNA synthesis begins at origins of replication. (b) The loading of the origin recognition complexes (ORCs) is triggered by SCdk. (c) The phosphorylation and degradation of Cdc6 help to ensure that DNA is replicated only once in each cell cycle. (d) DNA synthesis can only begin after prereplicative complexes assemble on the ORCs.

(b) The loading of the origin recognition complexes (ORCs) is triggered by SCdk

Which of the following events does not usually occur during interphase? (a) Cells grow in size. (b) The nuclear envelope breaks down. (c) DNA is replicated. (d) The centrosomes are duplicated.

(b) The nuclear envelope breaks down

Which of the following statements is true? (a) Lysosomes are believed to have originated from the engulfment of bacteria specialized for digestion. (b) The nuclear membrane is thought to have arisen from the plasma membrane invaginating around the DNA. (c) Because bacteria do not have mitochondria, they cannot produce ATP in a membrane-dependent fashion. (d) Chloroplasts and mitochondria share their DNA.

(b) The nuclear membrane is thought to have arisen from the plasma membrane invaginating around the DNA

Review how a technology such as RNA interference or CRISPR can be used to assess the importance of a particular protein in a signaling pathway.

Conversely, the activity of a specific signaling protein can be inhibited or eliminated. In the case of Ras, for example, one could shut down the expression of the Ras gene in cells by RNA interference or CRISPR. Such cells do not proliferate in response to extracellular mitogens, indicating the importance of normal Ras signaling in the proliferative response.

What is the function of rRNA?

Crucial components of ribosomes

Compare how cyclins and Cdks vary in concentration and activity during the cell cycle.

Cyclins are so-named because, unlike Cdks, their concentrations vary in a clinical fashion during the cell cycle. For example, M cyclin increases during interphase and dramatically drops during mitosis. M-Cdk is very low during interphase and rises at the start of mitosis and dramatically drops at the ends of mitosis.

Contrast the types of DNA sequences that can accommodate mutations with those that cannot.

DNA that doesn't code for proteins or RNA is only randomly mutated. Deletrious mutations in important genes are not accommodated so easily. when mutations occur, the faulty organism will almost always be eliminated or fail to reproduce.

Indicate whether each of the following conditions would increase or decrease the effect of acetylcholine. A. addition of a drug that stimulates the GTPase activity of the Gα subunit B. mutations in the K+ channel that keep it closed all the time C. modification of the Gα subunit by cholera toxin D. a mutation that decreases the affinity of the βγ complex of the G protein for the K+ channel E. a mutation in the acetylcholine receptor that prevents its localization on the cell surface F. adding acetylcholinesterase to the external environment of the cell

Decrease, decrease, increase, decrease, decrease, decrease

Which of the following statements is true? (a) Because endocrine signals are broadcast throughout the body, all cells will respond to the hormonal signal. (b) The regulation of inflammatory responses at the site of an infection is an example of paracrine signaling. (c) Paracrine signaling involves the secretion of signals into the bloodstream for distribution throughout the organism. (d) The axons of neurons typically signal target cells using membrane-bound signaling molecules that act on receptors in the target cells.

(b) The regulation of inflammatory responses at the site of an infection is an example of paracrine signaling.

Describe how cells behave when deprived of mitogens.

Deprived of mitogens, the cell cycle arrests in G1. If the cell is deprived of mitogens long enough, it will withdraw from the cell cycle and enter a nonproliferating state, in which the cell can remain.

Why does the activation of M-Cdk begin abruptly?

Each M-Cdk complex can activate more M-Cdk

Which statement about cell signaling is correct?

Each receptor is generally activated by only one type of signal molecule.

Distinguish the main types of signal-mediated cell-cell communication and identify the type of extracellular signal molecule involved in each.

Endocrine - hormone signaling molecules; long distance signaling; diffusion through the blood stream Paracrine - local mediators; diffuse locally through extracellular fluid in the same cell neighborhood (some cells respond to local mediators they produce themselves, autocrine signaling) Neuronal - neurotransmitters; can deliver long distances, delivered quickly and specifically through axon through electrical signals; axon terminal releases signal at synapse (tiny gap between two cells); the post synaptic cell is one with receptor that receives signal Contact Dependent - signals are bound to cell surface and bind to receptor on other cell surface; important for cell differentiation

Define endocrine, paracrine, neuronal and contact-dependent cell signaling.

Endocrine signaling- the most "public" style of cell-cell communication involves broadcasting the signal throughout the whole body by secreting it into an animal's bloodstream or a plant's sap. Extracellular signal molecules used in this way are called hormones. Paracrine signaling-the signal molecules diffuse locally through the extracellular fluid, remaining in the neighborhood of the cell that secretes them In neuronal signaling a message is delivered quickly and specifically to individual target cells through private lines.Contact-dependent signaling allows adjacent cells that are initially similar to become specialized to form different cell types

Describe how the ethylene signaling pathway regulates ripening of fruits.

Ethylene is a gaseous hormone that regulates a diverse array of developmental processes, including seed germination and fruit ripping. In the absence of ethylene, the empty receptor activates an associated protein kinase that ultimately shuts off the ethylene-responsive genes in the nucleus; when ethylene is present, the receptor and kinase are inactive, and the ethylene-responsive genes are transcribed.

The growth factor RGF stimulates proliferation of cultured rat cells. The receptor that binds RGF is a receptor tyrosine kinase called RGFR. Which of the following types of alteration would be most likely to prevent receptor dimerization? (a) a mutation that increases the affinity of RGFR for RGF (b) a mutation that prevents RGFR from binding to RGF (c) changing the tyrosines that are normally phosphorylated on RGFR dimerization to alanines (d) changing the tyrosines that are normally phosphorylated on RGFR dimerization to glutamic acid

(b) a mutation that prevents RGFR from binding to RGF

Male cockroaches with mutations that strongly decrease the function of an RTK called RTKX are oblivious to the charms of their female comrades. This particular RTK binds to a small molecule secreted by sexually mature females. Most males carrying loss-of-function mutations in the gene for Ras protein are also unable to respond to females. You have just read a paper in which the authors describe how they have screened cockroaches that are mutant in RTKX for additional mutations that partly restore the ability of males to respond to females. These mutations decrease the function of a protein that the authors call Z. Which of the following types of protein could Z be? Explain your answer. (a) a protein that activates the Ras protein by causing Ras to exchange GDP for GTP (b) a protein that stimulates hydrolysis of GTP by the Ras protein (c) an adaptor protein that mediates the binding of the RTKX to the Ras protein (d) a transcriptional regulator required for the expression of the Ras gene

(b) a protein that stimulates hydrolysis of GTP by the Ras protein

Which of the following conditions is likely to decrease the likelihood of skeletal muscle contraction? (a) partial depolarization of the T-tubule membrane, such that the resting potential is closer to zero (b) addition of a drug that blocks Ca2+ binding to troponin (c) an increase in the amount of ATP in the cell (d) a mutation in tropomyosin that decreases its affinity for the actin filament

(b) addition of a drug that blocks Ca2+ binding to troponin

The tryptophan operator ___________________________. (a) is an allosteric protein. (b) binds to the tryptophan repressor when the repressor is bound to tryptophan. (c) is required for production of the mRNA encoded by the tryptophan operon. (d) is important for the production of the tryptophan repressor.

(b) binds to the tryptophan repressor when the repressor is bound to tryptophan

How are most eukaryotic transcription regulators able to affect transcription when their binding sites are far from the promoter? (a) by binding to their binding site and sliding to the site of RNA polymerase assembly (b) by looping out the intervening DNA between their binding site and the promoter (c) by unwinding the DNA between their binding site and the promoter (d) by attracting RNA polymerase and modifying it before it can bind to the promoter

(b) by looping out the intervening DNA between their binding site and the promoter

After the first meiotic cell division ________. (a) two haploid gametes are produced. (b) cells are produced that contain the same number of chromosomes as somatic cells. (c) the number of chromosomes will vary depending on how the paternal and maternal chromosomes align at the metaphase plate. (d) DNA replication occurs.

(b) cells are produced that contain the same number of chromosomes as somatic cells

During recombination _________. (a) sister chromatids undergo crossing-over with each other. (b) chiasmata hold chromosomes together. (c) one crossover event occurs for each pair of human chromosomes. (d) the synaptonemal complex keeps the sister chromatids together until anaphase II.

(b) chiasmata hold chromosomes together

The activation of the serine/threonine protein kinase Akt requires phosphoinositide 3-kinase (PI 3-kinase) to _________. (a) activate the RTK. (b) create phosphorylated lipids that serve as docking sites that localize Akt to the plasma membrane. (c) directly phosphorylate Akt. (d) to create DAG.

(b) create phosphorylated lipids that serve as docking sites that localize Akt to the plasma membrane

A cell with nuclear lamins that cannot be phosphorylated in M phase will be unable to ________. (a) reassemble its nuclear envelope at telophase. (b) disassemble its nuclear lamina at prometaphase. (c) begin to assemble a mitotic spindle. (d) condense its chromosomes at prophase.

(b) disassemble its nuclear lamina at prometaphase

Which of the following statements about iPS cells is false? (a) iPS cells are created by adding a combination of transcription regulators to a fibroblast. (b) iPS cells created from mouse cells can differentiate into almost any human cell type. (c) Stimulation by extracellular signal molecules causes iPS cells to differentiate. (d) During the de-differentiation process to become an iPS, the fibroblast will undergo changes to its gene expression profile.

(b) iPS cells created from mouse cells can differentiate into almost any human cell type

You have isolated a strain of mutant yeast cells that divides normally at 30°C but cannot enter M phase at 37°C. You have isolated its mitotic cyclin and mitotic Cdk and find that both proteins are produced and can form a normal M-Cdk complex at both temperatures. Which of the following temperature-sensitive mutations could not be responsible for the behavior of this strain of yeast? (a) inactivation of a protein kinase that acts on the mitotic Cdk kinase (b) inactivation of an enzyme that ubiquitylates M cyclin (c) inactivation of a phosphatase that acts on the mitotic Cdk kinase (d) a decrease in the levels of a transcriptional regulator required for producing sufficient amounts of M cyclin

(b) inactivation of an enzyme that ubiquitylates M cyclin

Combinatorial control of gene expression ________________. (a) involves every gene using a different combination of transcriptional regulators for its proper expression. (b) involves groups of transcriptional regulators working together to determine the expression of a gene. (c) involves only the use of gene activators used together to regulate genes appropriately. (d) is seen only when genes are arranged in operons.

(b) involves groups of transcriptional regulators working together to determine the expression of a gene

What do you predict would happen if you created a tRNA with an anticodon of 5'-CAA-3' that is charged with methionine, and added this modified tRNA to a cell-free translation system that has all the normal components required for translating RNAs? (use codon table) (a) methionine would be incorporated into proteins at some positions where glutamine should be (b) methionine would be incorporated into proteins at some positions where leucine should be (c) methionine would be incorporated into proteins at some positions where valine should be (d) translation would no longer be able to initiate

(b) methionine would be incorporated into proteins at some positions where leucine should be

Which of the following DNA sequences is not commonly carried on a DNA-only transposon? (a) transposase gene (b) reverse transcriptase gene (c) recognition site for transposase (d) antibiotic-resistance gene

(b) reverse transcriptase gene

In eukaryotes, but not in prokaryotes, ribosomes find the start site of translation by ____________. (a) binding directly to a ribosome-binding site preceding the initiation codon. (b) scanning along the mRNA from the 5' end. (c) recognizing an AUG codon as the start of translation. (d) binding an initiator tRNA.

(b) scanning along the mRNA from the 5' end

A mutation in the tRNA for the amino acid lysine results in the anticodon sequence 5'-UAU-3' (instead of 5'-UUU-3'). Which of the following aberrations in protein synthesis might this tRNA cause? (use codon table) (a) read-through of stop codons (b) substitution of lysine for isoleucine (c) substitution of lysine for tyrosine (d) substitution of lysine for phenylalanine

(b) substitution of lysine for isoleucine

Progression through the cell cycle requires a cyclin to bind to a Cdk because _________. (a) the cyclins are the molecules with the enzymatic activity in the complex. (b) the binding of a cyclin to Cdk is required for Cdk enzymatic activity. (c) cyclin binding inhibits Cdk activity until the appropriate time in the cell cycle. (d) without cyclin binding, a cell-cycle checkpoint will be activated.

(b) the binding of a cyclin to Cdk is required for Cdk enzymatic activity

The owners of a local bakery ask for your help in improving a special yeast strain they use to make bread. They would like you to help them design experiments using RNA interference to turn off genes, to allow them to test their hypothesis that certain genes are important for the good flavors found in their bread. Of the components in the following list, which is the most important to check for in this yeast strain if you'd like this project to succeed? (a) the presence of foreign double-stranded RNA (b) the presence of genes in the genome that code for RISC proteins (c) the presence of miRNA genes in the genome (d) the presence of single-stranded siRNAs within the cell

(b) the presence of genes in the genome that code for RISC proteins

An individual transport vesicle ________. (a) contains only one type of protein in its lumen. (b) will fuse with only one type of membrane. (c) is endocytic if it is traveling toward the plasma membrane. (d) is enclosed by a membrane with the same lipid and protein composition as the membrane of the donor organelle.

(b) will fuse with only one type of membrane

In the absence of recombination, how many genetically different types of gamete can an organism with five homologous chromosome pairs produce? (a) 5 (b) 10 (c) 32 (d) 64

(c) 32

The growth factor Superchick stimulates the proliferation of cultured chicken cells. The receptor that binds Superchick is a receptor tyrosine kinase (RTK), and many chicken tumor cell lines have mutations in the gene that encodes this receptor. Which of the following types of mutation would be expected to promote uncontrolled cell proliferation? (a) a mutation that prevents dimerization of the receptor (b) a mutation that destroys the kinase activity of the receptor (c) a mutation that inactivates the protein tyrosine phosphatase that normally removes the phosphates from tyrosines on the activated receptor (d) a mutation that prevents the binding of the normal extracellular signal to the receptor

(c) a mutation that inactivates the protein tyrosine phosphatase that normally removes the phosphates from tyrosines on the activated receptor

Which of the following statements is correct? Kinesins and dyneins ________. (a) have tails that bind to the filaments. (b) move along both microtubules and actin filaments. (c) often move in opposite directions to each other. (d) derive their energy from GTP hydrolysis.

(c) often move in opposite directions to each other

Which of the following statements about the cytoskeleton is true? (a) All eukaryotic cells have actin, microtubules, and intermediate filaments in their cytoplasm. (b) The cytoskeleton provides a rigid and unchangeable structure important for the shape of the cell. (c) The three cytoskeletal filaments perform distinct tasks in the cell and act completely independently of one another. (d) Actin filaments and microtubules have an inherent polarity, with a plus end that grows more quickly than the minus end.

(d) Actin filaments and microtubules have an inherent polarity, with a plus end that grows more quickly than the minus end

Which of the following statements about apoptosis is true? (a) Cells that constitutively express Bcl2 will be more prone to undergo apoptosis. (b) The prodomain of procaspases contains the catalytic activity necessary for procaspase activation. (c) Bax and Bak promote apoptosis by binding to procaspases in the apoptosome. (d) Apoptosis is promoted by the release of cytochrome c into the cytosol from mitochondria.

(d) Apoptosis is promoted by the release of cytochrome c into the cytosol from mitochondria

Which of the following statements about disulfide bond formation is false? (a) Disulfide bonds do not form under reducing environments. (b) Disulfide bonding occurs by the oxidation of pairs of cysteine side chains on the protein. (c) Disulfide bonding stabilizes the structure of proteins. (d) Disulfide bonds form spontaneously within the ER because the lumen of the ER is oxidizing.

(d) Disulfide bonds form spontaneously within the ER because the lumen of the ER is oxidizing

Which of the following statements about mobile genetic elements is true? (a) Mobile genetic elements can sometimes rearrange the DNA sequences of the genome in which they are embedded by accidentally excising neighboring chromosomal regions and reinserting these sequences into different places within the genome. (b) DNA-only transposons do not code for proteins but instead rely on transposases found in cells that are infected by viruses. (c) The two major families of transposable sequences found in the human genome are DNA-only transposons that move by replicative transposition. (d) During cut-and-paste transposition, the donor DNA will no longer have the mobile genetic element embedded in its sequence when transposition is complete.

(d) During cut-and-paste transposition, the donor DNA will no longer have the mobile genetic element embedded in its sequence when transposition is complete

Which of the following choices reflects the appropriate order of locations through which a protein destined for the plasma membrane travels? (a) lysosome -> endosome -> plasma membrane (b) ER -> lysosome -> plasma membrane (c) Golgi -> lysosome -> plasma membrane (d) ER -> Golgi -> plasma membrane

(d) ER -> Golgi -> plasma membrane

Which of the following statements about DNA methylation in eukaryotes is false? (a) Appropriate inheritance of DNA methylation patterns involves maintenance methyltransferase. (b) DNA methylation involves a covalent modification of cytosine bases. (c) Methylation of DNA attracts proteins that block gene expression. (d) Immediately after DNA replication, each daughter helix contains one methylated DNA strand, which corresponds to the newly synthesized strand.

(d) Immediately after DNA replication, each daughter helix contains one methylated DNA strand, which corresponds to the newly synthesized strand

Which of the following statements is false? (a) Cytokinesis in plant cells is mediated by the microtubule cytoskeleton. (b) Small membrane vesicles derived from the Golgi apparatus deliver new cell-wall material for the new wall of the dividing cell. (c) The phragmoplast forms from the remains of interpolar microtubules of the mitotic spindle. (d) Motor proteins walking along the cytoskeleton are important for the contractile ring that guides formation of the new cell wall.

(d) Motor proteins walking along the cytoskeleton are important for the contractile ring that guides formation of the new cell wall

Different glycoproteins can have a diverse array of oligosaccharides. Which of the statements below about this diversity is true? (a) Extensive modification of oligosaccharides occurs in the extracellular space. (b) Different oligosaccharides are covalently linked to proteins in the ER and the Golgi. (c) A diversity of oligosaccharyl transferases recognizes specific protein sequences, resulting in the linkage of a variety of oligosaccharides to proteins. (d) Oligosaccharide diversity comes from modifications that occur in the ER and the Golgi of the 14-sugar oligosaccharide added to the protein in the ER.

(d) Oligosaccharide diversity comes from modifications that occur in the ER and the Golgi of the 14-sugar oligosaccharide added to the protein in the ER

You are interested in Fuzzy, a soluble protein that functions within the ER lumen. Given that information, which of the following statements must be true? (a) Fuzzy has a C-terminal signal sequence that binds to SRP. (b) Only one ribosome can be bound to the mRNA encoding Fuzzy during translation. (c) Fuzzy must contain a hydrophobic stop-transfer sequence. (d) Once the signal sequence from Fuzzy has been cleaved, the signal peptide will be ejected into the ER membrane and degraded.

(d) Once the signal sequence from Fuzzy has been cleaved, the signal peptide will be ejected into the ER membrane and degraded

Which of the following statements about the human genome is false? (a) About 50% of the human genome is made up of mobile genetic elements. (b) More of the human genome comprises intron sequences than exon sequences. (c) About 1.5% of the human genome codes for exons. (d) Only the exons are conserved between the genomes of humans and other mammals.

(d) Only the exons are conserved between the genomes of humans and other mammals

Unlike DNA, which typically forms a helical structure, different molecules of RNA can fold into a variety of three-dimensional shapes. This is largely because _____________. (a) RNA contains uracil and uses ribose as the sugar. (b) RNA bases cannot form hydrogen bonds with each other. (c) RNA nucleotides use a different chemical linkage between nucleotides compared to DNA. (d) RNA is single-stranded.

(d) RNA is single-stranded.

Which of the following functions do you not expect to find in the set of genes found in all organisms on Earth? (a) DNA replication (b) DNA repair (c) protein production (d) RNA splicing

(d) RNA splicing

What would be the most obvious outcome of repeated cell cycles consisting of S phase and M phase only? (a) Cells would not be able to replicate their DNA. (b) The mitotic spindle could not assemble. (c) Cells would get larger and larger. (d) The cells produced would get smaller and smaller.

(d) The cells produced would get smaller and smaller

Which of the following statements is false? (a) The cleavage furrow is a puckering of the plasma membrane caused by the constriction of a ring of filaments attached to the plasma membrane. (b) The cleavage furrow will not begin to form in the absence of a mitotic spindle. (c) The cleavage furrow always forms perpendicular to the interpolar microtubules. (d) The cleavage furrow always forms in the middle of the cell.

(d) The cleavage furrow always forms in the middle of the cell

Which of the following statements about G-protein-coupled receptors (GPCRs) is false? (a) GPCRs are the largest family of cell-surface receptors in humans. (b) GPCRs are used in endocrine, paracrine, and neuronal signaling. (c) GPCRs are found in yeast, mice, and humans. (d) The different classes of GPCR ligands (proteins, amino acid derivatives, or fatty acids) bind to receptors with different numbers of transmembrane domains.

(d) The different classes of GPCR ligands (proteins, amino acid derivatives, or fatty acids) bind to receptors with different numbers of transmembrane domains

Imagine meiosis in a diploid organism that only has a single chromosome. Like most diploid organisms, it received one copy of this chromosome from each of its parents and the two homologs are genetically distinct. If only a single homologous recombination event occurs during meiosis, which of the following choices below correctly describes the four gametes formed. (a) None of the gametes will contain chromosomes identical to the chromosomes found in the original diploid cell. (b) All four of the gametes will have chromosomes identical to the chromosomes found in the original diploid cell. (c) Three of the gametes will have chromosomes identical to the chromosomes found in the original diploid cell, while one of the gametes will have chromosomes that are different. (d) Two of the gametes will have chromosomes identical to the chromosomes found in the original diploid cell, while two of the gametes will have chromosomes that are different.

(d) Two of the gametes will have chromosomes identical to the chromosomes found in the original diploid cell, while two of the gametes will have chromosomes that are different

Which of the following is not good direct evidence that the cell-cycle control system is conserved through billions of years of divergent evolution? (a) A yeast cell lacking a Cdk function can use the human Cdk to substitute for its missing Cdk during the cell cycle. (b) The amino acid sequences of cyclins in plants are similar to the amino acid sequences of cyclins in humans. (c) The Cdk proteins in humans share conserved phosphorylation sites with the Cdk proteins in yeast. (d) Yeast cells have only one Cdk, whereas humans have many Cdks.

(d) Yeast cells have only one Cdk, whereas humans have many Cdks

Which of the following might decrease the transcription of only one specific gene in a bacterial cell? (a) a decrease in the amount of sigma factor (b) a decrease in the amount of RNA polymerase (c) a mutation that introduced a stop codon into the DNA that precedes the gene's coding sequence (d) a mutation that introduced extensive sequence changes into the DNA that precedes the gene's transcription start site

(d) a mutation that introduced extensive sequence changes into the DNA that precedes the gene's transcription start site

Which of the following protein families are not involved in directing transport vesicles to the target membrane? (a) SNAREs (b) Rabs (c) tethering proteins (d) adaptins

(d) adaptins

You are interested in understanding the regulation of nuclear lamina assembly. To create an in vitro system for studying this process you start with partly purified nuclear lamina subunits to which you will add back purified cellular components to drive nuclear lamina assembly. Before you start doing experiments, your instructor suggests that you consider what type of conditions would be most amenable to the assembly of the nuclear lamina from its individual subunits in vitro. Which of the following conditions do you predict would be most likely to enhance the assembly of the nuclear lamina? (a) addition of phosphatase inhibitors (b) addition of ATP (c) addition of a concentrated salt solution that is 10 times the concentration normally found in the nucleoplasm (d) addition of protein kinase inhibitors

(d) addition of protein kinase inhibitors

After isolating the rough endoplasmic reticulum from the rest of the cytoplasm, you purify the RNAs attached to it. Which of the following proteins do you expect the RNA from the rough endoplasmic reticulum to encode? (a) soluble secreted proteins (b) ER membrane proteins (c) plasma membrane proteins (d) all of the above

(d) all of the above

For both actin and microtubule polymerization, nucleotide hydrolysis is important for ______. (a) stabilizing the filaments once they are formed. (b) increasing the rate at which subunits are added to the filaments. (c) promoting nucleation of filaments. (d) decreasing the binding strength between subunits on filaments.

(d) decreasing the binding strength between subunits on filaments

During sexual reproduction, novel mixtures of alleles are generated. This is because ______. (a) in all diploid species, two alleles exist for every gene. (b) a diploid individual has two different alleles for every gene. (c) every gamete produced by a diploid individual has several different alleles of a single gene. (d) during meiosis, the segregation of homologs is random such that different gametes end up with different alleles of each gene.

(d) during meiosis, the segregation of homologs is random such that different gametes end up with different alleles of each gene

Which of the following is not a good example of a housekeeping protein? (a) DNA repair enzymes (b) histones (c) ATP synthase (d) hemoglobin

(d) hemoglobin

You isolate a pathogenic strain of E. coli from a patient and discover that this E.coli strain is resistant to an antibiotic. Common laboratory strains of E. coli are not resistant to this antibiotic, nor are any other previously isolated pathogenic E.coli strains. However, such resistance has been observed in other bacteria in the hospital in which the patient was treated. This newly discovered antibiotic resistance in E. coli is most likely due to _______. (a) a mutation within a gene. (b) a mutation within the regulatory DNA of a gene. (c) gene duplication. (d) horizontal gene transfer.

(d) horizontal gene transfer

Which of the following would contribute most to successful exon shuffling? (a) shorter introns (b) a haploid genome (c) exons that code for more than one protein domain (d) introns that contain regions of similarity to one another

(d) introns that contain regions of similarity to one another

The G1 DNA damage checkpoint ________. (a) causes cells to proceed through S phase more quickly. (b) involves the degradation of p53. (c) is activated by errors caused during DNA replication. (d) involves the inhibition of cyclin-Cdk complexes by p21.

(d) involves the inhibition of cyclin-Cdk complexes by p21

Total nucleic acids are extracted from a culture of yeast cells and are then mixed with resin beads to which the polynucleotide 5'-TTTTTTTTTTTTTTTTTTTTTTTTT-3' has been covalently attached. After a short incubation, the beads are then extracted from the mixture. When you analyze the cellular nucleic acids that have stuck to the beads, which of the following is most abundant? (a) DNA (b) tRNA (c) rRNA (d) mRNA

(d) mRNA

According to current thinking, the minimum requirement for life to have originated on Earth was the formation of a _______________. (a) molecule that could provide a template for the production of a complementary molecule. (b) double-stranded DNA helix. (c) molecule that could direct protein synthesis. (d) molecule that could catalyze its own replication.

(d) molecule that could catalyze its own replication

Disassembly of the nuclear envelope ________. (a) causes the inner nuclear membrane to separate from the outer nuclear membrane. (b) results in the conversion of the nuclear envelope into protein-free membrane vesicles. (c) is triggered by the phosphorylation of integrins. (d) must occur for kinetochore microtubules to form in animal cells.

(d) must occur for kinetochore microtubules to form in animal cells

Your friend works in a biotech company that has just discovered a drug that seems to promote lamellipodia formation in cells. Which of the following molecules is unlikely to be involved in the pathway that this drug affects? (a) Rac (b) ARP (c) actin (d) myosin

(d) myosin

Transcription is similar to DNA replication in that ___________________. (a) an RNA transcript is synthesized discontinuously and the pieces are then joined together. (b) it uses the same enzyme as that used to synthesize RNA primers during DNA replication. (c) the newly synthesized RNA remains paired to the template DNA. (d) nucleotide polymerization occurs only in the 5'-to-3' direction.

(d) nucleotide polymerization occurs only in the 5'-to-3' direction.

The length of time a G protein will signal is determined by _______. (a) the activity of phosphatases that turn off G proteins by dephosphorylating Gα. (b) the activity of phosphatases that turn GTP into GDP. (c) the degradation of the G protein after Gα separates from Gβγ (d) the GTPase activity of Gα

(d) the GTPase activity of Gα

N-linked oligosaccharides on secreted glycoproteins are attached to ________. (a) nitrogen atoms in the polypeptide backbone. (b) the serine or threonine in the sequence Asn-X-Ser/Thr. (c) the N-terminus of the protein. (d) the asparagine in the sequence Asn-X-Ser/Thr.

(d) the asparagine in the sequence Asn-X-Ser/Thr

You have a segment of DNA that contains the following sequence: 5′-GGACTAGACAATAGGGACCTAGAGATTCCGAAA-3′ 3′-CCTGATCTGTTATCCCTGGATCTCTAAGGCTTT-5′ You know that the RNA transcribed from this segment contains the following sequence: 5′-GGACUAGACAAUAGGGACCUAGAGAUUCCGAAA-3′ Which of the following choices best describes how transcription occurs? (a) the top strand is the template strand; RNA polymerase moves along this strand from 5' to 3' (b) the top strand is the template strand; RNA polymerase moves along this strand from 3' to 5' (c) the bottom strand is the template strand; RNA polymerase moves along this strand from 5' to 3' (d) the bottom strand is the template strand; RNA polymerase moves along this strand from 3' to 5'

(d) the bottom strand is the template strand; RNA polymerase moves along this strand from 3' to 5'

Using genetic engineering techniques, you remove the sequences that code for the ribosome-binding sequences of the bacterial LacZ gene. The removal of these sequences will lead to ___________. (a) more LacZ protein produced due to faster ribosome movement across the LacZ mRNA. (b) transcriptional repression, resulting in fewer mRNA molecules produced from this gene. (c) a longer half-life for the LacZ mRNA. (d) translational inhibition of the LacZ mRNA.

(d) translational inhibition of the LacZ mRNA

Organisms that reproduce sexually ________. (a) must be haploid, unlike organisms that reproduce asexually. (b) can reproduce only with a partner that carries the same alleles. (c) create zygotes that are genetically identical to each other. (d) undergo a sexual reproductive cycle that involves an alternation of haploid cells with the generation of diploid cells.

(d) undergo a sexual reproductive cycle that involves an alternation of haploid cells with the generation of diploid cells

A single nondisjunction event during meiosis _________. (a) will block recombination. (b) will occur only during meiosis II. (c) cannot occur with sex chromosomes. (d) will involve the production of two normal gametes if it occurs during meiosis II.

(d) will involve the production of two normal gametes if it occurs during meiosis II

Which of the following statements about the structure of microtubules is false? (a) Microtubules are built from protofilaments that come together to make a hollow structure. (b) The two ends of a protofilament are chemically distinct, with α-tubulin exposed at one end and β-tubulin exposed at the other end. (c) Within a microtubule, all protofilaments are arranged in the same orientation, giving the microtubule structural polarity. (d) α-Tubulin and β-tubulin are covalently bound to make the tubulin dimer that then assembles into protofilaments.

(d) α-Tubulin and β-tubulin are covalently bound to make the tubulin dimer that then assembles into protofilaments

Illustrate how selectively neutral mutations, combined with analysis of the fossil record, can be used to construct a phylogenetic tree.

For species that are closely related, it is most informative to focus on selectively neutral mutations because they accumulate steadily at a rate that is unconstrained by selection pressures, these mutations provide a metric for gauging how much modern species have diverged from their common ancestor. Such sequence comparisons allow the construction of a phylogenetic tree.

Summarize how incomplete or incorrect replication can arrest the cell cycle in G2.

For the cell to progress into mitosis, inhibitory phosphates must be removed by an activating protein called Cdc25. If DNA replication stalls, the appearance of single stranded DNA at the replication fork triggers a DNA damage response. Part of this response includes the inhibition of Cdc25, which prevents the removal of phosphates from M-cdk.

Distinguish the two main types of GTP-binding proteins.

G Proteins (large trimeric GTP binding proteins) - relay messages from G protein coupled receptors (GPCRs) GTPases (small, monomeric GTP binding proteins) - help relay signals Switch proteins aided by two sets of regulatory proteins to bind and hydrolyze GTP GEF - guanine nucleotide exchange factors, activate switch, GDP->GTP GAPs - GTPase activating proteins, inactivate, GTP hydrolysis, GTP->GDP

Outline how cyclic AMP is produced in response to G protein activation, and recall how caffeine can potentiate this response.

G protein is activated, switches on adenylyl cyclase, sudden increase in cAMP from ATP To terminate signal, enzyme cyclic AMP phosphodiesterase rapidly converts cAMP to AMP Caffeine inhibits this phosphodiesterase in nervous system, blocking cyclic AMP degradation & keeping concentration of it high

G-protein-coupled receptors (GPCRs) all have a similar structure with _______ transmembrane domains. When a GPCR binds an extracellular signal, an intracellular G protein, composed of ______ subunits, becomes activated. _____ of the G-protein subunits are tethered to the plasma membrane by short lipid tails. When unstimulated, the α subunit is bound to _______, which is exchanged for _______ on stimulation. The intrinsic _______ activity of the α subunit is important for inactivating the G protein. ______ inhibits this activity of the α subunit, thereby keeping the subunit in an active state. adenylyl cyclase, cholera toxin, GTPase, AMP, diacylglycerol, phosphodiesterase, ATP, five, seven, ATPase, four, three, Ca2+, GDP, twelve, cAMP, GTP, two,

G-protein-coupled receptors (GPCRs) all have a similar structure with seven transmembrane domains. When a GPCR binds an extracellular signal, an intracellular G protein, composed of three subunits, becomes activated. Two of the G-protein subunits are tethered to the plasma membrane by short lipid tails. When unstimulated, the α subunit is bound to GDP, which is exchanged for GTP on stimulation. The intrinsic GTPase activity of the α subunit is important for inactivating the G protein. Cholera toxin inhibits this activity of the α subunit, thereby keeping the subunit in an active state.

Review the structure of G-protein-coupled receptors (GPCRs) and describe the types of extracellular signal molecules that bind to them.

GPCRs are the largest family of cell surface receptors. They mediate responses to a variety of signal molecules: hormones, local mediators, neurotransmitters. Signals that bind can be proteins, small peptides, amino acid derivatives, fatty acids, etc. Each one has a different receptor or set of receptors. Still, all GPCRs have similar structure. · Single polypeptide chain that threads back ad forth through plasma membrane *7* times (alpha helices) · Cytoplasmic portion binds to G protein inside cell · For receptors that recognize small signals (acetylcholine & epinephrine), the ligand binds deep within membrane in pocket formed by AAs from transmembrane segments · Receptors that recognize proteins have large extracellular domain that bind protein to ligand One third of drugs used today work through GPCRs.

Articulate how gene duplication can lead to the evolution of genes with new functions or to the generation of pseudogenes.

Gene duplication can lead to the evolution of genes with new functions due to a cell needing a more efficient way to do things.?? There are several duplicated DNA sequences in the a- and B- globin gene clusters that are not function genes. They are similar in DNA sequence to the functional globulin games, but they have been disabled by the accumulation of many inactivating mutations. The existence of pseudogenes makes it clear that not every DNA duplication leads to a new functional gene. In fact, most gene duplication events are unsuccessful in that one copy is gradually inactivated by mutation.

The conversion of the genetic instructions in replication, transcription, and translation is termed:

Gene expression

Explain how studies of mutant yeast were used to dissect the cell-cycle control system.

Genes in yeast turned out to encode cyclin or Cdk proteins, which were unmistakable similar-in both amino acid sequence and function- to their counterparts in frogs and clams. Similar genes were soon identified in human cells. A yeast with a defective copy of the gene encoding its only Cdk fails to divide, but it divides normally if a copy of the appropriate human gene is artificially introduced into the defective cell.

Compare the fate of germ-line mutations that are deleterious, selectively neutral, or that provide a selective advantage.

Genes with selective advantage are more likely to be preserved. Deletrious are usually lost, selectively neutral may or may not persist.

Relate the speeds of the responses produced by G proteins activating an ion channel versus activating a membrane-bound enzyme.

Gi protein, βγ binds to intracellular face of K+ channel of pacemaker cell, forcing ion channel to open and allowing K+ to flow out. This increases the membrane's permeability to K+, making it less electrically active, slowing the heart rate. G proteins that interact with ion channels cause immediate change. With enzymes, the reaction is less rapid and more complex because they must produce additional intracellular signaling molecules.

Phragmoplast is built by vesicles largely originating from which organelle?

Golgi apparatus

What is the general name given to the most highly condensed form of chromatin?

Heterochromatin

Heterochromatin:

Highly condensed region of an interphase chromosome; generally gene-poor and transcriptionally inactive.

Define homologous genes and state the percentage of human genes that have clear homologs in species such as the fruit fly and nematode.

Homologus genes are those that are similar in nucleotide sequence due to a common ancestry. 50%of human genes have homologs in species like the fruit fly and nematode.

Illustrate how homologous recombination can lead to gene duplication.

Homologus recombination is the mechanism in which an intact chromosome is used as a template to repair a damaged sequence, but it can also catalyze crossovers that produce hybrid chromosomes.

Compare the number of protein-coding genes present in humans versus Drosophila, C. elegans, and Arabidopsis, and analyze how these values relate to the complexity of the organism.

Humans have 19,000 protein-coding genes; Drosophila has 14,000; C. elegans has 22,000; Arabidopis has 28,000. Organism complexity relates more to the innovations in regulatory DNA sequences than the proteins or functional RNAs.

State what occurs when the DNA damage detected in G1 is too extensive to be repaired.

If the damage is too severe, p53 can induce the cell to kill itself through apoptosis.

Express what conserved synteny between two extant—or living—species indicates about the genome of their common ancestor.

If there is conserved synteny between two species, the genes were neighbors in ancestral DNA and remain neighbors today.

Outline the steps involved in the integration and replication of a retrovirus.

In a double-stranded RNA form, retroviruses infect a host cell with their genome, and then are reverse transcribed into double-stranded DNA, with the DNA integrated into the host cell genome. When integrated into a host genome, a retrovirus is hard to detect and can lay dormant for prolonged periods, having no discernible effect on the host.

Compare and contrast translation in bacteria and eukaryotes.

In a prokaryotic cell, transcription and translation are coupled; that is, translation begins while the mRNA is still being synthesized. In a eukaryotic cell, transcription occurs in the nucleus, and translation occurs in the cytoplasm.

Compare the changes in the mitotic spindle that underlie chromosome segregation during anaphase A and anaphase B, and delineate the driving forces responsible for each process.

In anaphase A, the sister chromatids are pulled toward the opposite poles as the kinetochore microtubules depolymerize. The force driving this movement is generated mainly at the kinetochore. In anaphase B, the two spindle poles move apart as the result of two separate forces: (1) the elongation and sliding of the interpolar microtubules past one another pushes the two poles apart, and (2) forces exerted on the outward-pointing astral microtubules at each spindle pole pull the poles away from each other, toward the cell cortex.

Compare and contrast transcription in bacteria and eukaryotes.

In bacteria, transcription and translation can occur simultaneously in the cytoplasm of the cell, whereas in eukaryotes transcription occurs in the nucleus and translation occurs in the cytoplasm. There is only one type of bacterial RNA polymerase whereas eukaryotes have 3 types.

Summarize how adaptation in the intracellular signaling cascade of photoreceptors allows the eye to respond to dim or bright light.

In dim light, the signal is enormously amplified as it is relayed across the intracellular signaling pathway. Only a few dozen photons absorbed can be efficient to perceive a signal to be sent to the brain. Bright light, photons flood in large quantities through the photoreceptor cells. This is where the signaling cascade undergoes adaptation, minimizing the amplification. Adaptation depends on negative feedback: intense response in photoreceptor cell decreases the cytosolic Ca concentration, inhibiting enzymes responsible for signal amplification. Adaptation occurs more in intracellular signaling pathways that respond to extracellular signal molecules. This is so cells can respond to fluctuations in the concentration regardless of if the signal is present in small or large amounts, through positive and negative feedback mechanisms. Dim Light: · Absence of light, second messenger cGMP is continuously produced by guanylyl cyclase in cytosol of photoreceptor cell · cGMP binds to cation channels in the photoreceptor cell plasma membrane, keeping them OPEN Light: · Light activates rhodopsin resulting in activation of transducin α subunits · These turn on enzyme cyclic GMP phosphodiesterase, which breaks down cGMP to GMP · Sharp fall in cytosolic [cGMP] reduces amount of cGMP bound to cation channels · Cation channels close, decreasing influx of Na+ & altering voltage gradient across PM

Explain how the same signal molecule can induce different responses in different target cells.

Responses to a signal molecule depend on the receptor and the type of cell. Both heart pacemaker cells and salivary gland cells have the same receptor for acetylcholine. When exposed to acetylcholine, the heart peace maker cell decreases firing, while the salivary gland cell secretes saliva through exocytosis of vesicles. Skeletal muscle cells have a different receptor that receives two acetylcholine signal molecules, causing the cell to contract.

Differentiate between the amount of the human genome that codes for proteins versus the amount that consists of mobile genetic elements.

The first striking feature of the human genome is how little of it- less than 2% codes for proteins. In addition, almost half of our DNA is made up of mobile genetic elements that have colonized our genome over evolutionary time.

Define the cleavage furrow and explain how its position is determined.

The first visible sign of cytokinesis in animal cells is a puckering and furrowing of the plasma membrane that occurs during anaphase. The furrowing invariably occurs along a plane that runs perpendicular to the long axis of the mitotic spindle. The positioning ensures that the cleavage furrow cuts between the two groups of segregated chromosomes. If the mitotic spindle is deliberately displaced as soon as the furrow appears, the furrow will disappear and a new one will develop at a site corresponding to the new spindle location.

The four phases of the cell cycle, in order, are G1, _______, _______, and ______. A cell contains the most DNA after ______ phase of the cell cycle. A cell is smallest in size after ______ phase of the cell cycle. Growth occurs in ______, ______, and ______ phases of the cell cycle. A cell does not enter mitosis until it has completed _______ synthesis. DNA, M, protein, G1, nucleotide, S, G2, organelle

The four phases of the cell cycle, in order, are G1, S, G2, and M. A cell contains the most DNA after S phase of the cell cycle. A cell is smallest in size after M phase of the cell cycle. Growth occurs in G1, S, and G2 phases of the cell cycle. A cell does not enter mitosis until it has completed DNA synthesis

Describe how the synthesis of new DNA strands begins at an origin of replication.

The process of DNA synthesis starts by initiator proteins that bind to specific DNA sequences called replication origins. the initiator proteins pry the two DNA strands apart, breaking the hydrogen bonds between the bases. Once an initiator protein binds to DNA at a replication origin and locally opens up the double helix, it attracts a group of proteins that carry out DNA replication. These proteins form a replication machine, in which each protein carries out a specific function.

Shinya Yamanaka was awarded the Nobel Prize in Physiology or Medicine, 2012, for the discovery of induced pluripotent stem cells. Explain what these cell are and how they are produced. What cell function is manipulated?

these reprogrammed cells behave much like naturally occurring embryonic stem cells, and they can be directed to generate a variety of specialized differentiated cells. This approach, initially performed using cultured fibroblasts, has been adapted to produce iPS cells from a variety of specialized cell types, including those taken from humans. Differentiated cells produced from human iPS cells are currently being used in the study or treatment of disease. When somatic cells were reprogrammed by transferring their nuclei into oocytes or by fusion with ES cells, genome-wide transcriptional activity and DNA methylation patterns were converted from the somatic state to an embryonic state.

Which form of control directly influences which mRNAs are selected by ribosomes for the synthesis of proteins?

translational control

Proteasomes act primarily on proteins that have been marked for destruction by the attachment of which small protein?

ubiquitin

The transcription initiation site of a eukaryotic gene is found at which location?

where RNA synthesis begins

Describe the three stages of translation.

Translation takes place in a four-step cycle, which is repeated over and over during the synthesis of a protein. In step 1, a charged tRNA carrying the next amino acid to be added to the polypeptide chain binds to the vacant A site on the ribosome by forming base pairs with the mRNA codon that is exposed there. Only a matching tRNA molecule can base-pair with this codon, which determines the specific amino acid added. The A and P sites are sufficiently close together that their two tRNA molecules are forced to form base pairs with codons that are contiguous, with no stray bases in-between. This positioning of the tRNAs ensures that the correct reading frame will be preserved throughout the synthesis of the protein. In step 2, the carboxyl end of the polypeptide chain (amino acid 3 in step 1) is uncoupled from the tRNA at the P site and joined by a peptide bond to the free amino group of the amino acid linked to the tRNA at the A site. This reaction is carried out by a catalytic site in the large subunit. In step 3, a shift of the large subunit relative to the small subunit moves the two bound tRNAs into the E and P sites of the large subunit. In step 4, the small subunit moves exactly three nucleotides along the mRNA molecule, bringing it back to its original position relative to the large subunit. This movement ejects the spent tRNA and resets the ribosome with an empty A site so that the next charged tRNA molecule can bind

Which of the following statements are correct? In bacteria, but not in eukaryotes, many mRNAs contain the coding region for more than one gene. Explain your answers

True. Prokaryotic mRNAs are often transcripts of entire operons. Ribosomes can initiate translation at the internal AUG start sites of these "polycistronic" mRNAs Most DNA-binding proteins bind to the major groove of the DNA double helix. True. The major groove of double-stranded DNA is sufficiently wide to allow a protein surface, such as one face of an α helix, access to the base pairs. The sequence of H-bond donors and acceptors in the major groove can then be "read" by the protein to determine the sequence of the DNA Of the major control points in gene expression (transcription, RNA processing, RNA transport, translation, and control of a protein's activity), transcription initiation is one of the most common True. It is advantageous to exert control at the earliest possible point in a pathway. This conserves metabolic energy because unnecessary products are not made

UNIT 3 STUDY GUIDE

UNIT 3 STUDY GUIDE

Review how the binding of a signal molecule activates RTKs to trigger the assembly of an intracellular signaling complex.

Unlike GPCRs, enzyme coupled receptor proteins usually only have one transmembrane segment, which spans the lipid bilayer as a single alpha helix. Because a single alpha helix is poorly suited to transmit a conformational change across the bilayer, enzyme coupled receptors have a different strategy for transducing the extracellular signal. In many cases, the binding of an extracellular signal molecule causes two receptor molecules to come together in the plasma membrane forming a dimer. This pairing brings the two intracellular tails of the receptors together and activates their kinase domains, such that each receptor tail phosphorylates the other. In the case of RTKs, the phosphorylation's occur on specific tryosines. T-phosphorylation then triggers the assembly of transient but elaborate intracellular signaling complex on the cytosolic tails of the receptors. A new phosphorylated tyrosine serves as a docking site for a whole zoo of intracellular signaling proteins.

Describe how the nuclear envelope reassembles during telophase.

Vesicles of nuclear membrane associate with the clustered chromosomes and then fuse to re-form the nuclear envelope. During this process, the nuclear pore proteins and nuclear lamins that were phosphorylated during prometaphase are now dephosphorylated, which allows them to reassemble and rebuild the nuclear envelope and lamina.

Review how and why viruses use the host's biochemical machinery to reproduce themselves.

Viruses need a host due to their small size and inability to reproduce.

Describe how the early estimate of 100,000 human genes was derived.

Walter Gilbert estimated this number based on the average size of the few known human genes in the mid-1980s, as well as the size of the human genome.

How does PI 3-kinase activate the Akt kinase after activation of an RTK?

When PI 3-kinase is activated by an activated RTK, it phosphorylates a specific inositol phospholipid in the plasma membrane. The resulting phosphorylated inositol phospholipid then recruits to the plasma membrane both Akt and another protein kinase that helps phosphorylate and activate Akt. A third kinase that is permanently associated with the membrane also helps activate Akt

Briefly explain the Delta-Notch cell signaling system.

When the cell-surface receptor Notch interacts with a ligand (e.g., Delta), its intracellular domain is cleaved and travels to the nucleus to regulate transcription. The Notch pathway regulates cell proliferation, cell fate, differentiation, and cell death in all metazoans.

Describe how asymmetric divisions are set up during embryonic development.

When the mitotic spindle is located centrally in the cell - the usual situation in most dividing cells - the two daughter cells will be of equal size. During embryonic development, however, there are some instances in which the dividing cell moves its mitotic spindle to an asymmetrical position, and, consequently, the furrow creates two daughter cells that differ in size. In most of these asymmetric divisions, the daughters also differ in the molecules they inherit.

Contrast the causes of necrosis and apoptosis and describe the consequences of each on nearby cells and tissues.

While apoptosis often provides beneficial effects to the organism, necrosis is almost always detrimental and can be fatal.

How or where do most transcription regulators bind?

as dimers

Give a molecular definition of the term gene.

at the molecular level, a gene is defined as an organized unit of DNA sequences that enables a segment of DNA to be transcribed into RNA, resulting in the formation of a functional product.

Indicate whether each of the following statements refers to a ciliary microtubule, a microtubule of the mitotic spindle, both types of microtubule, or neither type of microtubule. A. The basal body is the organizing center. B. The monomer is sequestered by profilin. C. It is arranged in a "9 + 2" array. D. It is nucleated at the centrosome. E. It uses dynein motors. F. It is involved in sperm motility. G. It is involved in moving fluid over the surface of cells.

ciliary, neither, ciliary, mitotic, both, neither, ciliary

List the functions of helicase, topoisomerase, single-strand binding proteins, primase, and DNA polymerase at the replication fork.

helicase- unwinds the helix topoisomerase- prevents the DNA from becoming too tightly coiled ahead of the replication fork single-strand binding proteins- prevent the helix from re-forming primase- forms an RNA primer DNA polymerase- an enzyme that catalyzes the addition of nucleotides to the 3' end of a growing DNA strand.

Would they be higher or lower than in normal cells treated with adrenaline? A. cells that lack adenylyl cyclase B. cells that lack the GPCR C. cells that lack cAMP phosphodiesterase D. cells that have an α subunit that cannot hydrolyze GTP but can interact properly with the β and γ subunits

higher, higher, lower, lower

What is a phragmoplast and what is its function?

phragmoplast: structure that is formed in plant cells between two daughter nuclei during cytokinesis; made up of actin and myosin filaments functions: help in partitioning the two daughter cells during cytokinesis; helps the cell plate to fuse with the parent cell and two daughter cells separate with individual cell components

What is the most general term for a change in a single nucleotide pair?

point mutation

The nucleotide sequences of which of the following have been used to create a single all-encompassing tree of life?

rRNA gene

Recall why a ribosomal RNA gene was selected to construct a phylogenetic "tree of life."

rRNA is used because its genetic information has been conserved since the beginning of life.

The assembly of general transcription factors at a eukaryotic promoter typically begins at what site?

the TATA box

What is the difference between viruses and retroviruses?

the main difference between the two is how they replicate within a host cell. All viruses need to make messenger RNA (mRNA) to produce their proteins. Viruses do not have full instructions for producing proteins from DNA/RNA so they are dependent on the translational chemistry of the host cell. Retroviruses have an RNA genome, but not all RNA viruses are retroviruses. Retroviruses also have the enzymes, reverse transcriptase and retroviral integrase, these enzymes have the unique property of transcribing their RNA into DNA after entering a cell. The retroviral DNA can then integrate into the chromosomal DNA of the host cell where it is called a provirus.

Define epigenetics precisely

the study of heritable phenotype changes that do not involve alterations in the DNA sequence. The pattern of DNA methylation can be transmitted from one cell generation to the next, producing a form of epigenetic inheritance that helps a cell remember the state of gene expression in its parent cell. There is also evidence for a form of epigenetic inheritance based on transmitted chromatin structures.

Outline the general steps of pluripotent cell induction, and the use of the techniques for human health.

these reprogrammed cells behave much like naturally occurring ES cells, and they can be directed to generate a variety of specialized differentiated cells. This approach, initially performed using cultured fibroblasts, has been adapted to produce iPS cells from a variety of specialized cell types, including those taken from humans. Differentiated cells produced from human iPS cells are currently being used in the study or treatment of disease.

If DNA is damaged or incompletely replicated in S phase, the inhibition of which of these molecules prevents the cell from entering M phase?

Cdc25

Which of the following statements about meiosis is true? (a) During meiosis, the paternal chromosomes pair with the maternal chromosomes before lining up at the metaphase plate. (b) Unicellular organisms that have a haploid state undergo meiosis instead of mitosis during cell division. (c) Meiosis produces four genetically identical cells. (d) In general, meiosis is faster than mitosis.

(a) During meiosis, the paternal chromosomes pair with the maternal chromosomes before lining up at the metaphase plate

Which of the following statements is true? (a) Extracellular signal molecules that are hydrophilic must bind to a cellsurface receptor so as to signal a target cell to change its behavior. (b) To function, all extracellular signal molecules must be transported by their receptor across the plasma membrane into the cytosol. (c) A cell-surface receptor capable of binding only one type of signal molecule can mediate only one kind of cell response. (d) Any foreign substance that binds to a receptor for a normal signal molecule will always induce the same response that is produced by that signal molecule on the same cell type.

(a) Extracellular signal molecules that are hydrophilic must bind to a cellsurface receptor so as to signal a target cell to change its behavior

Which of the following statements about homologous genes is true? (a) For protein-coding genes, homologous genes will show more similarity in their amino acid sequences than in their nucleotide sequences. (b) Fewer than 1% of human genes have homologs in the nematode and the fruit fly. (c) Most homologous genes arose by gene duplication. (d) A gene in humans that has homologs in plants and prokaryotes will show the same level of similarity in nucleotide sequence when the human and prokaryotic sequences are compared as when the human and chimpanzee sequences are compared.

(a) For protein-coding genes, homologous genes will show more similarity in their amino acid sequences than in their nucleotide sequences

Which of the following statements about membrane-enclosed organelles is true? (a) In a typical cell, the area of the endoplasmic reticulum membrane far exceeds the area of plasma membrane. (b) The nucleus is the only organelle that is surrounded by a double membrane. (c) Other than the nucleus, most organelles are small and thus, in a typical cell, only about 10% of a cell's volume is occupied by membrane enclosed organelles; the other 90% of the cell volume is the cytosol. (d) The nucleus is the only organelle that contains DNA.

(a) In a typical cell, the area of the endoplasmic reticulum membrane far exceeds the area of plasma membrane

Which of the following statements is true? (a) The intron structure of most genes is conserved among vertebrates. (b) The more nucleotides there are in an organism's genome, the more genes there will be in its genome. (c) Because the fly Drosophila melanogaster and humans diverged from a common ancestor so long ago, a gene in the fly will show more similarity to another gene from the same species than it will to a human gene. (d) An organism from the same Order as another will be more likely to have a genome of the same size than will a more evolutionarily diverged animal.

(a) The intron structure of most genes is conserved among vertebrates

You are interested in cell-size regulation and discover that signaling through a GPCR called ERC1 is important in controlling cell size in embryonic rat cells. The G protein downstream of ERC1 activates adenylyl cyclase, which ultimately leads to the activation of PKA. You discover that cells that lack ERC1 are 15% smaller than normal cells, while cells that express a mutant, constitutively activated version of PKA are 15% larger than normal cells. Given these results, which of the following treatments to embryonic rat cells should lead to smaller cells? (a) addition of a drug that causes cyclic AMP phosphodiesterase to be hyperactive (b) addition of a drug that prevents GTP hydrolysis by Gα (c) addition of a drug that activates adenylyl cyclase (d) addition of a drug that mimics the ligand of ERC1

(a) addition of a drug that causes cyclic AMP phosphodiesterase to be hyperactive

Acetylcholine is a signaling molecule that elicits responses from heart muscle cells, salivary gland cells, and skeletal muscle cells. Which of the following statements is false? (a) Heart muscle cells decrease their rate and force of contraction when they receive acetylcholine, whereas skeletal muscle cells contract. (b) Heart muscle cells, salivary gland cells, and skeletal muscle cells all express an acetylcholine receptor that belongs to the transmitter-gated ion channel family. (c) Active acetylcholine receptors on salivary gland cells and heart muscle cells activate different intracellular signaling pathways. (d) Heart muscle cells, salivary gland cells, and skeletal muscle cells all respond to acetylcholine within minutes of receiving the signal.

(b) Heart muscle cells, salivary gland cells, and skeletal muscle cells all express an acetylcholine receptor that belongs to the transmitter-gated ion channel family

A poison added to an in vitro translation mixture containing mRNA molecules with the sequence 5′-AUGAAAAAAAAAAAAUAA-3′ has the following effect: the only product made is a Met-Lys dipeptide that remains attached to the ribosome. What is the most likely way in which the poison acts to inhibit protein synthesis? (a) It inhibits peptidyl transferase activity. (b) It inhibits movement of the small subunit relative to the large subunit. (c) It inhibits release factor. (d) It mimics release factor.

(b) It inhibits movement of the small subunit relative to the large subunit

In humans and in chimpanzees, 99% of the Alu retrotransposons are in corresponding positions. Which of the following statements below is the most likely explanation for this similarity? (a) The Alu retrotransposon is not capable of transposition in humans. (b) Most of the Alu sequences in the chimpanzee genome underwent duplication and divergence before humans and chimpanzees diverged. (c) The Alu retrotransposons are in the most beneficial position in the genome for primates. (d) The Alu retrotransposons must also be in the same position in flies.

(b) Most of the Alu sequences in the chimpanzee genome underwent duplication and divergence before humans and chimpanzees diverged

The local mediator nitric oxide stimulates the intracellular enzyme guanylyl cyclase by ________. (a) activating a G protein. (b) activating a receptor tyrosine kinase. (c) diffusing into cells and stimulating the cyclase directly. (d) activating an intracellular protein kinase.

(c) diffusing into cells and stimulating the cyclase directly

Intermediate filaments help protect animal cells from mechanical stress because ______. (a) filaments directly extend from the interior of the cell to the extracellular space and into the next cell, linking one cell to the next, helping to distribute locally applied forces. (b) filaments in each cell are indirectly connected to the filaments of a neighboring cell through the desmosome, creating a continuous mechanical link between cells. (c) filaments remain independent of other cytoskeletal elements and keep the mechanical stress away from other cellular components. (d) filaments make up the desmosome junctions that connect cells; these junctions are more important than the internal network of filaments for protecting cells against mechanical stress.

(b) filaments in each cell are indirectly connected to the filaments of a neighboring cell through the desmosome, creating a continuous mechanical link between cells

When a signal needs to be sent to most cells throughout a multicellular organism, the signal most suited for this is a _______. (a) neurotransmitter. (b) hormone. (c) dissolved gas. (d) scaffold.

(b) hormone

In mammals, there are two sex chromosomes, X and Y, which behave like homologous chromosomes during meiosis. Normal males have one X chromosome and one Y chromosome, and normal females have two X chromosomes. Males with an extra Y chromosome (XYY) are found occasionally. Which of the following could give rise to such an XYY male? Explain your answer. (a) nondisjunction in the first meiotic division of spermatogenesis; normal meiosis in the mother (b) nondisjunction in the second meiotic division of spermatogenesis; normal meiosis in the mother (c) nondisjunction in the first meiotic division of oogenesis; normal meiosis in the father (d) nondisjunction in the second meiotic division of oogenesis; normal meiosis in the father

(b) nondisjunction in the second meiotic division of spermatogenesis; normal meiosis in the mother

You examine a worm that has two genders: males that produce sperm and hermaphrodites that produce both sperm and eggs. The diploid adult has four homologous pairs of chromosomes that undergo very little recombination. Given a choice, the hermaphrodites prefer to mate with males, but just to annoy the worm, you pluck a hermaphrodite out of the wild and fertilize its eggs with its own sperm. Assuming that all the resulting offspring are viable, what fraction do you expect to be genetically identical to the parent worm? Assume that each chromosome in the original hermaphrodite is genetically distinct from its homolog. (a) all (b) none (c) 1/16 (d) 1/256

(c) 1/16

A finished draft of the human genome was published in ______. (a) 1965. (b) 1984. (c) 2004. (d) 2012.

(c) 2004

Which of the following statements about the unfolded protein response (UPR) is false? (a) Activation of the UPR results in the production of more ER membrane. (b) Activation of the UPR results in the production of more chaperone proteins. (c) Activation of the UPR occurs when receptors in the cytoplasm sense misfolded proteins. (d) Activation of the UPR results in the cytoplasmic activation of gene regulatory proteins.

(c) Activation of the UPR occurs when receptors in the cytoplasm sense misfolded proteins

Which of the following pairs of codons might you expect to be read by the same tRNA as a result of wobble? (a) CUU and UUU (b) GAU and GAA (c) CAC and CAU (d) AAU and AGU

(c) CAC and CAU

You are examining a cell line in which activation of the Rho family member Rac promotes lamellipodia formation. Which of the following statements is most likely to be true? (a) Cells carrying a Rac mutation that makes Rac act as if it is always bound to GTP will polymerize more unbranched actin filaments than normal cells. (b) Cells carrying a Rac mutation that makes Rac unable to exchange GDP for GTP will polymerize more unbranched actin filaments than normal cells. (c) Cells carrying a Rac mutation that makes Rac act as if it is always bound to GTP will polymerize more branched actin filaments than normal cells. (d) Cells carrying a Rac mutation that makes Rac unable to exchange GDP for GTP will polymerize more branched actin filaments than normal cells.

(c) Cells carrying a Rac mutation that makes Rac act as if it is always bound to GTP will polymerize more branched actin filaments than normal cells

Which of the following statements about the anaphase-promoting complex (APC) is false? (a) It promotes the degradation of proteins that regulate M phase. (b) It inhibits M-Cdk activity. (c) It is continuously active throughout the cell cycle. (d) M-Cdk stimulates its activity.

(c) It is continuously active throughout the cell cycle

Which of the following is not involved in post-transcriptional control? (a) the spliceosome (b) Dicer (c) Mediator (d) RISC

(c) Mediator

Which of the following statements about the proteasome is false? (a) Ubiquitin is a small protein that is covalently attached to proteins to mark them for delivery to the proteasome. (b) Proteases reside in the central cylinder of a proteasome. (c) Misfolded proteins are delivered to the proteasome, where they are sequestered from the cytoplasm and can attempt to refold. (d) The protein stoppers that surround the central cylinder of the proteasome use the energy from ATP hydrolysis to move proteins into the proteasome inner chamber.

(c) Misfolded proteins are delivered to the proteasome, where they are sequestered from the cytoplasm and can attempt to refold

You are studying nuclear lamins in yeast. Using recombinant DNA technology, you alter the coding sequence of a nuclear lamin gene such that the gene now codes for a nuclear lamin protein that can no longer be phosphorylated when the nuclear envelope is broken down during mitosis. What do you predict would happen if the yeast cell only had the altered nuclear lamin gene (and not the unaltered version)? (a) Mitosis should proceed as usual because the dephosphorylation of the lamin is what is important for nuclear lamina assembly during mitosis, so phosphorylation will not be necessary. (b) Disassembly of the nuclear lamins will occur prematurely because the lamins cannot be phosphorylated. (c) Nuclear lamins will no longer disassemble properly during mitosis. (d) Nuclear lamins will be unable to produce dimers, as coiled-coil formation will be disrupted.

(c) Nuclear lamins will no longer disassemble properly during mitosis

Which of the following statements about nuclear transport is true? (a) mRNAs and proteins transit the nucleus through different types of nuclear pores. (b) Nuclear import receptors bind to proteins in the cytosol and bring the proteins to the nuclear pores, where the proteins are released from the receptors into the pores for transit into the nucleus. (c) Nuclear pores have water-filled passages that small, water-soluble molecules can pass through in a nonselective fashion. (d) Nuclear pores are made up of many copies of a single protein.

(c) Nuclear pores have water-filled passages that small, water-soluble molecules can pass through in a nonselective fashion

The lab you work in has discovered a previously unidentified extracellular signal molecule called QGF, a 75,000-dalton protein. You add purified QGF to different types of cells to determine its effect on these cells. When you add QGF to heart muscle cells, you observe an increase in cell contraction. When you add it to fibroblasts, they undergo cell division. When you add it to nerve cells, they die. When you add it to glial cells, you do not see any effect on cell division or survival. Given these observations, which of the following statements is most likely to be true? (a) Because it acts on so many diverse cell types, QGF probably diffuses across the plasma membrane into the cytoplasm of these cells. (b) Glial cells do not have a receptor for QGF. (c) QGF activates different intracellular signaling pathways in heart muscle cells, fibroblasts, and nerve cells to produce the different responses observed. (d) Heart muscle cells, fibroblasts, and nerve cells must all have the same receptor for QGF.

(c) QGF activates different intracellular signaling pathways in heart muscle cells, fibroblasts, and nerve cells to produce the different responses observed

Which of the following molecules is thought to have arisen first during evolution? (a) protein (b) DNA (c) RNA (d) all came to be at the same time

(c) RNA

HIV is a human retrovirus that integrates into the host cell's genome and will eventually replicate, produce viral proteins, and ultimately escape from the host cell. Which of the following proteins is not encoded in the HIV genome? (a) reverse transcriptase (b) envelope protein (c) RNA polymerase (d) capsid protein

(c) RNA polymerase

Ribozymes catalyze which of the following reactions? (a) DNA synthesis (b) transcription (c) RNA splicing (d) protein hydrolysis

(c) RNA splicing

Which one of the following is the main reason that a typical eukaryotic gene is able to respond to a far greater variety of regulatory signals than a typical prokaryotic gene or operon? (a) Eukaryotes have three types of RNA polymerase. (b) Eukaryotic RNA polymerases require general transcription factors. (c) The transcription of a eukaryotic gene can be influenced by proteins that bind far from the promoter. (d) Prokaryotic genes are packaged into nucleosomes.

(c) The transcription of a eukaryotic gene can be influenced by proteins that bind far from the promoter.

The following happens when a G-protein-coupled receptor activates a G protein. (a) The β subunit exchanges its bound GDP for GTP. (b) The GDP bound to the α subunit is phosphorylated to form bound GTP. (c) The α subunit exchanges its bound GDP for GTP. (d) It activates the α subunit and inactivates the βγ complex.

(c) The α subunit exchanges its bound GDP for GTP

What is the most likely explanation of why the overall mutation rates in bacteria and in humans are roughly similar? (a) Cell division needs to be fast. (b) Most mutations are silent. (c) There is a narrow range of mutation rates that offers an optimal balance between keeping the genome stable and generating sufficient diversity in a population. (d) It benefits a multicellular organism to have some variability among its cells.

(c) There is a narrow range of mutation rates that offers an optimal balance between keeping the genome stable and generating sufficient diversity in a population

There are organisms that go through meiosis but do not undergo recombination when forming haploid gametes. Which of the following statements correctly describes the gametes produced by such an organism. (Assume that these organisms are diploid, that each of the two homologous chromosomes are genetically distinct as typically found in the wild, and that these organisms have more than one chromosome.) (a) All gametes formed during a single meiosis will be identical. (b) Due to the random assortment of homologs, each of the gametes formed during a single meiosis will be different. (c) This organism could potentially produce 2n genetically distinct gametes, where n is its haploid number of chromosomes. (d) The fusion of any two gametes produced by such an organism that does not undergo recombination during meiosis will create a cell that is genetically identical to that individual.

(c) This organism could potentially produce 2n genetically distinct gametes, where n is its haploid number of chromosomes

Which of the following does not describe a situation of asexual reproduction? (a) A bacterium multiplying by simple cell division. (b) Using a part of a plant to create a new independent plant. (c) Using in vitro fertilization to combine a sperm and an egg to create an embryo. (d) The parthenogenetic development of eggs produced by some species of lizards.

(c) Using in vitro fertilization to combine a sperm and an egg to create an embryo

Which of the following statements about how fruit flies can develop an eye in the middle of a leg is true? (a) When the Ey gene is expressed in adult leg cells, these cells dedifferentiate and become eye cells. (b) The Ey gene encodes a transcription regulator that is the only transcription regulator used to produce a fruit-fly eye. (c) When the Ey gene is introduced into cells that would normally give rise to a leg, the transcription regulators used to control its expression in the leg are different from those that are normally used to control Ey expression in the eye. (d) All the eye cells found in the adult leg are a single cell type and have identical characteristics.

(c) When the Ey gene is introduced into cells that would normally give rise to a leg, the transcription regulators used to control its expression in the leg are different from those that are normally used to control Ey expression in the eye

Which of the following changes is least likely to arise from a point mutation in a regulatory region of a gene? (a) a mutation that changes the time in an organism's life during which a protein is expressed (b) a mutation that eliminates the production of a protein in a specific cell type (c) a mutation that changes the subcellular localization of a protein (d) a mutation that increases the level of protein production in a cell

(c) a mutation that changes the subcellular localization of a protein

Which of the following would not lead to aneuploidy during meiosis? (a) sister chromatids segregating inappropriately (b) non-sister chromatids segregating inappropriately (c) a reciprocal rearrangement of parts between nonhomologous chromosomes (for example, the left arm of Chromosome 2 exchanging places with the right arm of Chromosome 3) (d) an extra set of chromosomes produced during S phase (for example, if paternal Chromosome 3 were replicated twice)

(c) a reciprocal rearrangement of parts between nonhomologous chromosomes (for example, the left arm of Chromosome 2 exchanging places with the right arm of Chromosome 3)

Vesicles from the ER enter the Golgi at the ______. (a) medial cisternae. (b) trans Golgi network. (c) cis Golgi network. (d) trans cisternae.

(c) cis Golgi network

The concentration of mitotic cyclin (M cyclin) ________. (a) rises markedly during M phase. (b) is activated by phosphorylation. (c) falls toward the end of M phase as a result of ubiquitylation and degradation. (d) is highest in G1 phase.

(c) falls toward the end of M phase as a result of ubiquitylation and degradation

In which cellular location would you expect to find ribosomes translating mRNAs that encode ribosomal proteins? (a) the nucleus (b) on the rough ER (c) in the cytosol (d) in the lumen of the ER

(c) in the cytosol

The Retinoblastoma (Rb) protein blocks cells from entering the cell cycle by ______. (a) phosphorylating Cdk. (b) marking cyclins for destruction by proteolysis. (c) inhibiting cyclin transcription. (d) activating apoptosis.

(c) inhibiting cyclin transcription

Foreign substances like nicotine, morphine, and menthol exert their initial effects by _____. (a) killing cells immediately, exerting their physiological effects by causing cell death. (b) diffusing through cell plasma membranes and binding to transcription factors to change gene expression. (c) interacting with cell-surface receptors, causing the receptors to transduce signal inappropriately in the absence of the normal stimulus. (d) removing cell-surface receptors from the plasma membrane.

(c) interacting with cell-surface receptors, causing the receptors to transduce signal inappropriately in the absence of the normal stimulus

A protein kinase can act as an integrating device in signaling if it ______. (a) phosphorylates more than one substrate. (b) catalyzes its own phosphorylation. (c) is activated by two or more proteins in different signaling pathways. (d) initiates a phosphorylation cascade involving two or more protein kinases.

(c) is activated by two or more proteins in different signaling pathways

In principle, how many different cell types can an organism having four different types of transcription regulator and thousands of genes create? (a) up to 4 (b) up to 8 (c) up to 16 (d) thousands

(c) up to 16

You are studying a gene that has four exons and can undergo alternative splicing. Exon 1 has two alternatives, exon 2 has five alternatives, exon 3 has three alternatives, and exon 4 has four alternatives. If all possible splicing combinations were used, how many different splice isoforms could be produced for this gene? (a) 22 (b) 30 (c) 60 (d) 120

(d) 120 (2x5x3x4)

An extraterrestrial organism (ET) is discovered whose basic cell biology seems pretty much the same as that of terrestrial organisms except that it uses a different genetic code to translate RNA into protein. You set out to break the code by translation experiments using RNAs of known sequence and cell-free extracts of ET cells to supply the necessary protein-synthesizing machinery. In experiments using the RNAs below, the following results were obtained when the 20 possible amino acids were added either singly or in different combinations of two or three: RNA 1: 5′-GCGCGCGCGCGCGCGCGCGCGCGCGCGC-3′ RNA 2: 5′-GCCGCCGCCGCCGCCGCCGCCGCCGCCGCC-3′ Using RNA 1, a polypeptide was produced only if alanine and valine were added to the reaction mixture. Using RNA 2, a polypeptide was produced only if leucine and serine and cysteine were added to the reaction mixture. Assuming that protein synthesis can start anywhere on the template, that the ET genetic code is nonoverlapping and linear, and that each codon is the same length (like the terrestrial triplet code), how many nucleotides does an ET codon contain? (a) 2 (b) 3 (c) 4 (d) 5 (e) 6

(d) 5

Viral genomes _________. (a) can be made of DNA. (b) can be made of RNA. (c) can be either double-stranded or single-stranded. (d) All answers above are true.

(d) All answers above are true

MPF activity was discovered when cytoplasm from a Xenopus M-phase cell was injected into Xenopus oocytes, inducing the oocytes to form a mitotic spindle. In a control experiment, Xenopus interphase cytoplasm was injected into oocytes and shown not to induce the formation of a mitotic spindle. Which of the following statements is not a legitimate conclusion from the control experiment? (a) The piercing of the oocyte membrane by a needle is insufficient to cause mitotic spindle formation. (b) An increased volume of cytoplasm is insufficient to cause mitotic spindle formation. (c) Injection of extra RNA molecules is insufficient to cause mitotic spindle formation. (d) Components of an interphase nucleus are insufficient to cause mitotic spindle formation.

(d) Components of an interphase nucleus are insufficient to cause mitotic spindle formation

Which of the following statements is false? (a) Nucleotides and amino acids can act as extracellular signal molecules. (b) Some signal molecules can bind directly to intracellular proteins that bind DNA and regulate gene transcription. (c) Some signal molecules are transmembrane proteins. (d) Dissolved gases such as nitric oxide (NO) can act as signal molecules, but because they cannot interact with proteins they must act by affecting membrane lipids.

(d) Dissolved gases such as nitric oxide (NO) can act as signal molecules, but because they cannot interact with proteins they must act by affecting membrane lipids

Which of the following statements about differentiated cells is true? (a) Cells of distinct types express nonoverlapping sets of transcription factors. (b) Once a cell has differentiated, it can no longer change its gene expression. (c) Once a cell has differentiated, it will no longer need to transcribe RNA. (d) Some of the proteins found in differentiated cells are found in all cells of a multicellular organism.

(d) Some of the proteins found in differentiated cells are found in all cells of a multicellular organism

Which of the following statements about a protein in the lumen of the ER is false? (a) A protein in the lumen of the ER is synthesized by ribosomes on the ER membrane. (b) Some of the proteins in the lumen of the ER can end up in the extracellular space. (c) Some of the proteins in the lumen of the ER can end up in the lumen of an organelle in the endomembrane system. (d) Some of the proteins in the lumen of the ER can end up in the plasma membrane.

(d) Some of the proteins in the lumen of the ER can end up in the plasma membrane

Which of the following statements is false? (a) Mitotic Cdk must be phosphorylated by an activating kinase (Cak) before it is active. (b) Phosphorylation of mitotic Cdk by the inhibitory kinase (Wee1) makes the Cdk inactive, even if it is phosphorylated by the activating kinase. (c) Active M-Cdk phosphorylates the activating phosphatase (Cdc25) in a positive feedback loop. (d) The activating phosphatase (Cdc25) removes all phosphates from mitotic Cdk so that M-Cdk will be active.

(d) The activating phosphatase (Cdc25) removes all phosphates from mitotic Cdk so that M-Cdk will be active

Which of the following statements is false? (a) A new RNA molecule can begin to be synthesized from a gene before the previous RNA molecule's synthesis is completed. (b) If two genes are to be expressed in a cell, these two genes can be transcribed with different efficiencies. (c) RNA polymerase is responsible for both unwinding the DNA helix and catalyzing the formation of the phosphodiester bonds between nucleotides. (d) Unlike DNA, RNA uses a uracil base and a deoxyribose sugar.

(d) Unlike DNA, RNA uses a uracil base and a deoxyribose sugar.

Describe the structure of nucleosomes.

-Made up of eight proteins consisting of pairs of four types of histones. DNA makes 1.75 turns around this octomer. -The "string" part is known as the linker DNA They are formed by wrapping DNA around a core protein octamer composed of two subunits each of histones H2A, H2B, H3, and H4

Outline the experiments which led to the discovery that DNA was the molecule of inheritance.

1920- Griffith confirmed that when the S form is killed by heating, it is no longer infectious. But he then discovered that if he injected mice with both the heat-killed S-form pneumococci and with live, harmless bacteria, the animals died of pneumonia. Proved protein is not the genetic material. 1940s- Avery, MacLeod, and McCarty performed an experiment demonstrating that DNA is the genetic material. The researchers prepared extracts from a disease-causing S strain of pneumococcus and showed that the "transforming principle" that would permanently change a harmless R strain into the infectious S strain is DNA. 1952- researchers working with a virus called T2 conducted a different kind of test to determine whether genes are made of DNA or protein. When this virus—which is made entirely of DNA and protein—infects E. coli, it injects its genetic material into the bacterial cell, leaving the empty virus head stuck to the cell surface. 1950s-Rosalind Franklin and Maurice Wilkins studied DNA with x-ray diffraction analysis and determined its 3-D atomic structure. 1953- Their results led to Watson and Crick's model of the double-helical structure of DNA.

Match the components involved with ER transport with the appropriate cellular location. Locations can be used more than once, or not at all. 1. signal-recognition particle _____ 2. protein translocator _____ 3. mRNA _____ 4. SRP receptor _____ 5. active site of signal peptidase ____ A. cytosol, B. ER lumen, C. ER membrane

1—A; 2—C; 3—A; 4—C; 5—B

Each type of genetic change may be used more than once, or may not be used at all. Phenotypic changes: 1. A protein normally localized in the nucleus is now localized in the cytoplasm. 2. A protein acquires a DNA-binding domain. 3. Tandem copies of a gene are found in the genome. 4. A copy of a bacterial gene is now found integrated on a human chromosome. 5. A protein becomes much more unstable. 6. A protein normally expressed only in the liver is now expressed in blood cells. A. mutation within a gene, B. gene duplication, C. mutation in a regulatory region, D. exon shuffling, E. horizontal gene transfer

1—A; 2—D; 3—B; 4—E; 5—A; 6—C

Rank the following cytoskeletal filaments from smallest to largest in diameter (1 = smallest in diameter, 4 = largest) ______ intermediate filaments ______ microtubules ______ actin filament ______ myofibril

2, 3, 1, 4

Which of the following statements are correct? Explain your answers. A.Cells do not pass from G1 into the M phase of the cell cycle unless there are sufficient nutrients to complete an entire cell cycle.

A. False. There is no G1 to M phase transition. The statement is correct, however, for the G1 to S phase transition, in which cells commit themselves to a division cycle. B.Apoptosis is mediated by special intracellular proteases, one of which cleaves nuclear lamins. TRUE C.Developing neurons compete for limited amounts of survival factors. TRUE D.Some vertebrate cell-cycle control proteins function when expressed in yeast cells. TRUE E.The enzymatic activity of a Cdk protein is determined both by the presence of a bound cyclin and by the phosphorylation state of the Cdk. TRUE

Circle the phrase in each pair that is likely to occur more rapidly in response to an extracellular signal. A. changes in cell secretion / increased cell division B. changes in protein phosphorylation / changes in proteins being synthesized C. changes in mRNA levels / changes in membrane potential

A. changes in cell secretion B. changes in protein phosphorylation C. changes in membrane potential

Indicate whether each of the following is true for meiosis, mitosis, both, or neither. A. formation of a bivalent B. genetically identical products C. condensation of chromosomes D. segregation of all paternal chromosomes to one cell E. involvement of DNA replication

A. meiosis, B. mitosis, C. both, D. neither, E. both

Describe alternative splicing, what is it? Why is it important?

Alternative splicing is a process that enables a messenger RNA (mRNA) to direct synthesis of different protein variants (isoforms) that may have different cellular functions or properties. It occurs by rearranging the pattern of intron and exon elements that are joined by splicing to alter the mRNA coding sequence.

Which of the following statements are correct? Explain your answers.

A.Centrosomes are replicated before the M phase begins. TRUE B.Two sister chromatids arise by replication of the DNA of the same chromosome and remain paired as they line up on the metaphase plate. TRUE C.Interpolar microtubules attach end-to-end and are therefore continuous from one spindle pole to the other. False. The ends of interpolar microtubules overlap and attach to one another via proteins (including motor proteins) that bridge between the microtubules. D.Microtubule polymerization and depolymerization and microtubule motor proteins are all required for DNA replication. False. Microtubules and their motor proteins play no role in DNA replication. E.Microtubules nucleate at the centromeres and then connect to the kinetochores, which are structures at the centrosome regions of chromosomes. False. To be a correct statement, the terms "centromere" and "centrosome" must be switched

What do you suppose happens in mutant cells that:

A.cannot degrade M cyclin? would enter mitosis but would not be able to exit mitosis. B.always express high levels of p21? would arrest permanently in G1 because the cyclin-Cdk complexes that act in G1 would be inactivated. C.cannot phosphorylate Rb? would not be able to activate the transcription of genes required for cell division because the required transcription regulators would be constantly inhibited by unphosphorylated Rb.

The translation of an mRNA begins at the start codon. What is the sequence of this codon?

AUG

What is the function of tRNA?

Acts as adaptor molecules in proteins synthesis

Summarize the factors that determine the duration of a GPCR-stimulated response.

Amount of time either subunit is switched on and able to relay signals is determined by the α subunit. α subunit has intrinsic GTPase activity (ability to hydrolyze GTP to GDP), returning the entire G protein back to original inactive confirmation. This usually occurs within seconds after activation. Inactive G protein is ready to be reactivated by another activated receptor. Activated α activates target protein.

Outline how gene duplication and divergence gave rise to the globin gene family.

An ancestral globin gene encoding a single-chain globin molecule gave rise to the pair of genes that produce four-chain hemoglobin proteins of modern humans and other mammals.

Summarize the molecular events that trigger the separation of sister chromatids at the start of anaphase.

Anaphase begins abruptly with the breakage of the cohesin linkages that hold together the sister chromatids in a duplicated chromosome. This release allows each chromosome to be pulled toward the spindle pole to which it is attached. The cohesin linkage is destroyed by a protease called separase. Before anaphase begins, this protease is held in an inactive state by an inhibitory protein called securing. At the beginning of anaphase, securin is targeted for destruction by APC/C - same protein complex that marks M cyclin for degradation. Once securin is removed, separase is free to sever linkages.

Explain how apoptosis participates in the development of a mouse or a human—or in the metamorphosis of a frog.

Apoptosis is programmed cell death. The amount of apoptosis that occurs in both developing and adult animal tissues can be astonishing. In the developing vertebrate system for example, more than half of some types of nerve cells normally die soon after they are formed. In a healthy adult human, billions of cells in the bone marrow and intestine perish every hour. Cell death balances cell division, unless the tissue is growing or shrinking.

Summarize the mechanisms that allow cells to either pause or continue through different transition points in the cell cycle.

At the G1 to S transition, it uses Cdk inhibitors to keep cells from entering S phase and replicating their DNA. At the G2 to M transition, it suppresses the activation of M-Cdk by inhibiting the phosphatase required to activate the Cdk. And it can then delay the exit from mitosis by inhibiting the activation of APC/C, thus preventing the degradation of M cyclin. The control system monitors the cell's internal state and conditions in its environment, before allowing the cell to continue through the cycle. For example, it allows entry into S phase only if environmental conditions are appropriate; it triggers mitosis only after the DNA has been completely replicated; and it initiates chromosome segregation only after the duplicated chromosomes are correctly aligned on the mitotic spindle.

Review how chromosomes are captured by spindle microtubules, and describe the structure of the point of attachment.

Attachment of microtubules to chromosomes is mediated by kinetochores, which actively monitor spindle formation and prevent premature anaphase onset. Microtubule polymerization and depolymerization dynamic drive chromosome congression. Depolymerization of microtubules generates tension at kinetochores.

Define bi-orientation and explain its importance for chromosome segregation.

Bi-orientation is the symmetrical attachment of a sister chromatid pair on the mitotic spindle, such that one chromatid in the duplicated chromosome is attached to one spindle pole and the other is attached to the opposite pole. This process generates tension in the kinetochores, which are being pulled in opposite directions. This tension signals to the sister kinetochores that they are attached correctly and are ready to be separated.

Articulate how viruses differ from mobile genetic elements.

Both are mobile, but viruses can move to other cells and organisms. Viruses are small genomes and their reproduction can lead to the death of host cells.

Cells signal to one another in various ways. Some use extracellular signal molecules that are dissolved gases, such as ______, which can diffuse easily into cells. Others use cytokines, which bind to cytokine receptors. Cytokine receptors have no intrinsic enzyme activity but are associated with cytoplasmic tyrosine kinases called ______s, which become activated on the binding of cytokine to its receptor and go on to phosphorylate and activate cytoplasmic transcriptional regulators called ______s. Some intracellular signaling pathways involve chains of protein kinases that phosphorylate each other, as seen in the ______ signaling module. Lipids can also relay signals in the cell, as we observe when phospholipase C cleaves the sugar-phosphate head off a lipid molecule to generate the two small messenger molecules ______ (which remains embedded in the plasma membrane) and ______ (which diffuses into the cytosol). cyclic GMP, MAP kinase, STAT, diacylglycerol, NO, TGFβ, IP3, Ras, JAK, SMAD

Cells signal to one another in various ways. Some use extracellular signal molecules that are dissolved gases, such as NO, which can diffuse easily into cells. Others use cytokines, which bind to cytokine receptors. Cytokine receptors have no intrinsic enzyme activity but are associated with cytoplasmic tyrosine kinases called JAKs, which become activated on the binding of cytokine to its receptor and go on to phosphorylate and activate cytoplasmic transcriptional regulators called STATs. Some intracellular signaling pathways involve chains of protein kinases that phosphorylate each other, as seen in the MAP kinase signaling module. Lipids can also relay signals in the cell, as we observe when phospholipase C cleaves the sugar-phosphate head off a lipid molecule to generate the two small messenger molecules diacylglycerol (which remains embedded in the plasma membrane) and IP3 (which diffuses into the cytosol).

Which of the following represents the specialized DNA sequence that attaches to microtubules and allows duplicated eukaryotic chromosomes to be separated during M phase?

Centromere

Outline the centrosome cycle, indicating how and when it is initiated.

Centrosome duplication begins at the same time as DNA replication and the process is triggered by the same Cdks - G1/S-Cdk and S-Cdk - that initiate DNA replication. Initially, when the centrosome duplicates, both copies remain together as a single complex on one side of the nucleus. As mitosis begins, however, the two centrosomes separate, and each nucleates a radial array of microtubules called an aster. The two asters move to opposite sides of the nucleus to form the two poles of their mitotic spindle.

Recall the type of cells that lack centrosomes.

Centrosomes are not essential in somatic cells in fruit flies, and many animal cells don't have them.

Contrast how cholera toxin and pertussis toxin exert their effects.

Cholera toxin - protein produced by bacterium that multiplies in intestine · toxin enters cells that line intestine & modifies α (called Gs because it stimulates adenylyl cyclase enzyme · mod prevents Gs from hydrolyzing the bound GTP, locking it in active state, continuously stimulating adenylyl cyclase This results in prolonged & excessive outflow of Cl- and water into gut = diarrhea and dehydration, can lead to death without replacement of water and ions Pertussis toxin - respiratory infection, bacterium colonizes lung, producing toxin, whooping cough · toxin (protein) alters α (Gi because it inhibits adenylyl cyclase) · mod disables subunit by locking it in inactive state (GDP bound) · results in inability to stimulate adenylyl cyclase

Describe some stages of the cell cycle:

Chromosomal segregation, cytokinesis, DNA replication

What role does cohesin play in the transition from metaphase to anaphase?

Cohesis are a protein complex that holds sister chromatids together. They assemble along the length of each chromatid as the DNA is replicated. This cohesion between sister chromatids is crucial for proper chromosome segregation, and it is broken completely only in late mitosis to allow the sisters to be pulled apart by the mitotic spindle. Anaphase begins abruptly with the breakage of the cohesin linkages that hold together the sister chromatids in a duplicated chromosome. This release allows each chromosome to be pulled toward the spindle pole to which it is attached. The cohesin linkage is destroyed by a protease called separase.

S-CDK/S-cyclin are very important to proper cell division. Why?

Cyclins drive the events of the cell cycle by partnering with a family of enzymes called the cyclin-dependent kinases (Cdks). A lone Cdk is inactive, but the binding of a cyclin activates it, making it a functional enzyme and allowing it to modify target proteins. The cyclin that acts in G2 to trigger entry into M phase is called the M cyclin, and the active complex it forms with its Cdk is called M-Cdk. Other cyclins, called S cyclins and G1/S cyclins, bidn to a distinct Cdk protein late in G1 to form S-Cdk and G1/S-Cdk, respectively; these cyclin-Cdk complexes help launch S phase. Another group of cyclins, called G1 cyclins, act earlier in G1 and bind to other Cdk proteins to form G1-Cdks, which help drive the cell through G1 toward S phase.

The following proteins are important for cell movement. Match the following proteins with their function. myosin _____ ARP proteins _____ profilin _____ integrins _____ formins _____ A. nucleation of new actin filaments at the side of an existing filament, B. regulation of the availability of actin monomers, C. important for the growth of straight, unbranched actin filaments, D. contracting the rear of the cell, E. involvement in focal contacts

D, A, B, E, C

Match the class of cell-surface receptor with the best description of its function G-protein-coupled receptors ______ ion-channel-coupled receptors ______ enzyme-coupled receptors ______ A. alter the membrane potential directly by changing the permeability of the plasma membrane B. signal by opening and closing in a ligand-independent manner C. must be coupled with intracellular monomeric GTP-binding proteins D. all receptors of this class are polypeptides with seven transmembrane domains E. discovered for their role in responding to growth factors in animal cells

D, A, E

Review how DNA damage can arrest cells in G1.

DNA damage in G1 causes an increase in both the concentration and activity of a protein called p53, which is a transcription regulator that activates the gene encoding a Cdk inhibitor protein called p21. The p21 protein binds to G1/S-Cdk and S-Cdk, preventing them from driving the cell into S phase. The arrest of the cell cycle gives the cell time to repair the damaged DNA.

Explain how DNA damage can occur in cells.

DNA damage occurs continuously as a result of various factors—intracellular metabolism, replication, and exposure to genotoxic agents, such as ionizing radiation and chemotherapy. If left unrepaired, this damage could result in changes or mutations within the cell genomic material.

Describe the cellular mechanisms for repairing DNA.

DNA ligase can seal the nick in damaged or incorrect DNA. DNA mismatch repair removes the replication errors that escape proofreading. Double-stranded DNA breaks require nonhomologous and homologous end joining. nonhomologous end repair results in the loss of some nucleotides at the repair site, but is energetically favorable. homologous end-joining does not lose any nucleotides but expends more energy.

Describe the mechanisms underlying post-transcriptional control (DNA methylation, histone modifications, RNA degradation & translation, miRNA, siRNA, and long noncoding RNAs)

DNA methylation-addition of a methyl (CH3) group to DNA, thereby often modifying the function of the genes and affecting gene expression histone modifications- Addition of acetyl groups to particular lysines in histone tails neutralizes the positive charge and loosens the nucleosome's grip on DNA RNA degradation and translation- occurs on translating mRNA and that mRNA decay factors can inhibit translational elongation as well as affect mRNA degradation. These defects feed into the quality control of translational processes miRNA- bind to the 3'-UTR (untranslated region) of their target mRNAs and repress protein production by destabilizing the mRNA and translational silencing. siRNA- The complex silences certain gene expression by cleaving the mRNA molecules coding the target genes. This cleavage results in mRNA fragments that are further degraded by cellular exonucleases. long noncoding RNAs-(lncRNAs) play important roles in cancer. They are involved in chromatin remodeling, as well as transcriptional and post-transcriptional regulation, through a variety of chromatin-based mechanisms and via cross-talk with other RNA species. can repress target genes through various mechanisms. a | Recruitment of proteins that repress gene expression. The lncRNAs can be transcribed from near the target gene or be brought to its proximity through pre-formed chromatin loops

Before a cell divides:

DNA polymerase synthesizes new DNA in the 5ʹ-to-3ʹ direc-tion, only the leading strand

Explain how the AT/GC rule underlies the ability of DNA to be replicated semiconservatively.

DNA replication produces two daughter DNA double helicies that intertwine with an old parental strand.

The flow of genetic information in all living cells is:

DNA-RNA-PROTEIN

During transcription in __________________ cells, transcriptional regulators that bind to DNA thousands of nucleotides away from a gene's promoter can affect a gene's transcription. The __________________ is a complex of proteins that links distantly bound transcription regulators with the proteins bound closer to the transcriptional start site. Transcriptional activators can also interact with histone __________________s, which alter chromatin by modifying lysines in the tail of histone proteins to allow greater accessibility to the underlying DNA. Gene repressor proteins can reduce the efficiency of transcription initiation by attracting histone __________________s. Sometimes, many contiguous genes can become transcriptionally inactive as a result of chromatin remodeling, like the __________________ found in interphase chromosomes. Acetylase, eukaryotic, operator, centrosome, helicase, peroxidase, deacetylase, heterochromatin, prokaryotic, deoxidase, leucine zipper, telomere, enhancer, Mediator, viral

During transcription in eukaryotic cells, transcriptional regulators that bind to DNA thousands of nucleotides away from a gene's promoter can affect a gene's transcription. The Mediator is a complex of proteins that links distantly bound transcription regulators with the proteins bound closer to the transcriptional start site. Transcriptional activators can also interact with histone acetylases, which alter chromatin by modifying lysines in the tail of histone proteins to allow greater accessibility to the underlying DNA. Gene repressor proteins can reduce the efficiency of transcription initiation by attracting histone deacetylases. Sometimes, many contiguous genes can become transcriptionally inactive as a result of chromatin remodeling, like the heterochromatin found in interphase chromosomes.

Outline an experimental approach to determining the rate at which point mutations accumulate in bacteria.

E. coli divides once every 20-25 minutes, and produces several offspring per day. this increases the probability of causing a point mutation, which is observable and measurable in the lab.

Eukaryotic cells are continually taking up materials from the extracellular space by the process of endocytosis. One type of endocytosis is ________, which uses ______ proteins to form small vesicles containing fluids and molecules. After these vesicles have pinched off from the plasma membrane, they will fuse with the ______, where materials that are taken into the vesicle are sorted. A second type of endocytosis is ______, which is used to take up large vesicles that can contain microorganisms and cellular debris. Macrophages are especially suited for this process, as they extend ________ (sheetlike projections of their plasma membrane) to surround the invading microorganisms. Chaperone, Golgi apparatus, pseudopods, cholesterol, mycobacterium, rough ER, clathrin, phagocytosis, SNARE, endosome, pinocytosis, transcytosis

Eukaryotic cells are continually taking up materials from the extracellular space by the process of endocytosis. One type of endocytosis is pinocytosis, which uses clathrin proteins to form small vesicles containing fluids and molecules. After these vesicles have pinched off from the plasma membrane, they will fuse with the endosome, where materials that are taken into the vesicle are sorted. A second type of endocytosis is phagocytosis, which is used to take up large vesicles that can contain microorganisms and cellular debris. Macrophages are especially suited for this process, as they extend pseudopods (sheetlike projections of their plasma membrane) to surround the invading microorganisms

How many RNA-polymerase molecules are there and what does each polymerase do?

Eukaryotic cells contain three distinct nuclear RNA polymerases that transcribe different classes of genes. Protein-coding genes are transcribed by RNA polymerase II to yield mRNAs; ribosomal RNAs (rRNAs) and transfer RNAs (tRNAs) are transcribed by RNA polymerases I and III.

Which of the following does NOT cause a mutation?

Evolution

Define evolution. This can be done in one sentence.

Evolution is change in the heritable characteristics of biological populations over successive generations.

Express how exon shuffling can facilitate the evolution of new proteins.

Exon shuffling during evolution can generate proteins with new combinations of protein domains. These different protein domains were joined together by exon shuffling during evolution to create the modern-day human proteins.

Define chromatin, heterochromatin and euchromatin.

Heterochromatin and euchromatin are two major categories of chromatin higher order structure. Heterochromatin has condensed chromatin structure and is inactive for transcription, while euchromatin has loose chromatin structure and active for transcription.

Which of the following statements are correct? An individual ribosome can make only one type of protein. Explain your answers.

False. Ribosomes can make any protein that is specified by the particular mRNA that they are translating. After translation, ribosomes are released from the mRNA and can then start translating a different mRNA. It is true, however, that a ribosome can only make one type of protein at a time All mRNAs fold into particular three-dimensional structures that are required for their translation. False. mRNAs are translated as linear polymers; there is no requirement that they have any particular folded structure. In fact, such structures that are formed by mRNA can inhibit its translation, because the ribosome has to unfold the mRNA in order to read the message it contains The large and small subunits of an individual ribosome always stay together and never exchange partners. False. Ribosomal subunits can exchange partners after each round of translation. After a ribosome is released from an mRNA, its two subunits dissociate and enter a pool of free small and large subunits from which new ribosomes assemble around a new mRNA. Ribosomes are cytoplasmic organelles that are encapsulated by a single membrane. False. Ribosomes are not individually enclosed in a membrane. Because the two strands of DNA are complementary, the mRNA of a given gene can be synthesized using either strand as a template. False. The position of the promoter determines the direction in which transcription proceeds and therefore which of the two DNA strands is used as the template. Transcription of the other strand would produce an mRNA with a completely different (and in most cases meaningless) sequence An mRNA may contain the sequence ATTGACCCCGGTCAA. False. RNA contains uracil but not thymine. The amount of a protein present in a cell depends on its rate of synthesis, its catalytic activity, and its rate of degradation. False. The level of a protein depends on its rate of synthesis and degradation but not on its catalytic activity.

Most of the diversity in the rates of cell division in the adult body lies in variations in the time that each cell spends in which phase(s)?

G1 phase and G0 phase

Cells can pause in G1 when DNA is damaged, and can pause in S when there are replication errors. Indicate whether the mechanism below applies to a G1 arrest, an S-phase arrest, both types of arrest, or neither. A. p53 activates the transcription of a Cdk inhibitor. B. Cyclins are phosphorylated and destroyed. C. Cdk is unable to phosphorylate its substrates. D. The Cdc25 phosphatase is inhibited.

G1, neither, both S-phase

Review how different types of receptors can trigger a rise in the cytosolic concentration of calcium ions.

GPCR and RTKs can both trigger phospholipase C which will in turn cause a rise in Ca.

Compare the mechanisms of exon shuffling and gene duplication.

In exon shuffling, eukaryotic genes facilitate the evolution of new proteins by allowing exons from one gene to be added to another. Both mechanisms bring about change in new species and changes in genomes.

Compare and contrast the function of GEF and GAP proteins.

In general, GEFs turn on signaling by catalyzing the exchange from G-protein-bound GDP to GTP, whereas GAPs terminate signaling by inducing GTP hydrolysis. GEFs and GAPs are multidomain proteins that are regulated by extracellular signals and localized cues that control cellular events in time and space.

Explain how regulatory transcription factors are involved in the regulation of transcription.

In multicellular plants and animals, the production of different transcription regulators in different cell types ensures the expression of only those genes appropriate to the particular type of cell.

Intermediate filaments are elongated fibrous proteins with an N-terminal globular ______ region and a C-terminal globular _______ region; these regions flank the elongated rod domain. The α-helical region of the rod interacts with the α-helical region of another monomer in a _______ configuration to form a dimer. _______ dimers will line up to form a staggered tetramer. _____ strands of tetramers come together and twist together to form the _____ nm filament. The ______ domains are exposed on the surface of the intermediate filament, allowing for interaction with cytoplasmic components. Antiparallel, four, tail, β barrel, globular, ten, coiled-coil, head, trimeric, covalent, rod, twenty-five, eight, seven, two

Intermediate filaments are elongated fibrous proteins with an N-terminal globular head region and a C-terminal globular tail region; these regions flank the elongated rod domain. The α-helical region of the rod interacts with the α-helical region of another monomer in a coiled-coil configuration to form a dimer. Two dimers will line up to form a staggered tetramer. Eight strands of tetramers come together and twist together to form the ten nm filament. The globular domains are exposed on the surface of the intermediate filament, allowing for interaction with cytoplasmic components

Recall when cytokinesis takes place with respect to mitosis.

It completes M phase. It usually begins in anaphase but is not completed until after the two daughter nuclei have re-formed in telophase.

Why does nitric oxide (NO) act as a paracrine signal that affects only neighboring cells?

It is rapidly converted to nitrates and nitrites in the extracellular fluid.

What must happen for a Cdk to be active?

It must bind to its cyclin partner

Compare the mitotic spindle and contractile ring in terms of composition, location, and the role each plays in cell division.

Mitotic spindle: composed of microtubules and the various proteins that interact with them, including microtubule-associated motor proteins. In all eukaryotic cells, the mitotic spindle is responsible for separating the duplicated chromosomes and allocating one copy of each chromosome to each daughter cell. Contractile ring: consists mainly of actin and myosin filaments arranged in a ring around the equator of the cell. It starts to assemble just beneath the plasma membrane toward the end of mitosis. As the ring contracts, it pulls the membrane inward, thereby dividing the cell in two.

Most variation between individual humans is in the form of _________. ______ may arise by recombination within introns and can create proteins with novel combinations of domains. Scientists and government regulators must be very careful when introducing herbicide resistant transgenic corn plants into the environment, because if resistant weeds arise from ________ then the herbicides could become useless. Families of related genes can arise from a single ancestral copy by _______ and subsequent _________. Divergence, purifying selection, exon shuffling, single-nucleotide polymorphisms, gene duplication, synteny, horizontal gene transfer, unequal crossing-over

Most variation between individual humans is in the form of single-nucleotide polymorphisms. Exon shuffling may arise by recombination within introns and can create proteins with novel combinations of domains. Scientists and government regulators must be very careful when introducing herbicide-resistant transgenic corn plants into the environment, because if resistant weeds arise from horizontal gene transfer then the herbicides could become useless. Families of related genes can arise from a single ancestral copy by gene duplication and subsequent divergence.

List some foreign substances that alter physiology by interacting with cell-surface receptors.

Nicotine - stimulates acetylcholine activated ICCRs, resulting in restricted blood vessels and blood pressure elevation · Menthol - stimulates temperature sensitive ICCRs, resulting in: moderate amounts, cool sensation; high doses, burning pain · Barbiturates & Benzos - stimulate GABA activated ICCRs resulting in anxiety relief and sedation · Morphine & Heroin - stimulate G protein coupled opiate receptors, resulting in euphoria and analgesia (pain relief) · Curare - blocks acetylcholine activated ICCRs, results in paralysis by blockage of neuromuscular transmission · Capsaicin - stimulates temperature sensitive ICCRs, results in painful burning; prolonged exposure paradoxically leads to pain relief · Strychnine - blocks glycine activated ICCRs, blocks inhibitory synapses in spinal cord and brain, seizures, muscle spasms

Outline how the gas nitric oxide (NO) can act as a signaling molecule to trigger the relaxation of smooth muscle cells.

Nitric Oxide is small enough and hydrophobic enough to pass through plasma membrane (unlike cAMP and Ca). Acetylcholine binds to GPCR on surface of endothelial cells, activating G protein (Gq) · Active Gq à IP3 à release of Ca inside cell · Ca stimulates nitric oxide synthase, enzyme produces NO from arginine (amino acid) · NO diffuses rapidly from endothelial cell to smooth muscle cell · In smooth muscle cells it regulates activity of specific proteins that relax the cell · Key target protein activated by NO is guanylyl cyclase, catalyzes production of cyclic GMP from GTP Treatment of patients with angina (pain causes by inadequate blood flow to heart muscle) In body, nitroglycerin is converted into NO, relaxing blood vessels

Indicate by writing "yes" or "no" whether amplification of a signal could occur at the particular steps described below. A. An extracellular signaling molecule binds and activates a GPCR. B. The activated GPCRs cause Gα to separate from Gβ and Gγ. C. Adenylyl cyclase produces cyclic AMP. D. cAMP activates protein kinase A. E. Protein kinase A phosphorylates target proteins.

No, Yes, Yes, No, Yes

What type of enzyme removes damaged DNA from the rest of the DNA molecule?

Nuclease

Explain why the G1-to-S transition in yeast cells is sometimes called "Start."

Once past this critical G1-to-S transition, a cell usually continues all the way through the rest of the cell cycle. In yeasts, the G1-to-S transition is therefore sometimes called start because passing it represents a commitment to complete a full cell cycle.

Recall how a combination of signals can evoke a response that is different from the sum of the effects that each signal can trigger on its own.

One combination of signals is necessary for a cell to survive. The addition of other signals can cause a cell to differentiate or grow and divide. The absence of signals leads to apoptosis.

Tyrosine phosphorylation serves to build binding sites for other proteins to bind to RTKs.

TRUE

Indicate how Cdk inhibitors can help regulate progression through the cell cycle.

The cell cycle uses these inhibitors to block the assembly or activity of certain cyclin-Cdk complexes. Some Cdk inhibitor proteins, for example, help maintain Cdks in an inactive state during the G1 phase of the cycle, this delaying progression into S phase. Pausing at this transition point in G1 gives the cell more time to grow, or allow it to wait until extracellular conditions are favorable for division.

Sexual reproduction in a multicellular organism involves specialized reproductive cells, called ______s, which come together to form a _______ that will divide to produce both reproductive and _____ cells. A point mutation in the DNA is considered a ________ mutation if it changes a nucleotide that leads to no phenotypic consequence; a point mutation is considered ________ if it changes a nucleotide within a gene and causes the protein to be nonfunctional. Cellulose, intron, common, neutral, deleterious, somatic, gamete, unequal, homologous, zygote

Sexual reproduction in a multicellular organism involves specialized reproductive cells, called gametes, which come together to form a zygote that will divide to produce both reproductive and somatic cells. A point mutation in the DNA is considered a neutral mutation if it changes a nucleotide that leads to no phenotypic consequence; a point mutation is considered deleterious if it changes a nucleotide within a gene and causes the protein to be nonfunctional

Define signal transduction and list the basic components involved in this process in cells.

Signal transduction is the conversion of an impulse or stimulus from one physical or chemical form to another. The signals that pass between cells are simpler than the sorts of messages that humans exchange. In typical communication between cells, the signaling cell produces a particular type of extracellular signal molecule that is detected by the target cell. Target cells possess proteins called receptors that recognize and respond specifically to the signal molecule. Signal transduction begins when the receptor on a target cell receives an incoming extracellular signal and then produces intracellular signaling molecules that alter cell behavior.

Explain how mutant proteins can be used to determine the order in which proteins participate in a signaling pathway.

Such classical genetic screens can also help sort out the order in which intracellular signaling proteins act in a pathway. Suppose that a genetic screen uncovers a pair of new proteins, X and Y, involved in the Ras signaling pathway. To determine whether these proteins lie upstream or downstream of Ras, one could create cells that express an inactive, mutant form of each protein, and then ask whether these mutant cells can be "rescued" by the addition of a continuously active form of Ras. If the constantly active Ras overcomes the blockage created by the mutant protein, the protein must operate upstream of Ras in the pathway. However, if Ras operates upstream of the protein, a constantly active Ras would be unable to transmit a signal past the obstruction caused by the disabled protein.

Both the GTP-bound α subunits and nucleotide-free βγ complexes—but not GDP-bound, fully assembled G proteins—can activate other molecules downstream of GPCRs.

TRUE

Cancer can result from the accumulation of mutations in somatic cells.

TRUE

Different signals originating from the plasma membrane can be integrated by cross-talk between different signaling pathways inside the cell.

TRUE

IP3 is produced directly by cleavage of an inositol phospholipid without incorporation of an additional phosphate group.

TRUE

In the absence of DNA repair, genes become less stable.

TRUE

None of the aberrant bases formed by deamination occur naturally in DNA

TRUE

The error rate of DNA replication is reduced both by proofreading by DNA polymerase and by DNA mismatch repair.

TRUE

The extracellular signal molecule acetylcholine has different effects on different cell types in an animal and often binds to different cell-surface receptor molecules on different cell types

TRUE

State the amount of variation that typically distinguishes one human genome from another and describe the most common form of genetic variation.

Nucleotide sequences differ by 0.1% between people. Genetic variation was mostly inherited from early human ancestors.

Describe the structure of nucleotides, a DNA strand, and the DNA double helix.

Nucleotides- a DNA molecule consists of 2 complementary chains of nucleotides, each of which composed of 4 types of subunits. Phosphodiester bonds hold the nucleotide subunits together. Sequences of nucleotides code for RNA and proteins. DNA strands- composed of 2 long polynucleotide chains, held together by hydrogen bonds. Packaged with protein to form chromatin. Nucleotides inside double helix and phosphate outside.

Review how open reading frames are used in estimating gene number.

ORFs are long sequences missing stop codons that likely encode proteins. computer programs search for ORFs

Mutation is:

Permanent change in a DNA sequence

Nucleotides in each strand of DNA are held together by what type of bond?

Phosphodiester bonds

Recall the location and action of the second messenger molecules produced by activated phospholipase C.

Phospholipase C (membrane bound enzyme) when activated increases the second messenger diacylglycerol (DAG), inositol triphosphate, and Ca2+ GCPRs (some) effect G protein (Gq) that activates phospholipase C instead of adenylyl cyclase. · Inositol phospholipid - made by cleavage of a lipid molecule, present in small quantities in cytosolic leaflet of membrane bilayer · Cleavage of membrane to inositol phospholipid by phospholipase C generates 2 secondary messengers: inositol 1,4,5-triphopshate (IP3) and diacylglycerol (DAG) both important for signal relay · IP3 is water soluble sugar phosphate released into cytosol, it binds to and opens calcium channels in ER membrane. Calcium rushes out into cytosol, which signals other proteins · DAG is a lipid, remains in plasma membrane after produced by phospholipase C; it recruits and activates protein kinase (translocates from cytosol to plasma membrane) This enzyme is protein kinase C (PKC) because it needs to bind to calcium to become active and phosphorylate sets of proteins

Explain how multiple signaling pathways can integrate information to produce a coordinated cell response.

The most extensive links among the pathways are mediated by the protein kinases present in each. These kinases often phosphorylate, and hence regulate, components in other signaling pathways, in addition to components in their own pathway. Many intracellular signaling proteins have several potential phosphorylation sites, each of which can be phosphorylated by a different protein kinase. These proteins can thus act as integrating devices. Information received from different intracellular signaling pathways can converge on such proteins, which then convert a multicomponent input to a single outgoing signal. These integrating proteins, in turn, can deliver a signal to many downstream targets. In this way, the intracellular signaling system may act like a network of nerve cells in the brain—or like a collection of microprocessors in a computer— interpreting complex information and generating complex responses.

Contrast the arrested state G0 with the cell-cycle withdrawal that occurs during terminal differentiation.

The most radical decision that the cell-cycle control system can make is to withdraw the cell from the cell cycle permanently. This decision has a special importance in multicellular organisms. Many cells in the human body permanently stop dividing when they differentiate. In such terminally differentiated cells, such as nerve or muscle cells, the cell-cycle control system is dismantled completely and genes encoding the relevant cyclins and Cdks are irreversibly shut down. Other cell types withdraw from the cycle only temporarily, entering G0. They retain the ability to reassemble the cell-cycle control system quickly and to divide again.

Compare the point mutation rates of E. coli and humans and assess the significance of these relative values.

The mutation rate in humans is about 1/3 of that in E. coli, which suggests that our mechanisms ensuring genome integrity evolved at a rate similar to that of E. coli.

Summarize how and when the nuclear envelope breaks down.

The nuclear envelope disassembly begins in prometaphase. It breaks up into small membrane vesicles. This process is triggered by the phosphorylation and consequent disassembly of nuclear pore proteins and the intermediate filament proteins of the nuclear lamina, a network of fibrous proteins that underlie and stabilize the nuclear envelope

Explain how the genetic code specifies the relationship between the sequence of codons in mRNA and the amino acid sequence of a polypeptide.

The nucleotide sequence in mRNA is read in consecutive sets of three nucleotides called codons; each codon corresponds to one amino acid. The correspondence between amino acids and codons is specified by the genetic code. The possible combinations of the 4 different nucleotides in RNA give 64 different codons in the genetic code. Most amino acids are specified by more than one codon.

Describe how chromatin modifying proteins help regulate transcription.

The structure of chromatin (DNA and its organizing proteins) can be regulated. More open or "relaxed" chromatin makes a gene more available for transcription. Sets of transcription factor proteins bind to specific DNA sequences in or near a gene and promote or repress its transcription into an RNA.

Describe the trp operon and how it is under negative control.

The trp operon, found in E. coli bacteria, is a group of genes that encode biosynthetic enzymes for the amino acid tryptophan. The trp operon is expressed (turned "on") when tryptophan levels are low and repressed (turned "off") when they are high. Like the lac operon, the trp operon is a negative control mechanism. The trp operon responds to a repressor protein that binds to two molecules of tryptophan. When the tryptophan is plentiful, this repressor-tryptophan complex binds to the trp operator. This binding prevents the binding of RNA polymerase, so the operon is not transcribed.

Signaling via a GPCR ceases when which condition occurs?

The α subunit hydrolyzes its bound GTP.

What is true of the GTP-binding proteins that act as molecular switches inside cells?

They are active when GTP is bound

What is true of ion-channel-coupled receptors?

They transduce signals in a simple and direct manner.

Which would be the best method for determining which genes are being transcribed in a particular cell type?

RNA sequencing (RNA seq)

Express how changes in regulatory DNA sequences can contribute to the evolution of species.

Regulatory DNA sequences dictate each organism's developmental program: the rules its cells follow as they proliferate, assess their positions in the embryo, and specialize by switching on and off specific genes at the right time and place.

To reproduce sexually, an organism must create haploid ______ cells, or ______, from diploid cells via a specialized cell division called _______. During mating, the father's haploid cells, called ______ in animals, fuse with the mother's haploid cells, called ______. Cell fusion produces a diploid cell called a ________, which undergoes many rounds of cell division to create the entire body of the new individual. The cells produced from the initial fusion event include _______ cells that form most of the tissues of the body as well as the _______-line cells that give rise to the next generation of progeny. Allele, germ, pollen, bivalent, meiosis, somatic, eggs, mitosis, sperm, gametes, pedigree, zygote

To reproduce sexually, an organism must create haploid germ cells, or gametes, from diploid cells via a specialized cell division called meiosis. During mating, the father's haploid cells, called sperm in animals, fuse with the mother's haploid cells, called eggs. Cell fusion produces a diploid cell called a zygote, which undergoes many rounds of cell division to create the entire body of the new individual. The cells produced from the initial fusion event include somatic cells that form most of the tissues of the body as well as the germ-line cells that give rise to the next generation of progeny

Outline the steps of transcription, and the role of RNA polymerase in this process.

Transcription involves four steps: Initiation-DNA unwinds and separates, and RNA polymerase binds to the leading strand. Elongation-RNA polymerase moves along the template strand, synthesising an mRNA molecule Termination-additional adenine nucleotides are added at the 3' of the RNA transcript Processing- After transcription the RNA molecule is processed in a number of ways: introns are removed and the exons are spliced together to form a mature mRNA molecule consisting of a single protein-coding sequence

Many mobile genetic elements encode an enzyme that mediates their movement. What is this enzyme called?

Transposase

Compare and contrast monmeric and trimeric G-proteins

Trimeric G Proteins are made up of alpha, beta and gamma subunits; trimeric G-proteins can self GTP-hydrolyse whereas in monomeric G-proteins a GAP (GTPase Activating Protein) is required as it possesses weak intrinsic GTPase activity

Outline how apoptosis is regulated and initiated by the Bcl2 family of proteins.

Two of the most important death-inducing family members are proteins called Bak and Bax. These proteins - which are activated in response to DNA damage or other insults- promote cell death by inducing the release of the electron-transport protein cytochrome c from mitochondria into the cytosol. Other members of the Bcl2 family inhibit apoptosis by preventing Bax and Bak from releasing cytochrome c.

What is an operon?

a set of genes transcribed as a single mRNA from a single promoter

What is the contractile ring made of?

actin and myosin

Which response to GPCR stimulation would be most rapid?

activation of a G protein that regulates the opening of an ion channel

Compare and contrast small interfering RNA and micro RNA. Define gene duplication and divergence.

miRNA- bind to the 3'-UTR (untranslated region) of their target mRNAs and repress protein production by destabilizing the mRNA and translational silencing. siRNA- The complex silences certain gene expression by cleaving the mRNA molecules coding the target genes. This cleavage results in mRNA fragments that are further degraded by cellular exonucleases. gene duplication-the process by which a region of DNA coding for a gene is copied. Gene duplication can occur as the result of an error in recombination or through a retrotransposition event. Duplicate genes are often immune to the selective pressure under which genes normally exist. followed by divergence is a source of new genes in genomes. Divergence literally means to go in different directions. In this context, we mean that the sequences of the gene copies are becoming different from each other because of the accumulation of mutations.

What is the mitotic spindle made of?

microtubules

Almost half of the human genome consists of what?

mobile genetic elements

Each eukaryotic chromosome must contain the following DNA sequence elements:

multiple origins of replication, two telomeres, and one centromere.

When a cell is acutely injured, it usually dies by which process?

necrosis

Interphase chromosome:

nuclear divisions—that is, in interphase—the chromatin of the interphase chromosomes is in a relatively extended form in the nucleus.

In mammals, individuals with two X chromosomes are female, and individuals with an X and a Y chromosome are male. It had long been known that a gene located on the Y chromosome was sufficient to induce the gonads to form testes, which is the main male-determining factor in development, and researchers sought the product of this gene, the so-called testesdetermining factor (TDF). For several years, the TDF was incorrectly thought to be a zinc finger protein encoded by a gene called BoY. Which of the following observations would most strongly suggest that BoY might not be the TDF? Explain your answer. (a) Some XY individuals that develop into females have mutations in a different gene, SRY, but are normal at BoY. (b) BoY is not expressed in the adult male testes. (c) Expression of BoY in adult females does not masculinize them. (d) A few of the genes that are known to be expressed only in the testes have binding sites for the BoY protein in their upstream regulatory sequences, but most do not.

(a) Some XY individuals that develop into females have mutations in a different gene, SRY, but are normal at BoY

Which of the following statements about the Lac operon is false? (a) The Lac repressor binds when lactose is present in the cell. (b) Even when the CAP activator is bound to DNA, if lactose is not present, the Lac operon will not be transcribed. (c) The CAP activator can only bind DNA when it is bound to cAMP. (d) The Lac operon only produces RNA when lactose is present and glucose is absent.

(a) The Lac repressor binds when lactose is present in the cell

When Ras is activated, cells will divide. A dominant-negative form of Ras clings too tightly to GDP. You introduce a dominant-negative form of Ras into cells that also have a normal version of Ras. Which of the following statements is true? (a) The cells you create will divide less frequently than normal cells in response to the extracellular signals that typically activate Ras. (b) The cells you create will run out of the GTP necessary to activate Ras. (c) The cells you create will divide more frequently compared to normal cells in response to the extracellular signals that typically activate Ras. (d) The normal Ras in the cells you create will not be able to bind GDP because the dominant-negative Ras binds to GDP too tightly.

(a) The cells you create will divide less frequently than normal cells in response to the extracellular signals that typically activate Ras

You are studying a set of mouse genes whose expression increases when cells are exposed to the hormone cortisol, and you believe that the same cortisol responsive transcriptional activator regulates all of these genes. Which of the following statements below should be true if your hypothesis is correct? (a) The cortisol-responsive genes share a DNA sequence in their regulatory regions that binds the cortisol-responsive transcriptional activator. (b) The cortisol-responsive genes must all be in an operon. (c) The transcriptional regulators that bind to the regulatory regions of the cortisol-responsive genes must all be the same. (d) The cortisol-responsive genes must not be transcribed in response to other hormones.

(a) The cortisol-responsive genes share a DNA sequence in their regulatory regions that binds the cortisol-responsive transcriptional activator

Which of the following statements about secretion is true? (a) The membrane of a secretory vesicle will fuse with the plasma membrane when it discharges its contents to the cell's exterior. (b) Vesicles for regulated exocytosis will not bud off the trans Golgi network until the appropriate signal has been received by the cell. (c) The signal sequences of proteins destined for constitutive exocytosis ensure their packaging into the correct vesicles. (d) Proteins destined for constitutive exocytosis aggregate as a result of the acidic pH of the trans Golgi network.

(a) The membrane of a secretory vesicle will fuse with the plasma membrane when it discharges its contents to the cell's exterior

Which of the following does not occur before a eukaryotic mRNA is exported from the nucleus? (a) The ribosome binds to the mRNA. (b) The mRNA is polyadenylated at its 3' end. (c) 7-methylguanosine is added in a 5'-to-5' linkage to the mRNA. (d) RNA polymerase dissociates.

(a) The ribosome binds to the mRNA

Which of the following statements about transcriptional regulators is false? (a) Transcriptional regulators usually interact with the sugar-phosphate backbone on the outside of the double helix to determine where to bind on the DNA helix. (b) Transcriptional regulators will form hydrogen bonds, ionic bonds, and hydrophobic interactions with DNA. (c) The DNA-binding motifs of transcriptional regulators usually bind in the major groove of the DNA helix. (d) The binding of transcriptional regulators generally does not disrupt the hydrogen bonds that hold the double helix together.

(a) Transcriptional regulators usually interact with the sugar-phosphate backbone on the outside of the double helix to determine where to bind on the DNA helix

Both budding yeast and the bacteria E. coli are unicellular life. Which of the following statements explains why budding yeast can undergo sexual reproduction while E. coli cannot. (a) Unlike E. coli, budding yeast can alternate between a diploid state and a haploid state. (b) Unlike E. coli, budding yeast cannot multiply by undergoing cell division. (c) Unlike E. coli, haploid budding yeast cells can undergo meiosis to produce the gametes necessary for sexual reproduction. (d) E. coli DNA is unable to undergo homologous recombination, making it incapable of producing gametes.

(a) Unlike E. coli, budding yeast can alternate between a diploid state and a haploid state

During fertilization in humans, _________. (a) a wave of Ca2+ ions is released in the fertilized egg's cytoplasm. (b) only one sperm binds to the unfertilized egg. (c) a sperm moves in a random fashion until it encounters an egg. (d) several sperm pronuclei compete in the cytoplasm to fuse with the egg nucleus.

(a) a wave of Ca2+ ions is released in the fertilized egg's cytoplasm

What is cdc6 and what does it do? What enzymes does it effect?

Cell division cycle 6 (CDC6) is an essential regulator of DNA replication in eukaryotic cells. Its best-characterized function is the assembly of prereplicative complexes at origins of replication during the G1 phase of the cell division cycle.

You are working in a biotech company that has discovered a small-molecule drug called H5434. H5434 binds to LDL receptors when they are bound to cholesterol. H5434 binding does not alter the conformation of the LDL receptor's intracellular domain. Interestingly, in vitro experiments demonstrate that addition of H5434 increases the affinity of LDL for cholesterol and prevents cholesterol from dissociating from the LDL receptor even in acidic conditions. Which of the following is a reasonable prediction of what may happen when you add H5434 to cells? (a) Cytosolic cholesterol levels will remain unchanged relative to normal cells. (b) Cytosolic cholesterol levels will decrease relative to normal cells. (c) The LDL receptor will remain on the plasma membrane. (d) The uncoating of vesicles will not occur.

(b) Cytosolic cholesterol levels will decrease relative to normal cells

Which of the following statements about RNA splicing is false? (a) Conventional introns are not found in bacterial genes. (b) For a gene to function properly, every exon must be removed from the primary transcript in the same fashion on every mRNA molecule produced from the same gene. (c) Small RNA molecules in the nucleus perform the splicing reactions necessary for the removal of introns. (d) Splicing occurs after the 5' cap has been added to the end of the primary transcript.

(b) For a gene to function properly, every exon must be removed from the primary transcript in the same fashion on every mRNA molecule produced from the same gene

Which organelle fragments during mitosis? (a) endoplasmic reticulum (b) Golgi apparatus (c) mitochondrion (d) chloroplast

(b) Golgi apparatus

You discover a protein, MtA, and find that it binds to the plus ends of microtubules in cells. The hypothesis that best explains this localization is ________. (a) MtA is involved in stabilizing microtubules. (b) MtA binds to GTP-bound tubulin on microtubules. (c) MtA is important for the interaction of microtubules with the centrosome. (d) MtA will not bind to purified microtubules in a test tube.

(b) MtA binds to GTP-bound tubulin on microtubules

Which of the following statements is false? (a) Asexual reproduction typically gives rise to offspring that are genetically identical. (b) Mutations in somatic cells are passed on to individuals of the next generation. (c) Sexual reproduction allows for a wide variety of gene combinations. (d) Gametes are specialized sex cells.

(b) Mutations in somatic cells are passed on to individuals of the next generation

Which of the following statements about organellar movement in the cell is false? (a) Organelles undergo saltatory movement in the cell. (b) Only the microtubule cytoskeleton is involved in organellar movement. (c) Motor proteins involved in organellar movement use ATP hydrolysis for energy. (d) Organelles are attached to the tail domain of motor proteins.

(b) Only the microtubule cytoskeleton is involved in organellar movement

Which of the following statements is true? (a) MAP kinase is important for phosphorylating MAP kinase kinase. (b) PI 3-kinase phosphorylates a lipid in the plasma membrane. (c) Ras becomes activated when an RTK phosphorylates its bound GDP to create GTP. (d) STAT proteins phosphorylate JAK proteins, which then enter the nucleus and activate gene transcription.

(b) PI 3-kinase phosphorylates a lipid in the plasma membrane

There are several reasons why the primase used to make the RNA primer for DNA replication is not suitable for gene transcription. Which of the statements below is not one of those reasons? (a) Primase initiates RNA synthesis on a single-stranded DNA template. (b) Primase can initiate RNA synthesis without the need for a base-paired primer. (c) Primase synthesizes only RNAs of about 5-20 nucleotides in length. (d) The RNA synthesized by primase remains base-paired to the DNA template.

(b) Primase can initiate RNA synthesis without the need for a base-paired primer

Which of the following statements about the protein quality control system in the ER is false? (a) Chaperone proteins help misfolded proteins fold properly. (b) Proteins that are misfolded are degraded in the ER lumen. (c) Protein complexes are checked for proper assembly before they can exit the ER. (d) A chaperone protein will bind to a misfolded protein to retain it in the ER.

(b) Proteins that are misfolded are degraded in the ER lumen

Which of the following statements about the endoplasmic reticulum (ER) is false? (a) The ER is the major site for new membrane synthesis in the cell. (b) Proteins to be delivered to the ER lumen are synthesized on smooth ER. (c) Steroid hormones are synthesized on the smooth ER. (d) The ER membrane is contiguous with the outer nuclear membrane.

(b) Proteins to be delivered to the ER lumen are synthesized on smooth ER

You are studying a disease that is caused by a virus, but when you purify the virus particles and analyze them you find they contain no trace of DNA. Which of the following molecules are likely to contain the genetic information of the virus? (a) high-energy phosphate groups (b) RNA (c) lipids (d) carbohydrates

(b) RNA

Which of the following statements about RNAi is true? (a) The RNAi mechanism is found only in plants and animals. (b) RNAi is induced when double-stranded, foreign RNA is present in the cell. (c) RISC uses the siRNA duplex to locate complementary foreign RNA molecules. (d) siRNAs bind to miRNAs to induce RNAi.

(b) RNAi is induced when double-stranded, foreign RNA is present in the cell

Which of the following statements is true? (a) The signal sequences on mitochondrial proteins are usually at the Cterminus. (b) Most mitochondrial proteins are not imported from the cytosol but are synthesized inside the mitochondria. (c) Chaperone proteins in the mitochondria facilitate the movement of proteins across the outer and inner mitochondrial membranes. (d) Mitochondrial proteins cross the membrane in their native, folded state.

(c) Chaperone proteins in the mitochondria facilitate the movement of proteins across the outer and inner mitochondrial membranes

Which of the following statements about the cytoskeleton is false? (a) The cytoskeleton is made up of three types of protein filaments. (b) The cytoskeleton controls the location of organelles in eukaryotic cells. (c) Covalent bonds between protein monomers hold together cytoskeletal filaments. (d) The cytoskeleton of a cell can change in response to the environment.

(c) Covalent bonds between protein monomers hold together cytoskeletal filaments

The MyoD transcriptional regulator is normally found in differentiating muscle cells and participates in the transcription of genes that produce muscle-specific proteins, such as those needed in contractile tissue. Amazingly, expression of MyoD in fibroblasts causes these cells derived from skin connective tissue to produce proteins normally only seen in muscles. However, some other cell types do not transcribe muscle-specific genes when MyoD is expressed in them. Which of the following statements below is the best explanation of why MyoD can cause fibroblasts to express musclespecific genes? (a) Unlike some other cell types, fibroblasts have not lost the muscle-specific genes from their genome. (b) The muscle-specific genes must be in heterochromatin in fibroblasts. (c) During their developmental history, fibroblasts have accumulated some transcriptional regulators in common with differentiating muscle cells. (d) The presence of MyoD is sufficient to activate the transcription of musclespecific genes in all cell types.

(c) During their developmental history, fibroblasts have accumulated some transcriptional regulators in common with differentiating muscle cells

You engineer yeast cells that express the M cyclin during S phase by replacing the promoter sequence of the M cyclin gene with that of S cyclin. Keeping in mind that yeast cells have one common Cdk that binds to all cyclins, which of the following outcomes is least likely during this experiment? (a) There will be both M cyclin-Cdk and S cyclin-Cdk complexes in the cell during S phase. (b) Some substrates that are normally phosphorylated in M phase will now be phosphorylated in S phase. (c) G1 cyclins will be expressed during S phase. (d) S-Cdk targets will be phosphorylated during S phase.

(c) G1 cyclins will be expressed during S phase

Which of the following statements about nucleosomes is true? (a) Nucleosomes activate transcription when bound to the promoter. (b) Although RNA polymerase can access DNA packed within nucleosomes, the general transcription factors and transcriptional regulators cannot. (c) Histone acetyltransferases affect transcription by both altering chromatin structure to allow accessibility to the DNA and by adding acetyl groups to histones that can bind proteins that promote transcription. (d) Histone deacetylases remove lysines from histone tails.

(c) Histone acetyltransferases affect transcription by both altering chromatin structure to allow accessibility to the DNA and by adding acetyl groups to histones that can bind proteins that promote transcription

How does S-Cdk help guarantee that replication occurs only once during each cell cycle? (a) It blocks the rise of Cdc6 concentrations early in G1. (b) It phosphorylates and inactivates DNA helicase. (c) It phosphorylates the Cdc6 protein, marking it for destruction. (d) It promotes the assembly of a prereplicative complex.

(c) It phosphorylates the Cdc6 protein, marking it for destruction

The yeast genome was sequenced more than 15 years ago, yet the total number of genes continues to be refined. The sequencing of closely related yeast species was important for validating the identity of short (less than 100 nucleotides long) open reading frames (ORFs) that were otherwise difficult to predict. What is the main reason that these short ORFs are hard to find without the genomes of other yeast for comparison? (a) Short ORFs are found only in yeast. (b) The short ORFs code for RNAs. (c) Many short stretches of DNA may lack a stop codon simply by chance, making it difficult to distinguish those DNA sequences that code for proteins from those that do not. (d) Short ORFs occur mainly in gene-rich regions, making them difficult to identify by computer programs.

(c) Many short stretches of DNA may lack a stop codon simply by chance, making it difficult to distinguish those DNA sequences that code for proteins from those that do not

Alternative exons can arise through the duplication and divergence of existing exons. What type of mutation below would be least tolerated during the evolution of a new exon? (a) a nucleotide change of A to G (b) a deletion of three consecutive bases (c) mutation of the first nucleotide in the intron (d) a nucleotide change that alters a TT dinucleotide to AA

(c) mutation of the first nucleotide in the intron

Your friend works in a biotechnology company and has discovered a drug that blocks the ability of Ran to exchange GDP for GTP. What is the most likely effect of this drug on nuclear transport? (a) Nuclear transport receptors would be unable to bind cargo. (b) Nuclear transport receptors would be unable to enter the nucleus. (c) Nuclear transport receptors would be unable to release their cargo in the nucleus. (d) Nuclear transport receptors would interact irreversibly with the nuclear pore fibrils.

(c) Nuclear transport receptors would be unable to release their cargo in the nucleus

Which of the following statements is true? (a) Proteins destined for the ER are translated by a special pool of ribosomes whose subunits are always associated with the outer ER membrane. (b) Proteins destined for the ER translocate their associated mRNAs into the ER lumen where they are translated. (c) Proteins destined for the ER are translated by cytosolic ribosomes and are targeted to the ER when a signal sequence emerges during translation. (d) Proteins destined for the ER are translated by a pool of cytosolic ribosomes that contain ER-targeting sequences that interact with ER associated protein translocators.

(c) Proteins destined for the ER are translated by cytosolic ribosomes and are targeted to the ER when a signal sequence emerges during translation

Which of the following statements about vesicular membrane fusion is false? (a) Membrane fusion does not always immediately follow vesicle docking. (b) The hydrophilic surfaces of membranes have water molecules associated with them that must be displaced before vesicle fusion can occur. (c) The GTP hydrolysis of the Rab proteins provides the energy for membrane fusion. (d) The interactions of the v-SNAREs and the t-SNAREs pull the vesicle membrane and the target organelle membrane together so that their lipids can intermix.

(c) The GTP hydrolysis of the Rab proteins provides the energy for membrane fusion

The ribosome is important for catalyzing the formation of peptide bonds. Which of the following statements is true? (a) The number of rRNA molecules that make up a ribosome greatly exceeds the number of protein molecules found in the ribosome. (b) The large subunit of the ribosome is important for binding to the mRNA. (c) The catalytic site for peptide bond formation is formed primarily from an rRNA. (d) Once the large and small subunits of the ribosome assemble, they will not separate from each other until degraded by the proteasome.

(c) The catalytic site for peptide bond formation is formed primarily from an rRNA

Which of the following descriptions is consistent with the behavior of a cell that lacks a protein required for a checkpoint mechanism that operates in G2? (a) The cell would be unable to enter M phase. (b) The cell would be unable to enter G2. (c) The cell would enter M phase under conditions when normal cells would not. (d) The cell would pass through M phase more slowly than normal cells

(c) The cell would enter M phase under conditions when normal cells would not

List the three fundamental cell processes that determine the size of an an animal's organs or body.

Cell growth, cell division, and cell death

Your friend works in a lab that is studying why a particular mutant strain of Drosophila grows an eye on its wing. Your friend discovers that this mutant strain of Drosophila is expressing a transcription factor incorrectly. In the mutant Drosophila, this transcription factor, which is normally expressed in the primordial eye tissue, is now misexpressed in the primordial wing tissue, thus turning on transcription of the set of genes required to produce an eye in the wing primordial tissue. If this hypothesis is true, which of the following types of genetic change would most likely lead to this situation? (a) a mutation within the transcription factor gene that leads to a premature stop codon after the third amino acid (b) a mutation within the transcription factor gene that leads to a substitution of a positively charged amino acid for a negatively charged amino acid (c) a mutation within an upstream enhancer of the gene (d) a mutation in the TATA box of the gene

(c) a mutation within an upstream enhancer of the gene

Most proteins destined to enter the endoplasmic reticulum _________. (a) are transported across the membrane after their synthesis is complete. (b) are synthesized on free ribosomes in the cytosol. (c) begin to cross the membrane while still being synthesized. (d) remain within the endoplasmic reticulum.

(c) begin to cross the membrane while still being synthesized

The human genome encodes about 21,000 protein-coding genes. Approximately how many such genes does the typical differentiated human cell express at any one time? (a) 21,000—all of them (b) between 18,900 and 21,000—at least 90% of the genes (c) between 5000 and 15,000 (d) less than 2100

(c) between 5000 and 15,000

Apoptosis differs from necrosis in that necrosis _______. (a) requires the reception of an extracellular signal. (b) causes DNA to fragment. (c) causes cells to swell and burst, whereas apoptotic cells shrink and condense. (d) involves a caspase cascade.

(c) causes cells to swell and burst, whereas apoptotic cells shrink and condense

At the end of DNA replication, the sister chromatids are held together by the _______. (a) kinetochores. (b) securins. (c) cohesins. (d) histones.

(c) cohesins

Operons ___________________________. (a) are commonly found in eukaryotic cells. (b) are transcribed by RNA polymerase II. (c) contain a cluster of genes transcribed as a single mRNA. (d) can only be regulated by gene activator proteins.

(c) contain a cluster of genes transcribed as a single mRNA

Your friend has just joined a lab that studies vesicle budding from the Golgi and has been given a cell line that does not form mature vesicles. He wants to start designing some experiments but wasn't listening carefully when he was told about the molecular defect of this cell line. He's too embarrassed to ask and comes to you for help. He does recall that this cell line forms coated pits but vesicle budding and the removal of coat proteins don't happen. Which of the following proteins might be lacking in this cell line? (a) clathrin (b) Rab (c) dynamin (d) adaptin

(c) dynamin

Signal sequences that direct proteins to the correct compartment are _________. (a) added to proteins through post-translational modification. (b) added to a protein by a protein translocator. (c) encoded in the amino acid sequence and sufficient for targeting a protein to its correct destination. (d) always removed once a protein is at the correct destination.

(c) encoded in the amino acid sequence and sufficient for targeting a protein to its correct destination

RNA in cells differs from DNA in that ___________________. (a) it contains the base uracil, which pairs with cytosine. (b) it is single-stranded and cannot form base pairs. (c) it is single-stranded and can fold up into a variety of structures. (d) the sugar ribose contains fewer oxygen atoms than does deoxyribose.

(c) it is single-stranded and can fold up into a variety of structures.

Somatic cells _______. (a) are not necessary for sexual reproduction in all eukaryotic organisms. (b) are used to produce germ-line cells when organisms reach sexual maturity (c) leave no progeny. (d) do not contain sex chromosomes.

(c) leave no progeny

Which of the following statements about miRNAs is false? (a) One miRNA can regulate the expression of many genes. (b) miRNAs are transcribed in the nucleus from genomic DNA. (c) miRNAs are produced from rRNAs. (d) miRNAs are made by RNA polymerase.

(c) miRNAs are produced from rRNAs

Which of the following organelles is not part of the endomembrane system? (a) Golgi apparatus (b) the nucleus (c) mitochondria (d) lysosomes

(c) mitochondria

The concentration of a particular protein, X, in a normal human cell rises gradually from a low point, immediately after cell division, to a high point, just before cell division, and then drops sharply. The level of its mRNA in the cell remains fairly constant throughout this time. Protein X is required for cell growth and survival, but the drop in its level just before cell division is essential for division to proceed. You have isolated a line of human cells that grow in size in culture but cannot divide, and on analyzing these mutants, you find that levels of X mRNA in the mutant cells are normal. Which of the following mutations in the gene for X could explain these results? (a) the introduction of a stop codon that truncates protein X at the fourth amino acid (b) a change of the first ATG codon to CCA (c) the deletion of a sequence that encodes sites at which ubiquitin can be attached to the protein (d) a change at a splice site that prevents splicing of the RNA

(c) the deletion of a sequence that encodes sites at which ubiquitin can be attached to the protein

A neuron and a white blood cell have very different functions. For example, a neuron can receive and respond to electrical signals while a white blood cell defends the body against infection. This is because ______. (a) the proteins found in a neuron are completely different from the proteins found in a white blood cell. (b) the neuron and the white blood cell within an individual have the same genome. (c) the neuron expresses some mRNAs that the white blood cell does not. (d) neurons and white blood cells are differentiated cells and thus no longer need to transcribe and translate genes.

(c) the neuron expresses some mRNAs that the white blood cell does not

Review how calcium-responsive proteins such as calmodulin propagate a calcium ion signal.

Calmodulin - present in cytosol of all euks; calcium binds to it, protein confirmation changes, it interacts with many target proteins that alters their activities · Ca/Calmodulin dependent protein kinases (CaM-Kinases) when kinase is activated by binding of calmodulin w Ca, they phosphorylate selected proteins Mammalian brains: neuron specific CaM-kinase is abundant at synapses, important part in forms of learning and memory CaM-Kinase activated by pulses of Ca signals that occur during neural activity Mutant mice that lack this kinase have limited ability to remember Structure: calmodulin has dumbbell shape, with 2 globular ends connected by long alpha helix. Each globular end has 2 Ca binding sites (4 total) one on NH2 & one on COOH

What are the similarities and differences between the reactions that lead to the activation of G proteins and the reactions that lead to the activation of Ras?

Activation in both cases depends on proteins that catalyze GDP-GTP exchange on the G protein or Ras protein. Whereas activated GPCRs perform this function directly for G proteins, enzyme-linked receptors assemble multiple signaling proteins into a signaling complex when the receptors are activated by phosphorylation; one of these proteins is an adaptor protein that recruits a guanine nucleotide exchange factor that fulfills this function for Ras.

Name the classes of enzymes that are the most frequent targets of G proteins, and list the second messenger molecules they produce.

Adenylyl cyclase - cyclic AMP Phospholipase C - inositol triphosphate (promotes cytosolic Ca) & diacylglycerol

List several biological processes triggered by calcium ions.

After fertilization, rise in cytosolic calcium triggers egg to begin development. Muscle cells, signals from nerves cause rise in Ca initiating muscle contraction. Secretory cells (include nerve cells) Ca triggers secretion

Compare how Akt promotes cell survival via Bad and stimulates cell growth via Tor.

Akt promotes cell survival: It phosphorylates and inactivates a protein called Bad. In its unphosphorylated state, Bad promotes apoptosis (form of cell death) by binding to and inhibiting a protein called Bcl2, which otherwise suppresses apoptosis. When Bad is phosphorylated by Akt, Bad releases Bcl2, which now blocks apoptosis, thereby promoting cell survival. The binding of a growth factor to an RTK activates the PI 3-kinase-Akt signaling pathway. Akt then indirectly activates Tor by phosphorylating and inhibiting a protein that helps to keep Tor shut down. Tor stimulates protein synthesis and inhibits protein degradation by phosphorylating key proteins in these processes.

An extracellular signal molecule can act to change a cell's behavior by acting through cell-surface ________ that control intracellular signaling proteins. These intracellular signaling proteins ultimately change the activity of _______ proteins that bring about cell responses. Intracellular signaling proteins can _______ the signal received to evoke a strong response from just a few extracellular signal molecules. A cell that receives more than one extracellular signal at the same time can _______ this information using intracellular signaling proteins. ______ proteins can act as molecular switches, letting a cell know that a signal has been received. Enzymes that phosphorylate proteins, termed _______, can also serve as molecular switches; the actions of these enzymes are countered by the activity of _______. Acetylase, decouple, GTP-binding, AMP-binding, decrease, neurotransmitter, amplify, effector, protein kinases, autocrine, esterases, protein phosphatases, cleavage, integrate, receptors, convolute, GMP-binding, sterols

An extracellular signal molecule can act to change a cell's behavior by acting through cell-surface receptors that control intracellular signaling proteins. These intracellular signaling proteins ultimately change the activity of effector proteins that bring about cell responses. Intracellular signaling proteins can amplify the signal received to evoke a strong response from just a few extracellular signal molecules. A cell that receives more than one extracellular signal at the same time can integrate this information using intracellular signaling proteins. GTP-binding proteins can act as molecular switches, letting a cell know that a signal has been received. Enzymes that phosphorylate proteins, termed protein kinases, can also serve as molecular switches; the actions of these enzymes are countered by the activity of protein phosphatases.

Explain cell memory and the positive feedback loop.

At its simplest, cellular memory is achieved with a positive feedback loop--once activated by some external signal, the feedback loop will continually activate itself, even as the cell divides and the signal is taken away.

Outline the process of cytokinesis in plant cells.

At the beginning of telophase, after the chromosomes have segregated, a new cell wall starts to assemble inside the cell at the equator of the old spindle. The interpolar microtubules of the mitotic spindle remaining at telophase form the phragmoplast and guide vesicles, derived from the Golgi apparatus, toward the equator of the spindle. The vesicles which are filled with cell wall material, fuse to form the growing new cell wall that grows outward to reach the plasma membrane and original cell wall. The preexisting plasma membrane and the membrane surrounding the new cell wall then fuse, completely separating the two daughter cells.

Compare and contrast the structure of eukaryotic and prokaryotic chromosomes.

Bacteria carries genes on single circular DNA, Eukaryotic DNA is packaged into multiple chromosomes

Explain how extracts prepared from cells in different phases of the cell cycle have been used to identify components of the cell-cycle control system.

Because of synchrony (cells growing smaller and smaller with each division), it is possible to prepare an extract from frog eggs that is representative of the cell-cycle stage at which the extract is made. They found that an extract from an M-phase egg instantly drives the oocyte into M phase (MPF-maturation promoting factor), whereas cytoplasm from a cleaving egg at other phases of the cycle does not.

What are the three processes in RNA processing?

Capping, Addition of Poly A tail, and removal of introns (splicing) RNA processing is the term collectively used to describe the sequence of events through which the primary transcript from a gene acquires its mature form. Very soon after synthesis by RNA polymerase II begins, transcripts from nuclear protein-coding genes acquire a 5′ cap structure. The 3′ end of the messenger RNA (mRNA) is modified by the addition of a long string of adenosines in a process tightly linked to transcription termination. Finally, maturation of most eukaryotic mRNA precursors requires a process known as splicing, in which internal noncoding segments known as introns are removed and the coding segments, known as exons, are joined to produce functional mRNAs.

What is the function of mRNA?

Carry instruction for making proteins

Before mitosis, the number of centrosomes must [increase/decrease]. At the beginning of [anaphase/prophase] in animal cells, the centrosomes separate in a process driven partly by interactions between the [plus/minus] ends of microtubules arising from the two centrosomes. Centrosome separation initiates the assembly of the bipolar mitotic spindle and is associated with a sudden [increase/decrease] in the dynamic instability of microtubules. In comparison with an interphase microtubule array, a mitotic aster contains a [smaller/larger] number of [longer/shorter] microtubules. Extracts from M-phase cells exhibit [increased/decreased/unchanged] rates of microtubule polymerization and increased frequencies of microtubule [shrinkage/growth]. The changes in microtubule dynamics are largely due to [enhanced/reduced] activity of microtubule-associated proteins and [increased/decreased] activity of catastrophins. The new balance between polymerization and depolymerization of microtubules is necessary for the mitotic spindle to move the [replicated chromosomes/daughter chromosomes] to the metaphase plate.

Before mitosis, the number of centrosomes must increase. At the beginning of prophase in animal cells, the centrosomes separate in a process driven partly by interactions between the plus ends of microtubules arising from the two centrosomes. Centrosome separation initiates the assembly of the bipolar mitotic spindle and is associated with a sudden increase in the dynamic instability of microtubules. In comparison with an interphase microtubule array, a mitotic aster contains a larger number of shorter microtubules. Extracts from M-phase cells exhibit unchanged rates of microtubule polymerization and increased frequencies of microtubule shrinkage. The changes in microtubule dynamics are largely due to reduced activity of microtubule-associated proteins and increased activity of catastrophins. The new balance between polymerization and depolymerization of microtubules is necessary for the mitotic spindle to move the replicated chromosomes to the metaphase plate.

Outline the structural features of bacterial and eukaryotic ribosomes.

Both are composed of one large subunit and one small subunit, which fit together to form a complete ribosome. The small ribosomal subunit matches the tRNAs to the codons of the mRNA, while the large subunit catalyzes the formation of the peptide bonds that covalently link the amino acids together into a polypeptide chain. These two subunits come together on an mRNA molecule near its 5ʹ end to start the synthesis of a protein. The mRNA is then pulled through the ribosome like a long piece of tape. As the mRNA inches forward in a 5ʹ-to-3ʹ direction, the ribosome translates its nucleotide sequence into an amino acid sequence, one codon at a time, using the tRNAs as adaptors.

Compare and contrast signaling by neurons, which secrete neurotransmitters at synapses, with signaling carried out by endocrine cells, which secrete hormones into the blood; discuss the relative advantages of the two mechanisms.

Both types of signaling can occur over a long range: neurons can send action potentials along very long axons (think of the axons in the neck of a giraffe, for example), and hormones are carried via the bloodstream throughout the organism. Because neurons secrete large amounts of neurotransmitters at a synapse, a small, well-defined space between two cells, the concentrations of these signal molecules are high; neurotransmitter receptors, therefore, need to bind to neurotransmitters with only low affinity. Hormones, in contrast, are vastly diluted in the bloodstream, where they circulate at often minuscule concentrations; hormone receptors therefore generally bind their hormone with extremely high affinity. Whereas neuronal signaling is a private affair, with one neuron talking to a select group of target cells through specific synaptic connections, endocrine signaling is a public announcement, with any target cell with appropriate receptors able to respond to the hormone in the blood. Neuronal signaling is very fast, limited only by the speed of propagation of the action potential and the workings of the synapse, whereas endocrine signaling is slower, limited by blood flow and diffusion over larger distances

Assess how retroviruses and retrotransposons are similar.

Both use the reverse transcriptase enzyme to convert RNA to DNA.

Review how cyclin-Cdk complexes are inhibited as cells enter G1.

By eliminating all the existing cyclins, by blocking the synthesis of new ones, and by deploying Cdk inhibitor proteins to muffle the activity of any remaining cyclin-Cdk complexes. The use of multiple mechanisms makes this system of suppression robust, ensuring that essentially all Cdk activity is shut down.

Ca2+ can trigger biological effects in cells because an unstimulated cell has an extremely ______ concentration of free Ca2+ in the cytosol, compared with its concentration in the ________ space and in the ______, creating a steep electrochemical gradient. When Ca2+ enters the cytosol, it interacts with Ca2+-responsive proteins such as ______, which also binds diacylglycerol, and ______, which activates CaM-kinases. adenylyl cyclase, endoplasmic reticulum, nuclear, average, extracellular, peroxisome, Ca2+, high, phospholipase C, calmodulin, intracellular, protein kinase A, colorful, low, protein kinase C

Ca2+ can trigger biological effects in cells because an unstimulated cell has an extremely low concentration of free Ca2+ in the cytosol, compared with its concentration in the extracellular space and in the endoplasmic reticulum, creating a steep electrochemical gradient. When Ca2+ enters the cytosol, it interacts with Ca2+-responsive proteins such as protein kinase C, which also binds diacylglycerol, and calmodulin, which activates CaM-kinases

Summarize how apoptosis is mediated by a proteolytic caspase cascade.

Caspases are made as inactive precursors, called procaspases, which are activated in response to signals that induce apoptosis. Two types of caspases work together to take a cell apart. Initiator caspases cleave, and thereby activate downstream executioner caspases, which dismember numerous key proteins in the cell.

Before chromosomes segregate in M phase, they and the segregation machinery must be appropriately prepared. Indicate whether the following statements are true or false. If false, change a single noun to make the statement true. A. Sister chromatids are held together by condensins from the time they arise by DNA replication until the time they separate at anaphase. B. Cohesins are required to make the chromosomes more compact and thus to prevent tangling between different chromosomes. C. The mitotic spindle is composed of actin filaments and myosin filaments. D. Microtubule-dependent motor proteins and microtubule polymerization and depolymerization are mainly responsible for the organized movements of chromosomes during mitosis. E. The centromere nucleates a radial array of microtubules called an aster, and its duplication is triggered by S-Cdk. F. Each centrosome contains a pair of centrioles and hundreds of γ-tubulin rings that nucleate the growth of microtubules.

False, False, False, True, False, True

After acetylcholine is secreted from cells, it is long-lived, because it has to reach target cells all over the body.

False. Acetylcholine is short-lived and exerts its effects locally. Indeed, the consequences of prolonging its lifetime can be disastrous. Compounds that inhibit the enzyme acetylcholinesterase, which normally breaks down acetylcholine at a nerve-muscle synapse, are extremely toxic: for example, the nerve gas sarin, used in chemical warfare, is an acetylcholinesterase inhibitor.

A bacterial replication fork is asymmetrical because it contains two DNA polymerase molecules that are structurally distinct.

False. Identical DNA polymerase molecules catalyze DNA synthesis on the leading and lagging strands of a bacterial replication fork. The replication fork is asymmetrical because the lagging strand is made in pieces while the leading strand is synthesized continuously.

Okazaki fragments are removed by a nuclease that degrades RNA

False. Okazaki fragments initially contain both RNA primers and DNA, but only the RNA primers are removed by RNA nucleases.

Compare a signaling pathway in which cyclic AMP produces a response within seconds to one in which the response takes minutes or hours to develop.

Fight or flight - adrenal gland secrets epinephrine, binds to GPCRs called adrenergic receptors to prepare for sudden action · Skeletal muscle, epinephrine increases cAMP to breakdown glycogen by activating PKA (cyclic AMP dependent protein kinase), this active PKA then phosphorylates and activates enzyme [phosphorylase kinase]. This kinase then activates glycogen phosphorylase, an enzyme that breaks down glycogen. These reactions happen quickly because they do not require change in gene transcription or new protein synthesis. cAMP responses that involve changes in gene expression or protein synthesis take minutes or hours · PKA phosphorylates transcription regulators

Outline the main classes of extracellular signal molecules, and describe the types of receptors to which they bind.

First and largest class of signals consists of molecules too large or too hydrophilic to cross the plasma membrane of the target cell. These signals rely on receptors on the surface of the target cell to relay their message across the plasma membrane. The second class of signals consists of molecules that are small enough or hydrophobic enough to pass through the plasma membrane and into the cytosol of the target cell.

P27 and P21, What are they and what is their function?

Following anti-mitogenic signals or DNA damage, p21 and p27 bind to cyclin-CDK complexes to inhibit their catalytic activity and induce cell-cycle arrest. P21 is a cyclin-dependent kinase inhibitor (CKI) that is capable of inhibiting all cyclin/CDK complexes, though is primarily associated with inhibition of CDK2.

For a cell's genetic material to be used, the information is first copied from the DNA into the nucleotide sequence of RNA in a process called __________________. Various kinds of RNA are produced, each with different functions. __________________ molecules code for proteins, __________________ molecules act as adaptors for protein synthesis, __________________ molecules are integral components of the ribosome, and __________________ molecules are important in the splicing of RNA transcripts. Incorporation, rRNA, translation, mRNA, snRNA, transmembrane, pRNA, transcription, tRNA, proteins

For a cell's genetic material to be used, the information is first copied from the DNA into the nucleotide sequence of RNA in a process called transcription. Various kinds of RNA are produced, each with different functions. mRNA molecules code for proteins, tRNA molecules act as adaptors for protein synthesis, rRNA molecules are integral components of the ribosome, and snRNA molecules are important in the splicing of RNA transcripts.

In eukaryotic cells, general transcription factors are required for the activity of all promoters transcribed by RNA polymerase II. The assembly of the general transcription factors begins with the binding of the factor __________________ to DNA, causing a marked local distortion in the DNA. This factor binds at the DNA sequence called the __________________ box, which is typically located 25 nucleotides upstream from the transcription start site. Once RNA polymerase II has been brought to the promoter DNA, it must be released to begin making transcripts. This release process is facilitated by the addition of phosphate groups to the tail of RNA polymerase by the factor __________________. It must be remembered that the general transcription factors and RNA polymerase are not sufficient to initiate transcription in the cell and are affected by proteins bound thousands of nucleotides away from the promoter. Proteins that link the distantly bound transcription regulators to RNA polymerase and the general transcription factors include the large complex of proteins called the__________________. The packing of DNA into chromatin also affects transcriptional initiation, and histone __________________ is an enzyme that can render the DNA less accessible to the general transcription factors. Activator, lac, TFIIA, CAP, ligase, TFIID, deacetylase, Mediator, TFIIH, enhancer, TATA

In eukaryotic cells, general transcription factors are required for the activity of all promoters transcribed by RNA polymerase II. The assembly of the general transcription factors begins with the binding of the factor TFIID to DNA, causing a marked local distortion in the DNA. This factor binds at the DNA sequence called the TATA box, which is typically located 25 nucleotides upstream from the transcription start site. Once RNA polymerase II has been brought to the promoter DNA, it must be released to begin making transcripts. This release process is facilitated by the addition of phosphate groups to the tail of RNA polymerase by the factor TFIIH. It must be remembered that the general transcription factors and RNA polymerase are not sufficient to initiate transcription in the cell and are affected by proteins bound thousands of nucleotides away from the promoter. Proteins that link the distantly bound transcription regulators to RNA polymerase and the general transcription factors include the large complex of proteins called the Mediator. The packing of DNA into chromatin also affects transcriptional initiation, and histone deacetylase is an enzyme that can render the DNA less accessible to the general transcription factors

Compare and contrast RAN and RAS.

In general, the Ras family is responsible for cell proliferation: Rho for cell morphology, Ran for nuclear transport, and Rab and Arf for vesicle transport. Ras proteins function as binary molecular switches that control intracellular signaling networks. Ras-regulated signal pathways control such processes as actin cytoskeletal integrity, cell proliferation, cell differentiation, cell adhesion, apoptosis, and cell migration. Ran is a small 25 kDa protein that is involved in transport into and out of the cell nucleus during interphase and also involved in mitosis. It is a member of the Ras superfamily. Ran is a small G protein that is essential for the translocation of RNA and proteins through the nuclear pore complex.

When compared to each other, the two replication forks that form at an origin of replication move in which direction?

In opposite direction

Contrast the fate of mutations that arise in somatic cells versus germ-line cells.

In sexually reproducing organisms, genetic information is only passed onto the next generation by germ-line cells. somatic cells form the organism's body, but aren't passed on.

What effect can viruses have on the genome? Consider a new to science or novel virus, like Covid-19. What is the potential for covid-19 patients, given this knowledge?

In some cases, scientists are finding, it actually has a beneficial impact. When viruses infect us, they can embed small chunks of their genetic material in our DNA. Although infrequent, the incorporation of this material into the human genome has been occurring for millions of years.

Outline how RTKs activate the MAP kinase signaling module.

In this pathway, MAP kinase is phosphorylated and activated by an enzyme called MAP kinase kinase. This protein is itself switched on by a MAP kinase kinase kinase (which is activated by Ras). At the end of the MAP-kinase cascade, MAP kinase phosphorylates various effector proteins, including certain transcription regulators, altering their ability to control gene transcription. The resulting change in the pattern of gene expression may stimulate cell proliferation, promote cell survival, or induce cell differentiation.

Review how the Notch receptor activates target genes in response to activation by Delta, and explain how this pathway 15. Outline how steroid hormones trigger the transcription of different sets of target genes.

In this simple signaling pathway, the receptor itself acts as a transcription regulator. When activated by the binding of Delta, a transmembrane signal protein on the surface of a neighboring cell, the Notch receptor is cleaved. This cleavage releases the cytosolic tail of the receptor, which is then free to move the nucleus, where it helps to activate the appropriate set of Notch-responsive genes. When a hormone (steroid) binds, the nuclear receptor undergoes a large conformational change that activates the protein, allowing it to promote or inhibit the transcription of specific target genes.

Explain how Rb blocks cell proliferation and how mitogens reverse this inhibition.

Rb is abundant in the nuclei of all vertebrate cells, where it binds to particular transcription regulators and prevents them from turning on the genes required for cell proliferation. Mitogens release the Rb brake by triggering the activation of G1-Cdks and G1/S-Cdks. These complexes phosphorylate the Rb protein, altering its conformation so that it releases its bound transcription regulators, which are then free to activate the genes required for entry into S phase.

Intermediate filaments are found mainly in cells that are subject to mechanical stress. Gene mutations that disrupt intermediate filaments cause some rare human genetic diseases. For example, the skin of people with epidermolysis bullosa simplex is very susceptible to mechanical injury; people with this disorder have mutations in their ______ genes, which code for the intermediate filament found in epithelial cells. These filaments are usually connected from cell to cell through junctions called ______s. The main filaments found in muscle cells belong to the _____ family; people with disruptions in these intermediate filaments can have muscular dystrophy. In the nervous system, ______s help strengthen the extremely long extensions often present in nerve cell axons; disruptions in these intermediate filaments can lead to neurodegeneration. People who carry mutations in the gene for _____, an important protein for cross-linking intermediate filaments, have a disease that combines symptoms of epidermolysis bullosa simplex, muscular dystrophy, and neurodegeneration. Humans with progeria, a disease that causes premature aging, carry mutations in a nuclear _______. Desmosome, lamin, synapse, keratin, neurofilament, vimentin, kinase, plectin

Intermediate filaments are found mainly in cells that are subject to mechanical stress. Gene mutations that disrupt intermediate filaments cause some rare human genetic diseases. For example, the skin of people with epidermolysis bullosa simplex is very susceptible to mechanical injury; people with this disorder have mutations in their keratin genes, the intermediate filament found in epithelial cells. These filaments are usually connected from cell to cell through junctions called desmosomes. The main filaments found in muscle cells belong to the vimentin family; people with disruptions in these intermediate filaments can have muscular dystrophy. In the nervous system, neurofilaments help strengthen the extremely long extensions often present in nerve cell axons; disruptions in these intermediate filaments can lead to neurodegeneration. People who carry mutations in the gene for plectin, an important protein for cross-linking intermediate filaments, have a disease that combines symptoms of epidermolysis bullosa simplex, muscular dystrophy, and neurodegeneration. Humans with progeria, a disease that causes premature aging, carry mutations in a nuclear lamin.

Differentiate the three main classes of cell-surface receptors and provide an example of each.

Ion Channel Coupled Receptors - changes permeability of plasma membrane f or specific ions, alters membrane potential, produces electrical current in proper conditions · Acetylcholine increases contractions in skeletal muscle cell via ICCR GPCRs - activate membrane bound trimeric GTP binding proteins, this activates or inhibits enzymes or ion channels in plasma membrane, begins an intracellular signaling cascade · Acetylcholine reduces contractions in heart pacemaker cells via GPCR Enzyme Coupled Receptors - act as enzymes or associate with enzymes, stimulated enzymes activate variety of signaling pathways

Explain RNA world and autocatalysis.

It has been proposed that RNA served as both the genome and the catalysts in the first cells, before DNA replaced RNA as a more stable molecule for storing genetic information, and proteins replaced RNAs as the major catalytic and structural components. RNA catalysts in modern cells are thought to provide a glimpse into an ancient, RNAbased world.

Review the contractile ring in terms of its composition, assembly, and mechanism of action.

It is composed mainly of an overlapping array of actin and myosin filaments. It assembles at anaphase and is attached to membrane-associated proteins on the cytosolic face of the plasma membrane. Once assembled, the contractile ring can exert a force strong enough to bend a fine glass needle inserted into the cell before cytokinesis.

List the four phases of the eukaryotic cell cycle and summarize what takes place in each.

M phase- seen in a microscope, the nucleus divides, a process called mitosis, and then when the cell itself splits into two, a process called cytokinesis. In a mammalian cell, the whole M phase takes about an hour, which is only a small fraction of the total cell-cycle time. Interphase- the period between one M phase and the next. The cell increases in size. Contains 3 phases. S phase- the cell replicates its DNA. G1 phase and G2 phase- the cell continues to grow. The cell monitors both its internal state and external environment. This monitoring ensures that conditions are suitable for reproduction and that preparations are complete before the cell commits to the major upheavals of S phase (which follows G1) and mitosis (following G2). At particular points, the cell decides whether to process to the next phase or pause to allow more time to prepare.

Explain how M-Cdk is activated at the end of G2 and indicate why this activation is sudden and explosive.

M-Cdk complexes accumulate throughout G2. It isn't switched on until the end of G2, when the activating phosphatase Cdc25 removes the inhibitory phosphates holding M-Cdk activity in check. This act of activation is self-reinforcing: once activated, each M-Cdk complex can indirectly turn on additional M-Cdk complexes. The overall consequence is that, once M-Cdk activation begins, it ignites an explosive increase in M-Cdk activity that drives the cell abruptly from G2 to M phase.

Many features of _______ cells make them suitable for biochemical studies of the cell-cycle control system. For example, the cells are unusually large and are arrested in a _______-like phase. When the cells are triggered to resume cycling, the cell divisions have especially _______ G1 and G2 phases and occur _______. Studies with Xenopus eggs identified a partly purified activity called _______ that drives a resting Xenopus oocyte into M phase. MPF activity was found to _______ during the cell cycle, although the amount of its kinase component, called _______, remained constant. The regulatory component of MPF, called ________, has a ______ effect on MPF activity and plays a part in regulating interactions with its _______s. The components of MPF are evolutionarily ______ from yeast to humans, so that the corresponding human genes are _______ to function in yeast. Able, hexokinase, short, asynchronously, inhibitory, sperm, Cdk, long, steady, conserved, M, stimulatory, cyclin, maturation promoting factor, substrate, divergent, oscillate, synchronously, egg, PI 3-kinase, ubiquitin, fibroblast, regulin, unable, G1, S, uniform, G2

Many features of egg cells make them suitable for biochemical studies of the cell-cycle control system. For example, the cells are unusually large and are arrested in a G2-like phase. When the cells are triggered to resume cycling, the cell divisions have especially short G1 and G2 phases and occur synchronously. Studies with Xenopus eggs identified a partly purified activity called maturation promoting factor that drives a resting Xenopus oocyte into M phase. MPF activity was found to be oscillate during the cell cycle, although the amount of its kinase component, called Cdk, remained constant. The regulatory component of MPF, called cyclin, has a stimulatory effect on MPF activity and plays a part in regulating interactions with its substrates. The components of MPF are evolutionarily conserved from yeast to humans, so that the corresponding human genes are able to function in yeast.

Explain how some antibiotics inhibit the growth of bacteria by interfering with translation.

Many of our most effective antibiotics are compounds that act by inhibiting bacterial, but not eukaryotic, gene expression. Some of these drugs exploit the small structural and functional differences between bacterial and eukaryotic ribosomes, so that they interfere preferentially with bacterial protein synthesis

In addition to its role in DNA repair, homologous recombination is also responsible for generating genetic diversity during what process?

Meiosis

Explain how cells keep the concentration of calcium ions in the cytosol low and how they terminate a calcium ion signal.

Membrane embedded calcium pumps actively remove Ca from cytosol, either to ER or across plasma membrane. This results in steep electrochemical gradients of Ca across both ER & plasma membranes. When signal opens channels, calcium rushes down gradient, this triggers changes in Ca responsive proteins

MicroRNAs are noncoding RNAs that are incorporated into a protein complex called ____________, which searches the ____________s in the cytoplasm for sequence complementary to that of the miRNA. When such a molecule is found, it is then targeted for ___________. RNAi is triggered by the presence of foreign __________ molecules, which are digested by the _________ enzyme into shorter fragments approximately 23 nucleotide pairs in length. Acetylation, methylation, riboswitch, destruction, mitochondria, RISC, Dicer, mRNA, rRNA, DNA, phosphorylation, single-stranded RNA, double-stranded RNA, prokaryotic, tRNA

MicroRNAs are noncoding RNAs that are incorporated into a protein complex called RISC, which searches the mRNAs in the cytoplasm for sequence complementary to that of the miRNA. When such a molecule is found, it is then targeted for destruction. RNAi is triggered by the presence of foreign double-stranded RNA molecules, which are digested by the Dicer enzyme into shorter fragments approximately 23 nucleotide pairs in length.

Microtubules are formed from the tubulin heterodimer, which is composed of the nucleotide-binding ______ protein and the ______ protein. Tubulin dimers are stacked together into protofilaments; ______ parallel protofilaments form the tubelike structure of a microtubule. _______ rings are important for microtubule nucleation and are found in the _______ , which is usually found near the cell's nucleus in cells that are not undergoing mitosis. A microtubule that is quickly growing will have a _______ cap that helps prevent the loss of subunits from its growing end. Stable microtubules are used in cilia and flagella; these microtubules are nucleated from a ______ and involve a "_______ plus two" array of microtubules. The motor protein _______ generates the bending motion in cilia; the lack of this protein can cause Kartagener's syndrome in humans. α-tubulin, dynein, nine, ATP, four, thirteen, basal body, γ-tubulin, twenty-one, β-tubulin, GTP, UTP, centrosome, kinesin two, vimentin, δ-tubulin, myosin

Microtubules are formed from the tubulin heterodimer, which is composed of the nucleotide-binding β-tubulin protein and the α-tubulin protein. Tubulin dimers are stacked together into protofilaments; thirteen parallel protofilaments form the tubelike structure of a microtubule. γ-Tubulin rings are important for microtubule nucleation and are found in the centrosome, which is usually found near the cell's nucleus in cells that are not undergoing mitosis. A microtubule that is quickly growing will have a GTP cap that helps prevent the loss of subunits from its growing end. Stable microtubules are used in cilia and flagella; these microtubules are nucleated from a basal body and involve a "nine plus two" array of microtubules. The motor protein dynein generates the bending motion in cilia; the lack of this protein can cause Kartagener's syndrome in humans

Explain how membrane-enclosed organelles are distributed to daughter cells during cell division.

Mitochondria and chloroplasts are usually present in large numbers and will be safely inherited if, on average, their numbers simply double once each cell cycle. The ER in interphase cells is continuous with the nuclear membrane and is organize by the microtubule cytoskeleton. Upon entry into M phase, the reorganization of the microtubules releases the ER; in most cells, the released ER remains intact during mitosis and is cut in two during cytokinesis .The Golgi apparatus fragments during mitosis; the fragments associate with the spindle microtubules via motor proteins, thereby hitching a ride into the daughter cells as the spindle elongates in anaphase.

Define the function of MAP-Kinase, MAP-Kinase-Kinase and MAP-Kinase-Kinase-Kinase.

Mitogen Activated Protein (MAP) kinase kinase kinase, MAPKKK (or MAP3K) is a serine/threonine-specific protein kinase which acts upon MAP kinase kinase. Subsequently, MAP kinase kinase activates MAP kinase. Several types of MAPKKK can exist but are mainly characterized by the MAP kinases they activate. MAPKKK is responsible for various cell functions such as cell proliferation, cell differentiation, and apoptosis. Because MAPKKKs are involved in a wide range of cell responses occurring both in the cytoplasm and the nucleus, a mutation in these genes can cause several diseases. Over-expression of the MAPKKK upstream of the ERK 1/2 MAPK and an increase in epidermal growth factor receptor (EGFR) can lead to tumor formation, such as triple negative breast cancer.

What is a retrotransposon?

Mobile genetic element

Explain how mobile genetic elements can alter the activity of regulation of a gene or promote gene duplication and exon shuffling.

Mobile genetic elements are DNA sequences that can move from one chromosomal location to another. These cause spontaneous mutations and sometimes novel genes that spread with exon shuffling and gene duplication, like a parasite.

Describe how monomeric GTPases toggle between active and inactive forms.

Monomeric GTPases switch on and off by two sets of regulatory proteins that aid in binding and hydrolyzing GTP GEF - guanine nucleotide exchange factors, activate switch, GDP->GTP GAPs - GTPase activating proteins, inactivate, GTP hydrolysis, GTP->GDP

Explain combinatorial control.

Most importantly is the idea of combinatorial control, which is that any given gene is likely controlled by a specific combination of factors to control transcription. In a hypothetical example, the factors A and B might regulate a distinct set of genes from the combination of factors A and C.

Summarize six basic mechanisms that generate genetic change.

Mutation within a gene, mutation within regulatory DNA sequences, gene duplication and divergence, exon shuffling, transposition of mobile genetic elements, and horizontal gene transfer.

Recall how point mutations typically arise and evaluate how the locations of these mutations can dictate their effects on an organism's appearance or fitness.

Mutations outside the coding sequences of genes can sometimes affect the regulatory DNA sequences, which control the timing, location, and level of gene expression. This can lead to the inhibition of protein production.

Describe an extracellular signal protein that inhibits tissue growth.

Myostatin, for example, is a secreted signal protein that normally inhibits the growth and proliferation of the precursor cells (myoblasts) that fuse to form skeletal muscle cells during mammalian development. When the gene that encodes myostatin is deleted in mice, their muscles grow to be several times larger than normal, because both the number and size of muscle cells is increased.

Compare and contrast necrosis and apoptosis.

Necrosis is the sum of cellular changes that occur after local cell death; it is the process of cellular self-digestion, or autolysis. Cells die long before any necrotic changes are noted by light microscopy. The structural signs that indicate irreversible injury and progression to necrosis are dense clumping and progressive disruption of genetic material and disruption of the plasma and organelle membranes. Apoptosis is an active process of cellular self-destruction implicated in both normal and pathologic tissue changes. When dead cells are observed histologically, necrosis is the correct descriptive term. If the cells have apoptotic structure, then apoptotic necrosis is the correct term.

Describe non-coding DNA. What is it and what function does it have?

Non-coding DNA sequences do not code for amino acids. Most non-coding DNA lies between genes on the chromosome and has no known function. Other non-coding DNA, called introns, is found within genes. Some non-coding DNA plays a role in the regulation of gene expression.

What structure in an interphase eukaryotic cell is the site of ribosomal RNA transCription?

Nucleolus

Once an mRNA is produced, its message can be decoded on ribosomes. The ribosome is composed of two subunits: the ___________ subunit, which catalyzes the formation of the peptide bonds that link the amino acids together into a polypeptide chain, and the __________ subunit, which matches the tRNAs to the codons of the mRNA. During the chain elongation process of translating an mRNA into protein, the growing polypeptide chain attached to a tRNA is bound to the __________ site of the ribosome. An incoming aminoacyl-tRNA carrying the next amino acid in the chain will bind to the _________ site by forming base pairs with the exposed codon in the mRNA. The ___________ enzyme catalyzes the formation of a new peptide bond between the growing polypeptide chain and the newly arriving amino acid. The end of a protein-coding message is signaled by the presence of a stop codon, which binds the __________ called release factor. Eventually, most proteins will be degraded by a large complex of proteolytic enzymes called the _________. A, medium, protein, central, P, RNA, DNA, peptidyl transferase, small, E, polymerase, T, large, proteasome, ubiquitin

Once an mRNA is produced, its message can be decoded on ribosomes. The ribosome is composed of two subunits: the large subunit, which catalyzes the formation of the peptide bonds that link the amino acids together into a polypeptide chain, and the small subunit, which matches the tRNAs to the codons of the mRNA. During the chain elongation process of translating an mRNA into protein, the growing polypeptide chain attached to a tRNA is bound to the P site of the ribosome. An incoming aminoacyl-tRNA carrying the next amino acid in the chain will bind to the A site by forming base pairs with the exposed codon in the mRNA. The peptidyl transferase enzyme catalyzes the formation of a new peptide bond between the growing polypeptide chain and the newly arriving amino acid. The end of a protein-coding message is signaled by the presence of a stop codon, which binds the protein called release factor. Eventually, most proteins will be degraded by a large complex of proteolytic enzymes called the proteasome.

Differentiate between L1 elements and Alu sequences.

One abundant human retrotransposon, the L1 element (LINE-1, a long interspersed nuclear element), is transcribed into RNA by a host cell's RNA polymerase. A double stranded DNA copy of this RNA is then made using an enzyme called reverse transcriptase, an unusual DNA polymerase that can use RNA as a template. The reverse transcriptase is encoded by the L1 element itself. They constitute about 15% of the human genome. The Alu sequence is present in about 1 million copies, making up about 10% of our genome. Alu elements do not encode their own reverse transcriptase and thus depend on enzymes already present in the cell to help them move.

Describe a method to identify proteins that interact in response to stimulation by an extracellular signal.

One method involves using a protein as "bait." For example, to isolate the receptor that binds to insulin, one could attach insulin to a chromatography column. Cells that respond to the hormone are broken open with detergents that disrupt their membranes, releasing the transmembrane receptor proteins. When this slurry is poured over the chromatography column, the proteins that bind to insulin will stick and can later be eluted and identified. Protein-protein interactions in a signaling pathway can also be identified by co-immunoprecipitation. For example, cells exposed to an extracellular signal molecule can be broken open, and antibodies can be used to grab the receptor protein known to recognize the signal molecule. If the receptor is strongly associated with other proteins, these will be captured as well. In this way, researchers can identify which proteins interact when an extracellular signal molecule stimulates cells.

Histones:

One of a small group of abundant, highly conserved proteins around which DNA wraps to form nucleosomes, structures that represent the most fundamental level of chromatin packing

Review how death receptors can stimulate apoptosis.

One well-understood death receptor called Fas, is present on the surface of a variety of mammalian cell types. Fas is activated by a membrane-bound protein called Fas ligand, present on the surface of specialized immune cells called killer lymphocytes. These killer cells can help regulate immune responses by inducing apoptosis in other immune cells that are unwanted or are no longer needed. The binding of Fas ligand to its receptor triggers the assembly of a death-inducing signaling complex, which inclides specific initiator procaspases that, when activated, launch a caspase cascade that leads to cell death.

Which is true of the GTP-binding proteins that participate in intracellular signaling?

Only trimeric GTP-binding proteins relay messages from G-protein-coupled receptors.

Describe how the synthesis and destruction of cyclins regulate progression from one phase of the cell cycle to the next.

Over the course of the cell cycle, the concentration of each type of cyclin rises gradually and then falls abruptly. The gradual increase in cyclin concentration stems from continued transcription of cyclin genes and synthesis of cyclin proteins, whereas the rapid fall in cyclin concentration is precipitated by a full-scale target destruction of the protein. The anaphase-promoting complex or cyclosome (APC/C) tags cyclins with a chain of ubiquitin and they are directed to proteasomes where they are rapidly degraded. The ubiquitination and degradation of the cyclin returns its Cdk to an inactive state. Like cyclin accumulation, cyclin destruction can also help drive the transition from one phase of the cell cycle to the next. For example, M cyclin degradation- and the resulting inactivation of M-Cdk- leads to the molecular events that take the cell out of mitosis.

Review how extracellular signals that promote cell growth and survival activate PI-3-kinase signaling pathways.

PI 3-kinase phosphorylates inositol phospholipids in the plasma membrane. These lipids serve as docking sites for specific intracellular signaling proteins, which relocate from the cytosol to the plasma membrane, where they can activate one another.

What role does securin play in the transition from metaphase to anaphase?

PTTG is a mammalian securin, a key regulator of metaphase to anaphase transition during mitosis, and overexpression or suppression of PTTG results in aneuploidy, and also plays a role in DNA break repair. At the beginning of anaphase, securin is targeted for destruction by APC/C - same protein complex that marks M cyclin for degradation. Once securin is removed, separase is free to sever linkages.

Plasma membrane proteins are inserted into the membrane in the __________. The address information for protein sorting in a eukaryotic cell is contained in the ________ of the proteins. Proteins enter the nucleus in their _______ form. Proteins that remain in the cytosol do not contain a _______. Proteins are transported into the Golgi apparatus via ________. The proteins transported into the endoplasmic reticulum by ________ are in their _______ form. amino acid sequence, Golgi apparatus, sorting signal, endoplasmic reticulum, plasma membrane, transport vesicles, folded protein, translocators, unfolded

Plasma membrane proteins are inserted into the membrane in the endoplasmic reticulum. The address information for protein sorting in a eukaryotic cell is contained in the amino acid sequence of the proteins. Proteins enter the nucleus in their folded form. Proteins that remain in the cytosol do not contain a sorting signal. Proteins are transported into the Golgi apparatus via transport vesicles. The proteins transported into the endoplasmic reticulum by protein translocators are in their unfolded form.

Compare positive and negative feedback and contrast the types of responses produced by each.

Positive feedback is when a component downstream in the pathway acts to enhance the response of the initial signal. (can generate all-or-none) Negative feedback is when a component downstream inhibits the response of an earlier component, diminishing the response. (can generate oscillation responses)

Telomerase:

Prevents eukaryotic chromosomes from shortening with each cell division

The mitotic spindle begins to assemble in which phase of the cell cycle?

Prophase

Describe proteasomes and their regulatory function in the cell.

Proteasomes are large protein machines that break down proteins. They are present in both the cytosol and the nucleus. Proteasomes bind the proteins destined for degradation (marked by the covalent attachment of ubiquitin) and use ATP-hydrolysis to unfold the proteinasnd thread them into the inner chamber of its cylinder to chop them into short peptides.

Proteins are transported out of a cell via the _______ or ______ pathway. Fluids and macromolecules are transported into the cell via the ______ pathway. All proteins being transported out of the cell pass through the _______ and the _______. Transport vesicles link organelles of the ______ system. The formation of _______ in the endoplasmic reticulum stabilizes protein structure. Carbohydrate, Golgi apparatus, disulfide bonds, hydrogen bonds, endocytic, ionic bonds, endomembrane, lysosome, endoplasmic reticulum, protein, endosome, secretory, exocytic

Proteins are transported out of a cell via the secretory or exocytic pathway. Fluid and macromolecules are transported into the cell via the endocytic pathway. All proteins being transported out of the cell pass through the endoplasmic reticulum and the Golgi apparatus. Transport vesicles link organelles of the endomembrane system. The formation of disulfide bonds in the endoplasmic reticulum stabilizes protein structure

Please explain the function of the polyubiquitin targeting. Be complete and precise.

Proteins that are meant to be short-lived often contain a short amino acid sequence that identifies the protein as one to be ubiquitylated and degraded in proteasomes. Damaged or misfolded proteins, as well as proteins containing oxidized or otherwise abnormal amino acids, are also recognized and degraded by this ubiquitin-dependent proteolytic system. The enzymes that add a polyubiquitin chain to such proteins recognize signals that become exposed on these proteins as a result of the misfolding or chemical damage—for example, amino acid sequences or conformational motifs that are typically buried and inaccessible in a "healthy" protein. Proteins marked by a polyubiquitin chain are degraded by the proteasome.

Explain how purifying selection leads to the conservation of functionally important DNA sequences and characterize the roles that these conserved sequences might play.

Purifying selection is the elimination of individuals carrying mutations that interfere with important functions. By observing where the human and mouse sequences have remained nearly the same, one can thus see very clearly the regions where genetic changes are not tolerated. these sequences have been conserved by purifying selection.

Summarize how phosphorylation can act as a molecular switch, and identify the types of proteins that add and remove this chemical modification.

Signaling by protein phosphorylation - the signal protein is off until a protein kinase phosphorylates it through ATP hydrolysis. The addition of the phosphate to the protein switches it "on", sending out a signal. Protein phosphatase hydrolyzes the protein, removing the phosphate, switching it "off". Many switch proteins are protein kinases - phosphorylation cascades. One PK phosphorylates the next in a sequence. This amplifies, distributes, and regulates the transmission of the signal. Protein kinases 2 main types: serine/threonine kinases (phosphorylate proteins on serines and threonines) and tyrosine kinases (phosphorylate proteins of tyrosines)

Predict whether you would expect the cell's size to be bigger or smaller than normal cells if cells were treated in the following fashion. A. You change threonine 42 on TFS to an alanine residue. B. You create a version of the receptor that is constitutively active. C. You add a drug that inhibits adenylyl cyclase. D. You add a drug that increases the activity of cyclic AMP phosphodiesterase. E. You mutate the cAMP-binding sites in the regulatory subunits of PKA, so that the complex binds cAMP more tightly.

Smaller, bigger, smaller, smaller, bigger

List the three specialized DNA sequences and explain their functions.

Specialized DNA sequences are required for DNA replication and chromosome segragation. Replication Origins: Controls the beginning of DNA synthesis. Centromere: permit one copy each of the duplicated chromosomes to be pulled into each daughter cell at division. Telomere: allow ends to be efficiently replicated and also prevent chromosome ends from being recognized as breaks in need of repair.

Starting with a single diploid cell, mitosis produces [two/four] [identical/different] [haploid/diploid] cells, whereas meiosis yields [two/four] [identical/different] [haploid/diploid] cells. This is accomplished in meiosis because a single round of chromosome [replication/segregation] is followed by two sequential rounds of [replication/segregation]. Mitosis is more like meiosis [I/II] than meiosis [I/II]. In meiosis I, the kinetochores on sister chromatids behave [independently/coordinately] and thus attach to microtubules from the [same/opposite] spindle. The cohesin-mediated glue between [chromatids/homologs] is regulated differently near the centromeres than along the chromosome arms. Cohesion is lost first at the [centromeres/arms] to allow segregation of [chromatids/homologs] and is lost later at the [centromeres/arms] to trigger segregation of [chromatids/homologs].

Starting with a single diploid cell, mitosis produces two identical diploid cells, whereas meiosis yields four different haploid cells. This is accomplished in meiosis because a single round of chromosome replication is followed by two sequential rounds of segregation. Mitosis is more like meiosis II than meiosis I. In meiosis I, the kinetochores on sister chromatids behave coordinately and thus attach to microtubules from the same spindle. The cohesin-mediated glue between chromatids is regulated differently near the centromeres than along the chromosome arms. Cohesion is lost first at the arms to allow segregation of homologs and is lost later at the centromeres to trigger segregation of chromatids

Describe how the ends of eukaryotic chromosomes (telomeres) are replicated.

Telomerase adds complementary RNA bases to the 3′ end of the DNA strand. Once the 3′ end of the lagging strand template is sufficiently elongated, DNA polymerase adds the complementary nucleotides to the ends of the chromosomes; thus, the ends of the chromosomes are replicated.

What is the Bad-protein how does it function, when phosphorilated, when dephosphorilated?

The BCL2 associated agonist of cell death (BAD) protein is a pro-apoptotic member of the Bcl-2 gene family which is involved in initiating apoptosis. BAD is a member of the BH3-only family, a subfamily of the Bcl-2 family. Dephosphorylated BAD forms a heterodimer with Bcl-2 and Bcl-xL, inactivating them and thus allowing Bax/Bak-triggered apoptosis. When BAD is phosphorylated by Akt/protein kinase B (triggered by PIP3), it forms the BAD-(14-3-3) protein heterodimer. This leaves Bcl-2 free to inhibit Bax-triggered apoptosis. BAD phosphorylation is thus anti-apoptotic, and BAD dephosphorylation (e.g., by Ca2+-stimulated Calcineurin) is pro-apoptotic. The latter may be involved in neural diseases such as schizophrenia.

Which of these represents a mechanism used to terminate a signal transmitted by activated RTKs?

The RTKs are internalized and digested in lysosomes.

Compare and contrast GRCR and RTKs (receptors).

The ability of a single ligand-binding event to trigger so many pathways is a key difference between receptor tyrosine kinases and G protein-coupled receptors. Notice that each has an extracellular ligand-binding site, an "alpha" helix spanning the membrane, and an intracellular tail containing multiple tyrosines.

What is the order in which the following events occur during cell division? A.anaphase B.metaphase C.prometaphase D.telophase E.mitosis F.prophase. Where does cytokinesis fit in?

The order is F, C, B, A, D. Together, these five steps are referred to as mitosis. Cytokinesis is the last step in M phase, which overlaps with anaphase and telophase. Mitosis and cytokinesis are both part of M phase.

Describe the role of chromosomes in assembly of the mitotic spindle.

The assembly of a mitotic spindle requires the interaction of microtubules with chromosomes. As a cell enters mitosis, long microtubules are converted to short ones, as microtubules become unstable. Dynamic microtubules are then stabilised by chromosomes, forming a bipolar spindle.

Explain why studies of yeast can lead to insights into the biology of human cancers.

The basic organization of the cycle is essentially the same in that all eukaryotes appear to use similar machinery and control mechanisms to drive and regulate cell-cycle events. The proteins of the cell-cycle control system first appeared more than a billion years ago, and they have been so well conserved over the course of evolution that many of them function perfectly when transferred from a human cell to a yeast.

How does Cholera toxin function, be precise

The catalytic portion of cholera toxin performs a single function: it seeks out the G proteins used for cellular signaling and attaches an ADP molecule to them. This converts the G-protein into a permanently active state, so it sends a never-ending signal.

The cell cycle consists of an alternation between _______, which appears as a period of dramatic activity under the microscope, and a preparative period called ______, which consists of three phases called _______, ______, and _______. During M phase, the nucleus divides in a process called _______, and the cytoplasm splits in two in a process called _______. The cell-cycle control system relies on sharp increases in the activities of regulatory proteins called _______, or ______, to trigger S phase and M phase. Inactivation of _______ is required to exit from M phase after chromosome segregation. APC, G2 phase, metaphase, Cdks, interphase, microtubules, condensation, intraphase, mitosis, cyclin-dependent kinases, kinesins, myosins, cytokinesis, M phase, S phase, G1 phase, M-Cdk, S-Cdk, G1-Cdk, meiosis

The cell cycle consists of an alternation between M phase, which appears as a period of dramatic activity under the microscope, and a preparative period called interphase, which consists of three phases called G1 phase, S phase, and G2 phase. During M phase, the nucleus divides in a process called mitosis, and the cytoplasm splits in two in a process called cytokinesis. The cell-cycle control system relies on sharp increases in the activities of regulatory proteins called cyclin-dependent kinases, or Cdks, to trigger S phase and M phase. Inactivation of M-Cdk is required to exit from M phase after chromosome segregation.

Contrast cut-and-paste and replicative transposition for DNA-only transposons.

The cut-and-paste mechanism occurs when the element is excised from the genome and inserted into a different site. Replicative transposition replicates first, and the new copy of the transposon inserts into a second chromosomal site, leaving the original intact.

Recall why different cyclin-Cdk complexes trigger different events in the cell cycle.

The cyclin that acts in G2 to trigger entry into M phase is called the M cyclin, and the active complex it forms with its Cdk is called M-Cdk. Other cyclins, called S cyclins and G1/S cyclins, bidn to a distinct Cdk protein late in G1 to form S-Cdk and G1/S-Cdk, respectively; these cyclin-Cdk complexes help launch S phase. Another group of cyclins, called G1 cyclins, act earlier in G1 and bind to other Cdk proteins to form G1-Cdks, which help drive the cell through G1 toward S phase.

Recall how dephosphorylation helps trigger the abrupt activation of cyclin-Cdk complexes.

The cyclin-Cdk complex contains inhibitory phosphates, and to become active, the Cdk must be dephosphorylated by a specific protein phosphatase. Thus protein kinases and phosphatases act together to regulate the activity of specific cyclin-Cdk complexes and help control progression through the cell cycle.

The genes of a bacterial ______________ are transcribed into a single mRNA. Many bacterial promoters contain a region known as a(n) _____________, to which a specific transcription regulator binds. Genes in which transcription is prevented are said to be _____________. The interaction of small molecules, such as tryptophan, with ______________ DNA-binding proteins, such as the tryptophan repressor, regulates bacterial genes. Genes that are being ____________ expressed are being transcribed all the time. Allosteric, negatively, positively, constitutively, operator, promoter, induced, operon, repressed

The genes of a bacterial operon are transcribed into a single mRNA. Many bacterial promoters contain a region known as an operator, to which a specific transcription regulator binds. Genes in which transcription is prevented are said to be repressed. The interaction of small molecules, such as tryptophan, with allosteric DNA-binding proteins, such as the tryptophan repressor, regulates bacterial genes. Genes that are being constitutively expressed are being transcribed all the time.

Review the structure of genes in the genomes of human, fly, and yeast, and compare how densely genes are distributed in the chromosomes of each organism.

The human genome is less dense with genes. The genes of a fly and yeast are more compact with fewer introns.

The instructions specified by the DNA will ultimately specify the sequence of proteins. This process involves DNA, made up of ____ different nucleotides, which gets ____________ into RNA, which is then _____________ into proteins, made up of _____ different amino acids. In eukaryotic cells, DNA gets made into RNA in the ____________, while proteins are produced from RNA in the _________________. The segment of DNA called a_______________ is the portion that is copied into RNA; this process is catalyzed by RNA ______________. 4, gene, proteasome, 20, Golgi, replisome, 109, kinase, sugar-phosphate, 128, nuclear pore, transcribed, cytoplasm, nucleus, transferase, exported, polymerase, translated

The instructions specified by the DNA will ultimately specify the sequence of proteins. This process involves DNA, made up of 4 different nucleotides, which gets transcribed into RNA, which is then translated into proteins, made up of 20 different amino acids. In eukaryotic cells, DNA gets made into RNA in the nucleus, while proteins are produced from RNA in the cytoplasm. The segment of DNA called a gene is the portion that is copied into RNA; this process is catalyzed by RNA polymerase.

Describe the organization of the lac operon and how it is under negative and positive control.

The lac operon exhibits both systems. It is a negative control system because expression is typically blocked by an active repressor (the lac repressor) that turns off transcription. However, when CAP (catabolite gene activating protein) binds upstream of this operator region near the promoter and transcription increases, this is an example of a positive control system. We see this positive control of transcription happen when glucose levels decline.

Indicate which phases of the cell cycle vary the most in length to influence the rates of cell division in the adult body.

The length of S phase varies according to the total DNA that the particular cell contains; the rate of synthesis of DNA is fairly constant between cells and species. Usually, cells will take between 5 and 6 hours to complete S phase. G2 is shorter, lasting only 3 to 4 hours in most cells.

What does depurination refer to?

The loss of A or G bases from DNA

Describe the structure of the mitotic spindle and explain how and when it begins to form.

The mitotic spindle begins to form in prophase (first stage of mitosis, during which the duplicated chromosomes condense and spindle forms). The assembly of this structure depends on microtubules. As stated before, microtubules continuously polymerize and depolymerize by the addition and loss of their tubulin subunits, and individual filaments alternate between growing and shrinking (dynamic instability). At the start of mitosis, dynamic stability rises - in part because M-Cdk phosphorylates microtubule-associated proteins that influence microtubule stability.

Contrast the differences between the chimp and human genomes with those between the human and mouse genomes.

The mobile genetic elements found in mouse and human DNA, although similar in nucleotide sequence, are distributed differently, as they have had more time to proliferate and move around the two genomes after these species diverged. The large scale- organization of the human and mouse genomes have been scrambled by many episodes of chromosome breakage and recombination over the past 75 million years: it is estimated that about 180 such "break and join" events have dramatically altered chromosome organization. For example, in humans most centromeres lie near the middle of the chromosome, whereas those of mice are located at the chromosome ends.

Evaluate how RNA sequencing can provide a more accurate estimate of the number of genes in a genome than DNA sequencing.

The number of predicted protein-coding genes has dropped because the technique detects only those genes that are actively transcribed. The approach allowed the detection of genes that do not code for proteins, but instead encode functional or regulatory RNAs

The survival, _______, and size of each cell in an animal are controlled by extracellular signal molecules secreted by neighboring and distant cells. Many of these signal molecules bind to a cell-surface ______ and trigger various intracellular signaling pathways. One class of signal molecules, called _______, stimulates cell division by releasing the molecular brakes that keep cells in the _______ or ______ phase of the cell cycle. Members of a second class of signal molecules are called _______, because they stimulate cell growth and an increase in cell mass. The third class of signal molecules, called _______, inhibits _______ by regulating members of the ______ family of proteins. In addition to such stimulatory factors, some signal proteins such as ______ act negatively on other cells, inhibiting their survival, growth, or proliferation. Anaphase, differentiation, myostatin, annihilation, G0, nourishment, apoptosis, G1, nutrition, arrestase, G2, phosphatases, Bcl2, growth factors, proliferation, biosynthetic, interphase, receptor, cascades, ligand, S, caspase, M, survival factors, Cdk, mitogens, transcription, cyclin

The survival, proliferation, and size of each cell in an animal are controlled by extracellular signal molecules secreted by neighboring and distant cells. Many of these signal molecules bind to a cell-surface receptor and trigger various intracellular signaling pathways. One class of signal molecules, called mitogens, stimulates cell division by releasing the molecular brakes that keep cells in the G0 or G1 phase of the cell cycle. Members of a second class of signal molecules are called growth factors, because they stimulate cell growth and an increase in cell mass. The third class of signal molecules, called survival factors, inhibits apoptosis by regulating members of the Bcl2 family of proteins. In addition to such stimulatory factors, some signal proteins such as myostatin act negatively on other cells, inhibiting their survival, growth, or proliferation

The transmission of information important for gene regulation from parent to daughter cell, without altering the actual nucleotide sequence, is called ___________ inheritance. This type of inheritance is seen with the inheritance of the covalent modifications on ____________ proteins bound to DNA; these modifications are important for reestablishing the pattern of chromatin structure found on the parent chromosome. Another way to inherit chromatin structure involves DNA __________, a covalent modification that occurs on cytosine bases that typically turns off the transcription of a gene. Gene transcription patterns can also be transmitted across generations through positive _____________ loops that can involve a transcription regulator activating its own transcription in addition to other genes. These mechanisms all allow for cell ________________, a property involving the maintenance of gene expression patterns important for cell identity. Combinatorial, feedback, phosphorylation, deacetylation, histone, pluripotency, differential, leucine zipper, proliferation, epigenetic, memory, receptor, expression, methylation, unwinding

The transmission of information important for gene regulation from parent to daughter cell, without altering the actual nucleotide sequence, is called epigenetic inheritance. This type of inheritance is seen with the inheritance of the covalent modifications on histone proteins bound to DNA; these modifications are important for reestablishing the pattern of chromatin structure found on the parent chromosome. Another way to inherit chromatin structure involves DNA methylation, a covalent modification that occurs on cytosine bases that typically turns off the transcription of a gene. Gene transcription patterns can also be transmitted across generations through positive feedback loops that can involve a transcription regulator activating its own transcription in addition to other genes. These mechanisms all allow for cell memory, a property involving the maintenance of gene expression patterns important for cell identity.

Summarize how cells change in shape and attachment throughout the cell cycle.

These changes result, in part, from the reorganization of actin and myosin filaments in the cell cortex, only one aspect of which is the assembly of the contractile ring. Mammalian fibroblasts in culture, for example, spread out flat during interphase, as a result of the strong adhesive contacts they make with the surface they are growing on. The change in shape takes part because some of the plasma membrane proteins responsible for attaching the cells to the substratum become phosphorylated and thus weaken their grip.

Review the main functions of an intracellular signaling pathway and identify the steps at which each can take place.

They can relay the signal to help it spread · Amplification of signal received, makes it stronger; few molecules can evoke a large response · Detect signals from multiple intracellular signaling pathways and integrate them · Distribution of the signal to multiple effector proteins; complex response · Module the response of signal by regulating the activity of upstream components via feedback

Explain why chromosomes align at the spindle equator during metaphase, and present experimental evidence for this mechanism.

They form halfway between the two spindle poles, forming the metaphase plate. This event defines the beginning of metaphase. Both the continual growth and shrinkage of the microtubules and the action of the microtubule motor proteins are required.

What do the phosphorylated tyrosines on activated RTKs do?

They serve as binding sites for a variety of intracellular signaling proteins

Describe the type of signal transduction carried out by ion-channel-coupled receptors.

They transduce a chemical signal into an electrical signal. Neurotransmitter binds to ICCR on the target cell surface, the receptor's confirmation is altered, opening a channel on the target cell membrane for specific types of ions. Without the neurotransmitter signal bound to the receptor, the ion gated channel is closed. When open, the ions migrate based on their electrochemical gradients, from high concentration to low concentration. This changes the membrane potential (very fast, milliseconds) Membrane potential change might trigger nerve impulse or alter the ability (easier or harder) for other neurotransmitters to do so.

Detail how S-Cdk initiates replication and prevents re-replication during the same cell cycle.

This kinase phosphorylates helicase, activating it. It also promotes the assembly of proteins at the replication fork. It prevents re-replication by inactivating cdc6 or ORC.

Summarize the function of the cell-cycle control system and describe the transition points at which progression through the cycle is regulated.

This system guarantees that the events of the cell cycle- DNA replication, mitosis, and so on- occur in a set sequence and that each process has been completed before the next one begins. At the transition from G1 to S phase, the control system conforms that the environment is favorable for proliferation before committing to DNA replication. At the transition from G2 to M phase, the control system confirms that the DNA is undamaged and fully replicated, ensuring that the cell does not enter mitosis unless its DNA is intact. Finally, during mitosis, the cell-cycle control machinery ensures that the duplicated chromosomes are properly attached to a cytoskeletal machine, called the mitotic spindle, before the spindle pulls the chromosomes apart and segregates them into the two daughter cells.

In the DNA of certain bacterial cells, 13% of the nucleotides contain adenine. What are the percentages of the other nucleotides?

Thymine= 13% Adenine= 13% Guanine= 37% Cytosine= 37%


Kaugnay na mga set ng pag-aaral

Introduction to Statistics: Chapter 4 Homework (Regression Analysis: Exploring Associations Between Variables)

View Set

Financial Markets and Institutions: Chapter 1, Chapter 2, Chapter 3

View Set

Respiratory Practice Questions 2.0

View Set

Essentials of Communication - Unit 3, Lesson 15: E-mail Etiquette

View Set

Chapter 21: Allergy and Anaphylaxis:

View Set

TestOut Network Pro 11.8 Remote Management

View Set

Chapter 22: Assessment of Integumentary System

View Set